import work from year 2016-2017
This commit is contained in:
78
3e/DM/DM_16_09_15/01_DM_16_09_15.tex
Normal file
78
3e/DM/DM_16_09_15/01_DM_16_09_15.tex
Normal file
@@ -0,0 +1,78 @@
|
||||
\documentclass[a5paper,12pt, table]{/media/documents/Cours/Prof/Enseignements/2016-2017/tools/style/classDS}
|
||||
\usepackage{/media/documents/Cours/Prof/Enseignements/2016-2017/theme}
|
||||
\geometry{left=10mm,right=10mm, top=10mm, bottom=10mm}
|
||||
|
||||
% Title Page
|
||||
\titre{1}
|
||||
% \seconde \premiereS \PSTMG \TSTMG
|
||||
\classe{Troisième}
|
||||
\date{15 septembre 2016}
|
||||
%\duree{1 heure}
|
||||
\sujet{01}
|
||||
% DS DSCorr DM DMCorr Corr
|
||||
\typedoc{DM}
|
||||
|
||||
%\printanswers
|
||||
|
||||
\begin{document}
|
||||
|
||||
\maketitle
|
||||
|
||||
Vous devez rendre le sujet avec la copie.
|
||||
|
||||
\begin{questions}
|
||||
|
||||
\question
|
||||
Faire les calculs suivants en détaillant des étapes.
|
||||
\begin{multicols}{2}
|
||||
\begin{parts}
|
||||
|
||||
\part $-1 - 4 \times 4$
|
||||
|
||||
\part $6 - ( -9 \times 10 )$
|
||||
|
||||
\part $-3 \times 4 - 8 \times 2$
|
||||
|
||||
\part $( -2 - 1 ) \times ( -9 ) + 5$
|
||||
|
||||
\part $-1 ( 10 + 9 ) \times 6$
|
||||
|
||||
\part $-3 ( 6 - 5 ) + 2$
|
||||
\end{parts}
|
||||
\end{multicols}
|
||||
\vfill
|
||||
|
||||
\question
|
||||
|
||||
|
||||
Dans un sac, il y a 18 bonbons à la menthe, 81 bonbons à la fraise et 4 au chocolat. On choisit un bonbon au hasard dans ce sac.
|
||||
\begin{parts}
|
||||
\part Combien y a-t-il d'issues en tout?
|
||||
\begin{solution}
|
||||
Il y a 103 bonbons.
|
||||
\end{solution}
|
||||
\part Calculer la probabilité de tirer un bonbon à la fraise.
|
||||
\begin{solution}
|
||||
$T($ tirer un bonbon à la fraise $) = \dfrac{18}{103}$
|
||||
\end{solution}
|
||||
\part Calculer la probabilité de tirer un bonbon qui n'est pas au chocolat.
|
||||
\begin{solution}
|
||||
|
||||
$T($ tirer un bonbon à la fraise $) = \dfrac{99}{103}$
|
||||
\end{solution}
|
||||
\part Calculer la probabilité de tirer un bonbon au réglisse.
|
||||
\begin{solution}
|
||||
$T($ tirer un bonbon au réglisse $) = \dfrac{0}{103} = 0$
|
||||
\end{solution}
|
||||
\part Dans un autre sac, on place 25 bonbons à la menthe et 34 bonbons à la fraise. Lise préfère les bonbons à la menthe. Dans quel sac doit-elle tirer un bonbon pour avoir le plus de chance d'avoir un bonbon qu'elle préfère?
|
||||
\end{parts}
|
||||
|
||||
\vfill
|
||||
\end{questions}
|
||||
|
||||
\end{document}
|
||||
|
||||
%%% Local Variables:
|
||||
%%% mode: latex
|
||||
%%% TeX-master: "master"
|
||||
%%% End:
|
78
3e/DM/DM_16_09_15/02_DM_16_09_15.tex
Normal file
78
3e/DM/DM_16_09_15/02_DM_16_09_15.tex
Normal file
@@ -0,0 +1,78 @@
|
||||
\documentclass[a5paper,12pt, table]{/media/documents/Cours/Prof/Enseignements/2016-2017/tools/style/classDS}
|
||||
\usepackage{/media/documents/Cours/Prof/Enseignements/2016-2017/theme}
|
||||
\geometry{left=10mm,right=10mm, top=10mm, bottom=10mm}
|
||||
|
||||
% Title Page
|
||||
\titre{1}
|
||||
% \seconde \premiereS \PSTMG \TSTMG
|
||||
\classe{Troisième}
|
||||
\date{15 septembre 2016}
|
||||
%\duree{1 heure}
|
||||
\sujet{02}
|
||||
% DS DSCorr DM DMCorr Corr
|
||||
\typedoc{DM}
|
||||
|
||||
%\printanswers
|
||||
|
||||
\begin{document}
|
||||
|
||||
\maketitle
|
||||
|
||||
Vous devez rendre le sujet avec la copie.
|
||||
|
||||
\begin{questions}
|
||||
|
||||
\question
|
||||
Faire les calculs suivants en détaillant des étapes.
|
||||
\begin{multicols}{2}
|
||||
\begin{parts}
|
||||
|
||||
\part $5 - 10 \times 9$
|
||||
|
||||
\part $4 - ( -3 \times 10 )$
|
||||
|
||||
\part $-4 \times 3 + 1 \times ( -10 )$
|
||||
|
||||
\part $( 3 - 8 ) \times 6 - 5$
|
||||
|
||||
\part $3 ( 3 - 7 ) \times 5$
|
||||
|
||||
\part $1 ( -1 - ( -7 ) ) - 3$
|
||||
\end{parts}
|
||||
\end{multicols}
|
||||
\vfill
|
||||
|
||||
\question
|
||||
|
||||
|
||||
Dans un sac, il y a 63 bonbons à la menthe, 18 bonbons à la fraise et 7 au chocolat. On choisit un bonbon au hasard dans ce sac.
|
||||
\begin{parts}
|
||||
\part Combien y a-t-il d'issues en tout?
|
||||
\begin{solution}
|
||||
Il y a 88 bonbons.
|
||||
\end{solution}
|
||||
\part Calculer la probabilité de tirer un bonbon à la fraise.
|
||||
\begin{solution}
|
||||
$T($ tirer un bonbon à la fraise $) = \dfrac{63}{88}$
|
||||
\end{solution}
|
||||
\part Calculer la probabilité de tirer un bonbon qui n'est pas au chocolat.
|
||||
\begin{solution}
|
||||
|
||||
$T($ tirer un bonbon à la fraise $) = \dfrac{81}{88}$
|
||||
\end{solution}
|
||||
\part Calculer la probabilité de tirer un bonbon au réglisse.
|
||||
\begin{solution}
|
||||
$T($ tirer un bonbon au réglisse $) = \dfrac{0}{88} = 0$
|
||||
\end{solution}
|
||||
\part Dans un autre sac, on place 25 bonbons à la menthe et 34 bonbons à la fraise. Lise préfère les bonbons à la menthe. Dans quel sac doit-elle tirer un bonbon pour avoir le plus de chance d'avoir un bonbon qu'elle préfère?
|
||||
\end{parts}
|
||||
|
||||
\vfill
|
||||
\end{questions}
|
||||
|
||||
\end{document}
|
||||
|
||||
%%% Local Variables:
|
||||
%%% mode: latex
|
||||
%%% TeX-master: "master"
|
||||
%%% End:
|
78
3e/DM/DM_16_09_15/03_DM_16_09_15.tex
Normal file
78
3e/DM/DM_16_09_15/03_DM_16_09_15.tex
Normal file
@@ -0,0 +1,78 @@
|
||||
\documentclass[a5paper,12pt, table]{/media/documents/Cours/Prof/Enseignements/2016-2017/tools/style/classDS}
|
||||
\usepackage{/media/documents/Cours/Prof/Enseignements/2016-2017/theme}
|
||||
\geometry{left=10mm,right=10mm, top=10mm, bottom=10mm}
|
||||
|
||||
% Title Page
|
||||
\titre{1}
|
||||
% \seconde \premiereS \PSTMG \TSTMG
|
||||
\classe{Troisième}
|
||||
\date{15 septembre 2016}
|
||||
%\duree{1 heure}
|
||||
\sujet{03}
|
||||
% DS DSCorr DM DMCorr Corr
|
||||
\typedoc{DM}
|
||||
|
||||
%\printanswers
|
||||
|
||||
\begin{document}
|
||||
|
||||
\maketitle
|
||||
|
||||
Vous devez rendre le sujet avec la copie.
|
||||
|
||||
\begin{questions}
|
||||
|
||||
\question
|
||||
Faire les calculs suivants en détaillant des étapes.
|
||||
\begin{multicols}{2}
|
||||
\begin{parts}
|
||||
|
||||
\part $-10 + 7 \times 10$
|
||||
|
||||
\part $5 - 9 \times 3$
|
||||
|
||||
\part $-7 \times ( -5 ) + 4 \times 8$
|
||||
|
||||
\part $( -1 - 5 ) \times ( -2 ) + 4$
|
||||
|
||||
\part $5 ( 9 + 9 ) \times 9$
|
||||
|
||||
\part $9 ( -3 - 4 ) - 10$
|
||||
\end{parts}
|
||||
\end{multicols}
|
||||
\vfill
|
||||
|
||||
\question
|
||||
|
||||
|
||||
Dans un sac, il y a 40 bonbons à la menthe, 16 bonbons à la fraise et 6 au chocolat. On choisit un bonbon au hasard dans ce sac.
|
||||
\begin{parts}
|
||||
\part Combien y a-t-il d'issues en tout?
|
||||
\begin{solution}
|
||||
Il y a 62 bonbons.
|
||||
\end{solution}
|
||||
\part Calculer la probabilité de tirer un bonbon à la fraise.
|
||||
\begin{solution}
|
||||
$T($ tirer un bonbon à la fraise $) = \dfrac{40}{62}$
|
||||
\end{solution}
|
||||
\part Calculer la probabilité de tirer un bonbon qui n'est pas au chocolat.
|
||||
\begin{solution}
|
||||
|
||||
$T($ tirer un bonbon à la fraise $) = \dfrac{56}{62}$
|
||||
\end{solution}
|
||||
\part Calculer la probabilité de tirer un bonbon au réglisse.
|
||||
\begin{solution}
|
||||
$T($ tirer un bonbon au réglisse $) = \dfrac{0}{62} = 0$
|
||||
\end{solution}
|
||||
\part Dans un autre sac, on place 25 bonbons à la menthe et 34 bonbons à la fraise. Lise préfère les bonbons à la menthe. Dans quel sac doit-elle tirer un bonbon pour avoir le plus de chance d'avoir un bonbon qu'elle préfère?
|
||||
\end{parts}
|
||||
|
||||
\vfill
|
||||
\end{questions}
|
||||
|
||||
\end{document}
|
||||
|
||||
%%% Local Variables:
|
||||
%%% mode: latex
|
||||
%%% TeX-master: "master"
|
||||
%%% End:
|
78
3e/DM/DM_16_09_15/04_DM_16_09_15.tex
Normal file
78
3e/DM/DM_16_09_15/04_DM_16_09_15.tex
Normal file
@@ -0,0 +1,78 @@
|
||||
\documentclass[a5paper,12pt, table]{/media/documents/Cours/Prof/Enseignements/2016-2017/tools/style/classDS}
|
||||
\usepackage{/media/documents/Cours/Prof/Enseignements/2016-2017/theme}
|
||||
\geometry{left=10mm,right=10mm, top=10mm, bottom=10mm}
|
||||
|
||||
% Title Page
|
||||
\titre{1}
|
||||
% \seconde \premiereS \PSTMG \TSTMG
|
||||
\classe{Troisième}
|
||||
\date{15 septembre 2016}
|
||||
%\duree{1 heure}
|
||||
\sujet{04}
|
||||
% DS DSCorr DM DMCorr Corr
|
||||
\typedoc{DM}
|
||||
|
||||
%\printanswers
|
||||
|
||||
\begin{document}
|
||||
|
||||
\maketitle
|
||||
|
||||
Vous devez rendre le sujet avec la copie.
|
||||
|
||||
\begin{questions}
|
||||
|
||||
\question
|
||||
Faire les calculs suivants en détaillant des étapes.
|
||||
\begin{multicols}{2}
|
||||
\begin{parts}
|
||||
|
||||
\part $8 + 4 \times 7$
|
||||
|
||||
\part $10 - 8 \times 10$
|
||||
|
||||
\part $-9 \times ( -5 ) + 1 \times 6$
|
||||
|
||||
\part $( -3 + 4 ) \times 9 - 4$
|
||||
|
||||
\part $-7 ( 10 + 7 ) \times 3$
|
||||
|
||||
\part $7 ( -9 - 3 ) - 4$
|
||||
\end{parts}
|
||||
\end{multicols}
|
||||
\vfill
|
||||
|
||||
\question
|
||||
|
||||
|
||||
Dans un sac, il y a 60 bonbons à la menthe, 100 bonbons à la fraise et 7 au chocolat. On choisit un bonbon au hasard dans ce sac.
|
||||
\begin{parts}
|
||||
\part Combien y a-t-il d'issues en tout?
|
||||
\begin{solution}
|
||||
Il y a 167 bonbons.
|
||||
\end{solution}
|
||||
\part Calculer la probabilité de tirer un bonbon à la fraise.
|
||||
\begin{solution}
|
||||
$T($ tirer un bonbon à la fraise $) = \dfrac{60}{167}$
|
||||
\end{solution}
|
||||
\part Calculer la probabilité de tirer un bonbon qui n'est pas au chocolat.
|
||||
\begin{solution}
|
||||
|
||||
$T($ tirer un bonbon à la fraise $) = \dfrac{160}{167}$
|
||||
\end{solution}
|
||||
\part Calculer la probabilité de tirer un bonbon au réglisse.
|
||||
\begin{solution}
|
||||
$T($ tirer un bonbon au réglisse $) = \dfrac{0}{167} = 0$
|
||||
\end{solution}
|
||||
\part Dans un autre sac, on place 25 bonbons à la menthe et 34 bonbons à la fraise. Lise préfère les bonbons à la menthe. Dans quel sac doit-elle tirer un bonbon pour avoir le plus de chance d'avoir un bonbon qu'elle préfère?
|
||||
\end{parts}
|
||||
|
||||
\vfill
|
||||
\end{questions}
|
||||
|
||||
\end{document}
|
||||
|
||||
%%% Local Variables:
|
||||
%%% mode: latex
|
||||
%%% TeX-master: "master"
|
||||
%%% End:
|
78
3e/DM/DM_16_09_15/05_DM_16_09_15.tex
Normal file
78
3e/DM/DM_16_09_15/05_DM_16_09_15.tex
Normal file
@@ -0,0 +1,78 @@
|
||||
\documentclass[a5paper,12pt, table]{/media/documents/Cours/Prof/Enseignements/2016-2017/tools/style/classDS}
|
||||
\usepackage{/media/documents/Cours/Prof/Enseignements/2016-2017/theme}
|
||||
\geometry{left=10mm,right=10mm, top=10mm, bottom=10mm}
|
||||
|
||||
% Title Page
|
||||
\titre{1}
|
||||
% \seconde \premiereS \PSTMG \TSTMG
|
||||
\classe{Troisième}
|
||||
\date{15 septembre 2016}
|
||||
%\duree{1 heure}
|
||||
\sujet{05}
|
||||
% DS DSCorr DM DMCorr Corr
|
||||
\typedoc{DM}
|
||||
|
||||
%\printanswers
|
||||
|
||||
\begin{document}
|
||||
|
||||
\maketitle
|
||||
|
||||
Vous devez rendre le sujet avec la copie.
|
||||
|
||||
\begin{questions}
|
||||
|
||||
\question
|
||||
Faire les calculs suivants en détaillant des étapes.
|
||||
\begin{multicols}{2}
|
||||
\begin{parts}
|
||||
|
||||
\part $1 + 7 \times ( -6 )$
|
||||
|
||||
\part $-4 - ( -7 \times 7 )$
|
||||
|
||||
\part $2 \times 5 - 6 \times 5$
|
||||
|
||||
\part $( -10 - 8 ) \times 3 + 10$
|
||||
|
||||
\part $2 ( -1 + 6 ) \times ( -3 )$
|
||||
|
||||
\part $6 ( -3 - 6 ) - 7$
|
||||
\end{parts}
|
||||
\end{multicols}
|
||||
\vfill
|
||||
|
||||
\question
|
||||
|
||||
|
||||
Dans un sac, il y a 14 bonbons à la menthe, 63 bonbons à la fraise et 9 au chocolat. On choisit un bonbon au hasard dans ce sac.
|
||||
\begin{parts}
|
||||
\part Combien y a-t-il d'issues en tout?
|
||||
\begin{solution}
|
||||
Il y a 86 bonbons.
|
||||
\end{solution}
|
||||
\part Calculer la probabilité de tirer un bonbon à la fraise.
|
||||
\begin{solution}
|
||||
$T($ tirer un bonbon à la fraise $) = \dfrac{14}{86}$
|
||||
\end{solution}
|
||||
\part Calculer la probabilité de tirer un bonbon qui n'est pas au chocolat.
|
||||
\begin{solution}
|
||||
|
||||
$T($ tirer un bonbon à la fraise $) = \dfrac{77}{86}$
|
||||
\end{solution}
|
||||
\part Calculer la probabilité de tirer un bonbon au réglisse.
|
||||
\begin{solution}
|
||||
$T($ tirer un bonbon au réglisse $) = \dfrac{0}{86} = 0$
|
||||
\end{solution}
|
||||
\part Dans un autre sac, on place 25 bonbons à la menthe et 34 bonbons à la fraise. Lise préfère les bonbons à la menthe. Dans quel sac doit-elle tirer un bonbon pour avoir le plus de chance d'avoir un bonbon qu'elle préfère?
|
||||
\end{parts}
|
||||
|
||||
\vfill
|
||||
\end{questions}
|
||||
|
||||
\end{document}
|
||||
|
||||
%%% Local Variables:
|
||||
%%% mode: latex
|
||||
%%% TeX-master: "master"
|
||||
%%% End:
|
78
3e/DM/DM_16_09_15/06_DM_16_09_15.tex
Normal file
78
3e/DM/DM_16_09_15/06_DM_16_09_15.tex
Normal file
@@ -0,0 +1,78 @@
|
||||
\documentclass[a5paper,12pt, table]{/media/documents/Cours/Prof/Enseignements/2016-2017/tools/style/classDS}
|
||||
\usepackage{/media/documents/Cours/Prof/Enseignements/2016-2017/theme}
|
||||
\geometry{left=10mm,right=10mm, top=10mm, bottom=10mm}
|
||||
|
||||
% Title Page
|
||||
\titre{1}
|
||||
% \seconde \premiereS \PSTMG \TSTMG
|
||||
\classe{Troisième}
|
||||
\date{15 septembre 2016}
|
||||
%\duree{1 heure}
|
||||
\sujet{06}
|
||||
% DS DSCorr DM DMCorr Corr
|
||||
\typedoc{DM}
|
||||
|
||||
%\printanswers
|
||||
|
||||
\begin{document}
|
||||
|
||||
\maketitle
|
||||
|
||||
Vous devez rendre le sujet avec la copie.
|
||||
|
||||
\begin{questions}
|
||||
|
||||
\question
|
||||
Faire les calculs suivants en détaillant des étapes.
|
||||
\begin{multicols}{2}
|
||||
\begin{parts}
|
||||
|
||||
\part $7 + 9 \times ( -7 )$
|
||||
|
||||
\part $-6 - 9 \times 7$
|
||||
|
||||
\part $-7 \times 8 - 10 \times ( -6 )$
|
||||
|
||||
\part $( 9 + 7 ) \times ( -4 ) + 7$
|
||||
|
||||
\part $-7 ( -6 + 10 ) \times ( -3 )$
|
||||
|
||||
\part $-2 ( -1 - ( -5 ) ) + 8$
|
||||
\end{parts}
|
||||
\end{multicols}
|
||||
\vfill
|
||||
|
||||
\question
|
||||
|
||||
|
||||
Dans un sac, il y a 30 bonbons à la menthe, 27 bonbons à la fraise et 6 au chocolat. On choisit un bonbon au hasard dans ce sac.
|
||||
\begin{parts}
|
||||
\part Combien y a-t-il d'issues en tout?
|
||||
\begin{solution}
|
||||
Il y a 63 bonbons.
|
||||
\end{solution}
|
||||
\part Calculer la probabilité de tirer un bonbon à la fraise.
|
||||
\begin{solution}
|
||||
$T($ tirer un bonbon à la fraise $) = \dfrac{30}{63}$
|
||||
\end{solution}
|
||||
\part Calculer la probabilité de tirer un bonbon qui n'est pas au chocolat.
|
||||
\begin{solution}
|
||||
|
||||
$T($ tirer un bonbon à la fraise $) = \dfrac{57}{63}$
|
||||
\end{solution}
|
||||
\part Calculer la probabilité de tirer un bonbon au réglisse.
|
||||
\begin{solution}
|
||||
$T($ tirer un bonbon au réglisse $) = \dfrac{0}{63} = 0$
|
||||
\end{solution}
|
||||
\part Dans un autre sac, on place 25 bonbons à la menthe et 34 bonbons à la fraise. Lise préfère les bonbons à la menthe. Dans quel sac doit-elle tirer un bonbon pour avoir le plus de chance d'avoir un bonbon qu'elle préfère?
|
||||
\end{parts}
|
||||
|
||||
\vfill
|
||||
\end{questions}
|
||||
|
||||
\end{document}
|
||||
|
||||
%%% Local Variables:
|
||||
%%% mode: latex
|
||||
%%% TeX-master: "master"
|
||||
%%% End:
|
78
3e/DM/DM_16_09_15/07_DM_16_09_15.tex
Normal file
78
3e/DM/DM_16_09_15/07_DM_16_09_15.tex
Normal file
@@ -0,0 +1,78 @@
|
||||
\documentclass[a5paper,12pt, table]{/media/documents/Cours/Prof/Enseignements/2016-2017/tools/style/classDS}
|
||||
\usepackage{/media/documents/Cours/Prof/Enseignements/2016-2017/theme}
|
||||
\geometry{left=10mm,right=10mm, top=10mm, bottom=10mm}
|
||||
|
||||
% Title Page
|
||||
\titre{1}
|
||||
% \seconde \premiereS \PSTMG \TSTMG
|
||||
\classe{Troisième}
|
||||
\date{15 septembre 2016}
|
||||
%\duree{1 heure}
|
||||
\sujet{07}
|
||||
% DS DSCorr DM DMCorr Corr
|
||||
\typedoc{DM}
|
||||
|
||||
%\printanswers
|
||||
|
||||
\begin{document}
|
||||
|
||||
\maketitle
|
||||
|
||||
Vous devez rendre le sujet avec la copie.
|
||||
|
||||
\begin{questions}
|
||||
|
||||
\question
|
||||
Faire les calculs suivants en détaillant des étapes.
|
||||
\begin{multicols}{2}
|
||||
\begin{parts}
|
||||
|
||||
\part $-7 + 10 \times 7$
|
||||
|
||||
\part $10 - ( -4 \times 3 )$
|
||||
|
||||
\part $4 \times 3 + 2 \times ( -10 )$
|
||||
|
||||
\part $( -6 - 8 ) \times ( -3 ) + 7$
|
||||
|
||||
\part $1 ( -5 + 2 ) \times 10$
|
||||
|
||||
\part $-1 ( 3 - 2 ) - 5$
|
||||
\end{parts}
|
||||
\end{multicols}
|
||||
\vfill
|
||||
|
||||
\question
|
||||
|
||||
|
||||
Dans un sac, il y a 49 bonbons à la menthe, 63 bonbons à la fraise et 9 au chocolat. On choisit un bonbon au hasard dans ce sac.
|
||||
\begin{parts}
|
||||
\part Combien y a-t-il d'issues en tout?
|
||||
\begin{solution}
|
||||
Il y a 121 bonbons.
|
||||
\end{solution}
|
||||
\part Calculer la probabilité de tirer un bonbon à la fraise.
|
||||
\begin{solution}
|
||||
$T($ tirer un bonbon à la fraise $) = \dfrac{49}{121}$
|
||||
\end{solution}
|
||||
\part Calculer la probabilité de tirer un bonbon qui n'est pas au chocolat.
|
||||
\begin{solution}
|
||||
|
||||
$T($ tirer un bonbon à la fraise $) = \dfrac{112}{121}$
|
||||
\end{solution}
|
||||
\part Calculer la probabilité de tirer un bonbon au réglisse.
|
||||
\begin{solution}
|
||||
$T($ tirer un bonbon au réglisse $) = \dfrac{0}{121} = 0$
|
||||
\end{solution}
|
||||
\part Dans un autre sac, on place 25 bonbons à la menthe et 34 bonbons à la fraise. Lise préfère les bonbons à la menthe. Dans quel sac doit-elle tirer un bonbon pour avoir le plus de chance d'avoir un bonbon qu'elle préfère?
|
||||
\end{parts}
|
||||
|
||||
\vfill
|
||||
\end{questions}
|
||||
|
||||
\end{document}
|
||||
|
||||
%%% Local Variables:
|
||||
%%% mode: latex
|
||||
%%% TeX-master: "master"
|
||||
%%% End:
|
78
3e/DM/DM_16_09_15/08_DM_16_09_15.tex
Normal file
78
3e/DM/DM_16_09_15/08_DM_16_09_15.tex
Normal file
@@ -0,0 +1,78 @@
|
||||
\documentclass[a5paper,12pt, table]{/media/documents/Cours/Prof/Enseignements/2016-2017/tools/style/classDS}
|
||||
\usepackage{/media/documents/Cours/Prof/Enseignements/2016-2017/theme}
|
||||
\geometry{left=10mm,right=10mm, top=10mm, bottom=10mm}
|
||||
|
||||
% Title Page
|
||||
\titre{1}
|
||||
% \seconde \premiereS \PSTMG \TSTMG
|
||||
\classe{Troisième}
|
||||
\date{15 septembre 2016}
|
||||
%\duree{1 heure}
|
||||
\sujet{08}
|
||||
% DS DSCorr DM DMCorr Corr
|
||||
\typedoc{DM}
|
||||
|
||||
%\printanswers
|
||||
|
||||
\begin{document}
|
||||
|
||||
\maketitle
|
||||
|
||||
Vous devez rendre le sujet avec la copie.
|
||||
|
||||
\begin{questions}
|
||||
|
||||
\question
|
||||
Faire les calculs suivants en détaillant des étapes.
|
||||
\begin{multicols}{2}
|
||||
\begin{parts}
|
||||
|
||||
\part $-6 + 10 \times ( -1 )$
|
||||
|
||||
\part $5 - ( -9 \times ( -8 ) )$
|
||||
|
||||
\part $-5 \times 8 + 7 \times ( -4 )$
|
||||
|
||||
\part $( -8 + 4 ) \times 9 - 10$
|
||||
|
||||
\part $-10 ( 2 + 2 ) \times 6$
|
||||
|
||||
\part $-6 ( 2 - 10 ) - 2$
|
||||
\end{parts}
|
||||
\end{multicols}
|
||||
\vfill
|
||||
|
||||
\question
|
||||
|
||||
|
||||
Dans un sac, il y a 30 bonbons à la menthe, 15 bonbons à la fraise et 10 au chocolat. On choisit un bonbon au hasard dans ce sac.
|
||||
\begin{parts}
|
||||
\part Combien y a-t-il d'issues en tout?
|
||||
\begin{solution}
|
||||
Il y a 55 bonbons.
|
||||
\end{solution}
|
||||
\part Calculer la probabilité de tirer un bonbon à la fraise.
|
||||
\begin{solution}
|
||||
$T($ tirer un bonbon à la fraise $) = \dfrac{30}{55}$
|
||||
\end{solution}
|
||||
\part Calculer la probabilité de tirer un bonbon qui n'est pas au chocolat.
|
||||
\begin{solution}
|
||||
|
||||
$T($ tirer un bonbon à la fraise $) = \dfrac{45}{55}$
|
||||
\end{solution}
|
||||
\part Calculer la probabilité de tirer un bonbon au réglisse.
|
||||
\begin{solution}
|
||||
$T($ tirer un bonbon au réglisse $) = \dfrac{0}{55} = 0$
|
||||
\end{solution}
|
||||
\part Dans un autre sac, on place 25 bonbons à la menthe et 34 bonbons à la fraise. Lise préfère les bonbons à la menthe. Dans quel sac doit-elle tirer un bonbon pour avoir le plus de chance d'avoir un bonbon qu'elle préfère?
|
||||
\end{parts}
|
||||
|
||||
\vfill
|
||||
\end{questions}
|
||||
|
||||
\end{document}
|
||||
|
||||
%%% Local Variables:
|
||||
%%% mode: latex
|
||||
%%% TeX-master: "master"
|
||||
%%% End:
|
78
3e/DM/DM_16_09_15/09_DM_16_09_15.tex
Normal file
78
3e/DM/DM_16_09_15/09_DM_16_09_15.tex
Normal file
@@ -0,0 +1,78 @@
|
||||
\documentclass[a5paper,12pt, table]{/media/documents/Cours/Prof/Enseignements/2016-2017/tools/style/classDS}
|
||||
\usepackage{/media/documents/Cours/Prof/Enseignements/2016-2017/theme}
|
||||
\geometry{left=10mm,right=10mm, top=10mm, bottom=10mm}
|
||||
|
||||
% Title Page
|
||||
\titre{1}
|
||||
% \seconde \premiereS \PSTMG \TSTMG
|
||||
\classe{Troisième}
|
||||
\date{15 septembre 2016}
|
||||
%\duree{1 heure}
|
||||
\sujet{09}
|
||||
% DS DSCorr DM DMCorr Corr
|
||||
\typedoc{DM}
|
||||
|
||||
%\printanswers
|
||||
|
||||
\begin{document}
|
||||
|
||||
\maketitle
|
||||
|
||||
Vous devez rendre le sujet avec la copie.
|
||||
|
||||
\begin{questions}
|
||||
|
||||
\question
|
||||
Faire les calculs suivants en détaillant des étapes.
|
||||
\begin{multicols}{2}
|
||||
\begin{parts}
|
||||
|
||||
\part $4 - 9 \times 2$
|
||||
|
||||
\part $6 - 1 \times ( -5 )$
|
||||
|
||||
\part $5 \times ( -5 ) + 7 \times ( -4 )$
|
||||
|
||||
\part $( -5 + 7 ) \times 2 + 10$
|
||||
|
||||
\part $-1 ( -5 - 3 ) \times 9$
|
||||
|
||||
\part $8 ( 3 - 3 ) + 6$
|
||||
\end{parts}
|
||||
\end{multicols}
|
||||
\vfill
|
||||
|
||||
\question
|
||||
|
||||
|
||||
Dans un sac, il y a 48 bonbons à la menthe, 40 bonbons à la fraise et 7 au chocolat. On choisit un bonbon au hasard dans ce sac.
|
||||
\begin{parts}
|
||||
\part Combien y a-t-il d'issues en tout?
|
||||
\begin{solution}
|
||||
Il y a 95 bonbons.
|
||||
\end{solution}
|
||||
\part Calculer la probabilité de tirer un bonbon à la fraise.
|
||||
\begin{solution}
|
||||
$T($ tirer un bonbon à la fraise $) = \dfrac{48}{95}$
|
||||
\end{solution}
|
||||
\part Calculer la probabilité de tirer un bonbon qui n'est pas au chocolat.
|
||||
\begin{solution}
|
||||
|
||||
$T($ tirer un bonbon à la fraise $) = \dfrac{88}{95}$
|
||||
\end{solution}
|
||||
\part Calculer la probabilité de tirer un bonbon au réglisse.
|
||||
\begin{solution}
|
||||
$T($ tirer un bonbon au réglisse $) = \dfrac{0}{95} = 0$
|
||||
\end{solution}
|
||||
\part Dans un autre sac, on place 25 bonbons à la menthe et 34 bonbons à la fraise. Lise préfère les bonbons à la menthe. Dans quel sac doit-elle tirer un bonbon pour avoir le plus de chance d'avoir un bonbon qu'elle préfère?
|
||||
\end{parts}
|
||||
|
||||
\vfill
|
||||
\end{questions}
|
||||
|
||||
\end{document}
|
||||
|
||||
%%% Local Variables:
|
||||
%%% mode: latex
|
||||
%%% TeX-master: "master"
|
||||
%%% End:
|
78
3e/DM/DM_16_09_15/10_DM_16_09_15.tex
Normal file
78
3e/DM/DM_16_09_15/10_DM_16_09_15.tex
Normal file
@@ -0,0 +1,78 @@
|
||||
\documentclass[a5paper,12pt, table]{/media/documents/Cours/Prof/Enseignements/2016-2017/tools/style/classDS}
|
||||
\usepackage{/media/documents/Cours/Prof/Enseignements/2016-2017/theme}
|
||||
\geometry{left=10mm,right=10mm, top=10mm, bottom=10mm}
|
||||
|
||||
% Title Page
|
||||
\titre{1}
|
||||
% \seconde \premiereS \PSTMG \TSTMG
|
||||
\classe{Troisième}
|
||||
\date{15 septembre 2016}
|
||||
%\duree{1 heure}
|
||||
\sujet{10}
|
||||
% DS DSCorr DM DMCorr Corr
|
||||
\typedoc{DM}
|
||||
|
||||
%\printanswers
|
||||
|
||||
\begin{document}
|
||||
|
||||
\maketitle
|
||||
|
||||
Vous devez rendre le sujet avec la copie.
|
||||
|
||||
\begin{questions}
|
||||
|
||||
\question
|
||||
Faire les calculs suivants en détaillant des étapes.
|
||||
\begin{multicols}{2}
|
||||
\begin{parts}
|
||||
|
||||
\part $-8 + 6 \times ( -2 )$
|
||||
|
||||
\part $-5 - ( -7 \times 10 )$
|
||||
|
||||
\part $-5 \times 2 + 4 \times 10$
|
||||
|
||||
\part $( -5 - 7 ) \times 2 + 6$
|
||||
|
||||
\part $4 ( -2 - 9 ) \times 5$
|
||||
|
||||
\part $-4 ( 6 - ( -5 ) ) + 3$
|
||||
\end{parts}
|
||||
\end{multicols}
|
||||
\vfill
|
||||
|
||||
\question
|
||||
|
||||
|
||||
Dans un sac, il y a 10 bonbons à la menthe, 30 bonbons à la fraise et 2 au chocolat. On choisit un bonbon au hasard dans ce sac.
|
||||
\begin{parts}
|
||||
\part Combien y a-t-il d'issues en tout?
|
||||
\begin{solution}
|
||||
Il y a 42 bonbons.
|
||||
\end{solution}
|
||||
\part Calculer la probabilité de tirer un bonbon à la fraise.
|
||||
\begin{solution}
|
||||
$T($ tirer un bonbon à la fraise $) = \dfrac{10}{42}$
|
||||
\end{solution}
|
||||
\part Calculer la probabilité de tirer un bonbon qui n'est pas au chocolat.
|
||||
\begin{solution}
|
||||
|
||||
$T($ tirer un bonbon à la fraise $) = \dfrac{40}{42}$
|
||||
\end{solution}
|
||||
\part Calculer la probabilité de tirer un bonbon au réglisse.
|
||||
\begin{solution}
|
||||
$T($ tirer un bonbon au réglisse $) = \dfrac{0}{42} = 0$
|
||||
\end{solution}
|
||||
\part Dans un autre sac, on place 25 bonbons à la menthe et 34 bonbons à la fraise. Lise préfère les bonbons à la menthe. Dans quel sac doit-elle tirer un bonbon pour avoir le plus de chance d'avoir un bonbon qu'elle préfère?
|
||||
\end{parts}
|
||||
|
||||
\vfill
|
||||
\end{questions}
|
||||
|
||||
\end{document}
|
||||
|
||||
%%% Local Variables:
|
||||
%%% mode: latex
|
||||
%%% TeX-master: "master"
|
||||
%%% End:
|
78
3e/DM/DM_16_09_15/11_DM_16_09_15.tex
Normal file
78
3e/DM/DM_16_09_15/11_DM_16_09_15.tex
Normal file
@@ -0,0 +1,78 @@
|
||||
\documentclass[a5paper,12pt, table]{/media/documents/Cours/Prof/Enseignements/2016-2017/tools/style/classDS}
|
||||
\usepackage{/media/documents/Cours/Prof/Enseignements/2016-2017/theme}
|
||||
\geometry{left=10mm,right=10mm, top=10mm, bottom=10mm}
|
||||
|
||||
% Title Page
|
||||
\titre{1}
|
||||
% \seconde \premiereS \PSTMG \TSTMG
|
||||
\classe{Troisième}
|
||||
\date{15 septembre 2016}
|
||||
%\duree{1 heure}
|
||||
\sujet{11}
|
||||
% DS DSCorr DM DMCorr Corr
|
||||
\typedoc{DM}
|
||||
|
||||
%\printanswers
|
||||
|
||||
\begin{document}
|
||||
|
||||
\maketitle
|
||||
|
||||
Vous devez rendre le sujet avec la copie.
|
||||
|
||||
\begin{questions}
|
||||
|
||||
\question
|
||||
Faire les calculs suivants en détaillant des étapes.
|
||||
\begin{multicols}{2}
|
||||
\begin{parts}
|
||||
|
||||
\part $-6 - 8 \times 10$
|
||||
|
||||
\part $6 - ( -5 \times 10 )$
|
||||
|
||||
\part $7 \times ( -8 ) + 3 \times ( -9 )$
|
||||
|
||||
\part $( -8 + 5 ) \times 5 + 3$
|
||||
|
||||
\part $9 ( 2 + 10 ) \times ( -10 )$
|
||||
|
||||
\part $-9 ( 5 - 5 ) + 8$
|
||||
\end{parts}
|
||||
\end{multicols}
|
||||
\vfill
|
||||
|
||||
\question
|
||||
|
||||
|
||||
Dans un sac, il y a 14 bonbons à la menthe, 28 bonbons à la fraise et 3 au chocolat. On choisit un bonbon au hasard dans ce sac.
|
||||
\begin{parts}
|
||||
\part Combien y a-t-il d'issues en tout?
|
||||
\begin{solution}
|
||||
Il y a 45 bonbons.
|
||||
\end{solution}
|
||||
\part Calculer la probabilité de tirer un bonbon à la fraise.
|
||||
\begin{solution}
|
||||
$T($ tirer un bonbon à la fraise $) = \dfrac{14}{45}$
|
||||
\end{solution}
|
||||
\part Calculer la probabilité de tirer un bonbon qui n'est pas au chocolat.
|
||||
\begin{solution}
|
||||
|
||||
$T($ tirer un bonbon à la fraise $) = \dfrac{42}{45}$
|
||||
\end{solution}
|
||||
\part Calculer la probabilité de tirer un bonbon au réglisse.
|
||||
\begin{solution}
|
||||
$T($ tirer un bonbon au réglisse $) = \dfrac{0}{45} = 0$
|
||||
\end{solution}
|
||||
\part Dans un autre sac, on place 25 bonbons à la menthe et 34 bonbons à la fraise. Lise préfère les bonbons à la menthe. Dans quel sac doit-elle tirer un bonbon pour avoir le plus de chance d'avoir un bonbon qu'elle préfère?
|
||||
\end{parts}
|
||||
|
||||
\vfill
|
||||
\end{questions}
|
||||
|
||||
\end{document}
|
||||
|
||||
%%% Local Variables:
|
||||
%%% mode: latex
|
||||
%%% TeX-master: "master"
|
||||
%%% End:
|
78
3e/DM/DM_16_09_15/12_DM_16_09_15.tex
Normal file
78
3e/DM/DM_16_09_15/12_DM_16_09_15.tex
Normal file
@@ -0,0 +1,78 @@
|
||||
\documentclass[a5paper,12pt, table]{/media/documents/Cours/Prof/Enseignements/2016-2017/tools/style/classDS}
|
||||
\usepackage{/media/documents/Cours/Prof/Enseignements/2016-2017/theme}
|
||||
\geometry{left=10mm,right=10mm, top=10mm, bottom=10mm}
|
||||
|
||||
% Title Page
|
||||
\titre{1}
|
||||
% \seconde \premiereS \PSTMG \TSTMG
|
||||
\classe{Troisième}
|
||||
\date{15 septembre 2016}
|
||||
%\duree{1 heure}
|
||||
\sujet{12}
|
||||
% DS DSCorr DM DMCorr Corr
|
||||
\typedoc{DM}
|
||||
|
||||
%\printanswers
|
||||
|
||||
\begin{document}
|
||||
|
||||
\maketitle
|
||||
|
||||
Vous devez rendre le sujet avec la copie.
|
||||
|
||||
\begin{questions}
|
||||
|
||||
\question
|
||||
Faire les calculs suivants en détaillant des étapes.
|
||||
\begin{multicols}{2}
|
||||
\begin{parts}
|
||||
|
||||
\part $3 - 6 \times 1$
|
||||
|
||||
\part $6 - 7 \times ( -5 )$
|
||||
|
||||
\part $3 \times 4 + 7 \times ( -2 )$
|
||||
|
||||
\part $( 8 + 3 ) \times 10 - 3$
|
||||
|
||||
\part $-9 ( -10 - 1 ) \times ( -1 )$
|
||||
|
||||
\part $-4 ( -9 - 1 ) - 6$
|
||||
\end{parts}
|
||||
\end{multicols}
|
||||
\vfill
|
||||
|
||||
\question
|
||||
|
||||
|
||||
Dans un sac, il y a 20 bonbons à la menthe, 8 bonbons à la fraise et 10 au chocolat. On choisit un bonbon au hasard dans ce sac.
|
||||
\begin{parts}
|
||||
\part Combien y a-t-il d'issues en tout?
|
||||
\begin{solution}
|
||||
Il y a 38 bonbons.
|
||||
\end{solution}
|
||||
\part Calculer la probabilité de tirer un bonbon à la fraise.
|
||||
\begin{solution}
|
||||
$T($ tirer un bonbon à la fraise $) = \dfrac{20}{38}$
|
||||
\end{solution}
|
||||
\part Calculer la probabilité de tirer un bonbon qui n'est pas au chocolat.
|
||||
\begin{solution}
|
||||
|
||||
$T($ tirer un bonbon à la fraise $) = \dfrac{28}{38}$
|
||||
\end{solution}
|
||||
\part Calculer la probabilité de tirer un bonbon au réglisse.
|
||||
\begin{solution}
|
||||
$T($ tirer un bonbon au réglisse $) = \dfrac{0}{38} = 0$
|
||||
\end{solution}
|
||||
\part Dans un autre sac, on place 25 bonbons à la menthe et 34 bonbons à la fraise. Lise préfère les bonbons à la menthe. Dans quel sac doit-elle tirer un bonbon pour avoir le plus de chance d'avoir un bonbon qu'elle préfère?
|
||||
\end{parts}
|
||||
|
||||
\vfill
|
||||
\end{questions}
|
||||
|
||||
\end{document}
|
||||
|
||||
%%% Local Variables:
|
||||
%%% mode: latex
|
||||
%%% TeX-master: "master"
|
||||
%%% End:
|
78
3e/DM/DM_16_09_15/13_DM_16_09_15.tex
Normal file
78
3e/DM/DM_16_09_15/13_DM_16_09_15.tex
Normal file
@@ -0,0 +1,78 @@
|
||||
\documentclass[a5paper,12pt, table]{/media/documents/Cours/Prof/Enseignements/2016-2017/tools/style/classDS}
|
||||
\usepackage{/media/documents/Cours/Prof/Enseignements/2016-2017/theme}
|
||||
\geometry{left=10mm,right=10mm, top=10mm, bottom=10mm}
|
||||
|
||||
% Title Page
|
||||
\titre{1}
|
||||
% \seconde \premiereS \PSTMG \TSTMG
|
||||
\classe{Troisième}
|
||||
\date{15 septembre 2016}
|
||||
%\duree{1 heure}
|
||||
\sujet{13}
|
||||
% DS DSCorr DM DMCorr Corr
|
||||
\typedoc{DM}
|
||||
|
||||
%\printanswers
|
||||
|
||||
\begin{document}
|
||||
|
||||
\maketitle
|
||||
|
||||
Vous devez rendre le sujet avec la copie.
|
||||
|
||||
\begin{questions}
|
||||
|
||||
\question
|
||||
Faire les calculs suivants en détaillant des étapes.
|
||||
\begin{multicols}{2}
|
||||
\begin{parts}
|
||||
|
||||
\part $-9 + 4 \times 5$
|
||||
|
||||
\part $7 - 2 \times ( -5 )$
|
||||
|
||||
\part $2 \times 2 - 1 \times 8$
|
||||
|
||||
\part $( 5 - 2 ) \times 7 - 8$
|
||||
|
||||
\part $-6 ( 2 - 8 ) \times ( -2 )$
|
||||
|
||||
\part $-6 ( -3 - ( -1 ) ) - 8$
|
||||
\end{parts}
|
||||
\end{multicols}
|
||||
\vfill
|
||||
|
||||
\question
|
||||
|
||||
|
||||
Dans un sac, il y a 8 bonbons à la menthe, 32 bonbons à la fraise et 6 au chocolat. On choisit un bonbon au hasard dans ce sac.
|
||||
\begin{parts}
|
||||
\part Combien y a-t-il d'issues en tout?
|
||||
\begin{solution}
|
||||
Il y a 46 bonbons.
|
||||
\end{solution}
|
||||
\part Calculer la probabilité de tirer un bonbon à la fraise.
|
||||
\begin{solution}
|
||||
$T($ tirer un bonbon à la fraise $) = \dfrac{8}{46}$
|
||||
\end{solution}
|
||||
\part Calculer la probabilité de tirer un bonbon qui n'est pas au chocolat.
|
||||
\begin{solution}
|
||||
|
||||
$T($ tirer un bonbon à la fraise $) = \dfrac{40}{46}$
|
||||
\end{solution}
|
||||
\part Calculer la probabilité de tirer un bonbon au réglisse.
|
||||
\begin{solution}
|
||||
$T($ tirer un bonbon au réglisse $) = \dfrac{0}{46} = 0$
|
||||
\end{solution}
|
||||
\part Dans un autre sac, on place 25 bonbons à la menthe et 34 bonbons à la fraise. Lise préfère les bonbons à la menthe. Dans quel sac doit-elle tirer un bonbon pour avoir le plus de chance d'avoir un bonbon qu'elle préfère?
|
||||
\end{parts}
|
||||
|
||||
\vfill
|
||||
\end{questions}
|
||||
|
||||
\end{document}
|
||||
|
||||
%%% Local Variables:
|
||||
%%% mode: latex
|
||||
%%% TeX-master: "master"
|
||||
%%% End:
|
78
3e/DM/DM_16_09_15/14_DM_16_09_15.tex
Normal file
78
3e/DM/DM_16_09_15/14_DM_16_09_15.tex
Normal file
@@ -0,0 +1,78 @@
|
||||
\documentclass[a5paper,12pt, table]{/media/documents/Cours/Prof/Enseignements/2016-2017/tools/style/classDS}
|
||||
\usepackage{/media/documents/Cours/Prof/Enseignements/2016-2017/theme}
|
||||
\geometry{left=10mm,right=10mm, top=10mm, bottom=10mm}
|
||||
|
||||
% Title Page
|
||||
\titre{1}
|
||||
% \seconde \premiereS \PSTMG \TSTMG
|
||||
\classe{Troisième}
|
||||
\date{15 septembre 2016}
|
||||
%\duree{1 heure}
|
||||
\sujet{14}
|
||||
% DS DSCorr DM DMCorr Corr
|
||||
\typedoc{DM}
|
||||
|
||||
%\printanswers
|
||||
|
||||
\begin{document}
|
||||
|
||||
\maketitle
|
||||
|
||||
Vous devez rendre le sujet avec la copie.
|
||||
|
||||
\begin{questions}
|
||||
|
||||
\question
|
||||
Faire les calculs suivants en détaillant des étapes.
|
||||
\begin{multicols}{2}
|
||||
\begin{parts}
|
||||
|
||||
\part $-1 - 9 \times 2$
|
||||
|
||||
\part $-8 - 5 \times ( -8 )$
|
||||
|
||||
\part $-5 \times ( -10 ) + 5 \times 6$
|
||||
|
||||
\part $( 4 + 2 ) \times 2 - 3$
|
||||
|
||||
\part $2 ( -10 - 7 ) \times ( -8 )$
|
||||
|
||||
\part $8 ( 5 - 6 ) - 3$
|
||||
\end{parts}
|
||||
\end{multicols}
|
||||
\vfill
|
||||
|
||||
\question
|
||||
|
||||
|
||||
Dans un sac, il y a 36 bonbons à la menthe, 8 bonbons à la fraise et 8 au chocolat. On choisit un bonbon au hasard dans ce sac.
|
||||
\begin{parts}
|
||||
\part Combien y a-t-il d'issues en tout?
|
||||
\begin{solution}
|
||||
Il y a 52 bonbons.
|
||||
\end{solution}
|
||||
\part Calculer la probabilité de tirer un bonbon à la fraise.
|
||||
\begin{solution}
|
||||
$T($ tirer un bonbon à la fraise $) = \dfrac{36}{52}$
|
||||
\end{solution}
|
||||
\part Calculer la probabilité de tirer un bonbon qui n'est pas au chocolat.
|
||||
\begin{solution}
|
||||
|
||||
$T($ tirer un bonbon à la fraise $) = \dfrac{44}{52}$
|
||||
\end{solution}
|
||||
\part Calculer la probabilité de tirer un bonbon au réglisse.
|
||||
\begin{solution}
|
||||
$T($ tirer un bonbon au réglisse $) = \dfrac{0}{52} = 0$
|
||||
\end{solution}
|
||||
\part Dans un autre sac, on place 25 bonbons à la menthe et 34 bonbons à la fraise. Lise préfère les bonbons à la menthe. Dans quel sac doit-elle tirer un bonbon pour avoir le plus de chance d'avoir un bonbon qu'elle préfère?
|
||||
\end{parts}
|
||||
|
||||
\vfill
|
||||
\end{questions}
|
||||
|
||||
\end{document}
|
||||
|
||||
%%% Local Variables:
|
||||
%%% mode: latex
|
||||
%%% TeX-master: "master"
|
||||
%%% End:
|
78
3e/DM/DM_16_09_15/15_DM_16_09_15.tex
Normal file
78
3e/DM/DM_16_09_15/15_DM_16_09_15.tex
Normal file
@@ -0,0 +1,78 @@
|
||||
\documentclass[a5paper,12pt, table]{/media/documents/Cours/Prof/Enseignements/2016-2017/tools/style/classDS}
|
||||
\usepackage{/media/documents/Cours/Prof/Enseignements/2016-2017/theme}
|
||||
\geometry{left=10mm,right=10mm, top=10mm, bottom=10mm}
|
||||
|
||||
% Title Page
|
||||
\titre{1}
|
||||
% \seconde \premiereS \PSTMG \TSTMG
|
||||
\classe{Troisième}
|
||||
\date{15 septembre 2016}
|
||||
%\duree{1 heure}
|
||||
\sujet{15}
|
||||
% DS DSCorr DM DMCorr Corr
|
||||
\typedoc{DM}
|
||||
|
||||
%\printanswers
|
||||
|
||||
\begin{document}
|
||||
|
||||
\maketitle
|
||||
|
||||
Vous devez rendre le sujet avec la copie.
|
||||
|
||||
\begin{questions}
|
||||
|
||||
\question
|
||||
Faire les calculs suivants en détaillant des étapes.
|
||||
\begin{multicols}{2}
|
||||
\begin{parts}
|
||||
|
||||
\part $-7 + 8 \times ( -9 )$
|
||||
|
||||
\part $6 - 5 \times ( -4 )$
|
||||
|
||||
\part $5 \times ( -6 ) + 1 \times ( -2 )$
|
||||
|
||||
\part $( 8 - 1 ) \times ( -9 ) + 7$
|
||||
|
||||
\part $6 ( -4 + 2 ) \times 10$
|
||||
|
||||
\part $7 ( 8 - ( -8 ) ) - 1$
|
||||
\end{parts}
|
||||
\end{multicols}
|
||||
\vfill
|
||||
|
||||
\question
|
||||
|
||||
|
||||
Dans un sac, il y a 24 bonbons à la menthe, 40 bonbons à la fraise et 4 au chocolat. On choisit un bonbon au hasard dans ce sac.
|
||||
\begin{parts}
|
||||
\part Combien y a-t-il d'issues en tout?
|
||||
\begin{solution}
|
||||
Il y a 68 bonbons.
|
||||
\end{solution}
|
||||
\part Calculer la probabilité de tirer un bonbon à la fraise.
|
||||
\begin{solution}
|
||||
$T($ tirer un bonbon à la fraise $) = \dfrac{24}{68}$
|
||||
\end{solution}
|
||||
\part Calculer la probabilité de tirer un bonbon qui n'est pas au chocolat.
|
||||
\begin{solution}
|
||||
|
||||
$T($ tirer un bonbon à la fraise $) = \dfrac{64}{68}$
|
||||
\end{solution}
|
||||
\part Calculer la probabilité de tirer un bonbon au réglisse.
|
||||
\begin{solution}
|
||||
$T($ tirer un bonbon au réglisse $) = \dfrac{0}{68} = 0$
|
||||
\end{solution}
|
||||
\part Dans un autre sac, on place 25 bonbons à la menthe et 34 bonbons à la fraise. Lise préfère les bonbons à la menthe. Dans quel sac doit-elle tirer un bonbon pour avoir le plus de chance d'avoir un bonbon qu'elle préfère?
|
||||
\end{parts}
|
||||
|
||||
\vfill
|
||||
\end{questions}
|
||||
|
||||
\end{document}
|
||||
|
||||
%%% Local Variables:
|
||||
%%% mode: latex
|
||||
%%% TeX-master: "master"
|
||||
%%% End:
|
78
3e/DM/DM_16_09_15/16_DM_16_09_15.tex
Normal file
78
3e/DM/DM_16_09_15/16_DM_16_09_15.tex
Normal file
@@ -0,0 +1,78 @@
|
||||
\documentclass[a5paper,12pt, table]{/media/documents/Cours/Prof/Enseignements/2016-2017/tools/style/classDS}
|
||||
\usepackage{/media/documents/Cours/Prof/Enseignements/2016-2017/theme}
|
||||
\geometry{left=10mm,right=10mm, top=10mm, bottom=10mm}
|
||||
|
||||
% Title Page
|
||||
\titre{1}
|
||||
% \seconde \premiereS \PSTMG \TSTMG
|
||||
\classe{Troisième}
|
||||
\date{15 septembre 2016}
|
||||
%\duree{1 heure}
|
||||
\sujet{16}
|
||||
% DS DSCorr DM DMCorr Corr
|
||||
\typedoc{DM}
|
||||
|
||||
%\printanswers
|
||||
|
||||
\begin{document}
|
||||
|
||||
\maketitle
|
||||
|
||||
Vous devez rendre le sujet avec la copie.
|
||||
|
||||
\begin{questions}
|
||||
|
||||
\question
|
||||
Faire les calculs suivants en détaillant des étapes.
|
||||
\begin{multicols}{2}
|
||||
\begin{parts}
|
||||
|
||||
\part $5 - 9 \times 2$
|
||||
|
||||
\part $8 - 4 \times 2$
|
||||
|
||||
\part $-10 \times ( -1 ) + 5 \times 7$
|
||||
|
||||
\part $( -1 - 5 ) \times 1 + 1$
|
||||
|
||||
\part $-6 ( 9 - 8 ) \times 2$
|
||||
|
||||
\part $1 ( -9 - ( -8 ) ) + 4$
|
||||
\end{parts}
|
||||
\end{multicols}
|
||||
\vfill
|
||||
|
||||
\question
|
||||
|
||||
|
||||
Dans un sac, il y a 16 bonbons à la menthe, 80 bonbons à la fraise et 5 au chocolat. On choisit un bonbon au hasard dans ce sac.
|
||||
\begin{parts}
|
||||
\part Combien y a-t-il d'issues en tout?
|
||||
\begin{solution}
|
||||
Il y a 101 bonbons.
|
||||
\end{solution}
|
||||
\part Calculer la probabilité de tirer un bonbon à la fraise.
|
||||
\begin{solution}
|
||||
$T($ tirer un bonbon à la fraise $) = \dfrac{16}{101}$
|
||||
\end{solution}
|
||||
\part Calculer la probabilité de tirer un bonbon qui n'est pas au chocolat.
|
||||
\begin{solution}
|
||||
|
||||
$T($ tirer un bonbon à la fraise $) = \dfrac{96}{101}$
|
||||
\end{solution}
|
||||
\part Calculer la probabilité de tirer un bonbon au réglisse.
|
||||
\begin{solution}
|
||||
$T($ tirer un bonbon au réglisse $) = \dfrac{0}{101} = 0$
|
||||
\end{solution}
|
||||
\part Dans un autre sac, on place 25 bonbons à la menthe et 34 bonbons à la fraise. Lise préfère les bonbons à la menthe. Dans quel sac doit-elle tirer un bonbon pour avoir le plus de chance d'avoir un bonbon qu'elle préfère?
|
||||
\end{parts}
|
||||
|
||||
\vfill
|
||||
\end{questions}
|
||||
|
||||
\end{document}
|
||||
|
||||
%%% Local Variables:
|
||||
%%% mode: latex
|
||||
%%% TeX-master: "master"
|
||||
%%% End:
|
78
3e/DM/DM_16_09_15/17_DM_16_09_15.tex
Normal file
78
3e/DM/DM_16_09_15/17_DM_16_09_15.tex
Normal file
@@ -0,0 +1,78 @@
|
||||
\documentclass[a5paper,12pt, table]{/media/documents/Cours/Prof/Enseignements/2016-2017/tools/style/classDS}
|
||||
\usepackage{/media/documents/Cours/Prof/Enseignements/2016-2017/theme}
|
||||
\geometry{left=10mm,right=10mm, top=10mm, bottom=10mm}
|
||||
|
||||
% Title Page
|
||||
\titre{1}
|
||||
% \seconde \premiereS \PSTMG \TSTMG
|
||||
\classe{Troisième}
|
||||
\date{15 septembre 2016}
|
||||
%\duree{1 heure}
|
||||
\sujet{17}
|
||||
% DS DSCorr DM DMCorr Corr
|
||||
\typedoc{DM}
|
||||
|
||||
%\printanswers
|
||||
|
||||
\begin{document}
|
||||
|
||||
\maketitle
|
||||
|
||||
Vous devez rendre le sujet avec la copie.
|
||||
|
||||
\begin{questions}
|
||||
|
||||
\question
|
||||
Faire les calculs suivants en détaillant des étapes.
|
||||
\begin{multicols}{2}
|
||||
\begin{parts}
|
||||
|
||||
\part $-7 + 9 \times ( -8 )$
|
||||
|
||||
\part $1 - 5 \times ( -10 )$
|
||||
|
||||
\part $-9 \times 10 + 8 \times 4$
|
||||
|
||||
\part $( 2 + 9 ) \times 4 + 9$
|
||||
|
||||
\part $-3 ( 6 + 4 ) \times 8$
|
||||
|
||||
\part $9 ( -10 - ( -9 ) ) - 2$
|
||||
\end{parts}
|
||||
\end{multicols}
|
||||
\vfill
|
||||
|
||||
\question
|
||||
|
||||
|
||||
Dans un sac, il y a 40 bonbons à la menthe, 100 bonbons à la fraise et 6 au chocolat. On choisit un bonbon au hasard dans ce sac.
|
||||
\begin{parts}
|
||||
\part Combien y a-t-il d'issues en tout?
|
||||
\begin{solution}
|
||||
Il y a 146 bonbons.
|
||||
\end{solution}
|
||||
\part Calculer la probabilité de tirer un bonbon à la fraise.
|
||||
\begin{solution}
|
||||
$T($ tirer un bonbon à la fraise $) = \dfrac{40}{146}$
|
||||
\end{solution}
|
||||
\part Calculer la probabilité de tirer un bonbon qui n'est pas au chocolat.
|
||||
\begin{solution}
|
||||
|
||||
$T($ tirer un bonbon à la fraise $) = \dfrac{140}{146}$
|
||||
\end{solution}
|
||||
\part Calculer la probabilité de tirer un bonbon au réglisse.
|
||||
\begin{solution}
|
||||
$T($ tirer un bonbon au réglisse $) = \dfrac{0}{146} = 0$
|
||||
\end{solution}
|
||||
\part Dans un autre sac, on place 25 bonbons à la menthe et 34 bonbons à la fraise. Lise préfère les bonbons à la menthe. Dans quel sac doit-elle tirer un bonbon pour avoir le plus de chance d'avoir un bonbon qu'elle préfère?
|
||||
\end{parts}
|
||||
|
||||
\vfill
|
||||
\end{questions}
|
||||
|
||||
\end{document}
|
||||
|
||||
%%% Local Variables:
|
||||
%%% mode: latex
|
||||
%%% TeX-master: "master"
|
||||
%%% End:
|
78
3e/DM/DM_16_09_15/18_DM_16_09_15.tex
Normal file
78
3e/DM/DM_16_09_15/18_DM_16_09_15.tex
Normal file
@@ -0,0 +1,78 @@
|
||||
\documentclass[a5paper,12pt, table]{/media/documents/Cours/Prof/Enseignements/2016-2017/tools/style/classDS}
|
||||
\usepackage{/media/documents/Cours/Prof/Enseignements/2016-2017/theme}
|
||||
\geometry{left=10mm,right=10mm, top=10mm, bottom=10mm}
|
||||
|
||||
% Title Page
|
||||
\titre{1}
|
||||
% \seconde \premiereS \PSTMG \TSTMG
|
||||
\classe{Troisième}
|
||||
\date{15 septembre 2016}
|
||||
%\duree{1 heure}
|
||||
\sujet{18}
|
||||
% DS DSCorr DM DMCorr Corr
|
||||
\typedoc{DM}
|
||||
|
||||
%\printanswers
|
||||
|
||||
\begin{document}
|
||||
|
||||
\maketitle
|
||||
|
||||
Vous devez rendre le sujet avec la copie.
|
||||
|
||||
\begin{questions}
|
||||
|
||||
\question
|
||||
Faire les calculs suivants en détaillant des étapes.
|
||||
\begin{multicols}{2}
|
||||
\begin{parts}
|
||||
|
||||
\part $-10 + 7 \times ( -1 )$
|
||||
|
||||
\part $7 - 3 \times ( -7 )$
|
||||
|
||||
\part $-10 \times ( -2 ) + 4 \times 7$
|
||||
|
||||
\part $( -2 + 9 ) \times 7 - 1$
|
||||
|
||||
\part $5 ( -1 - 8 ) \times 5$
|
||||
|
||||
\part $-5 ( -10 - ( -2 ) ) + 10$
|
||||
\end{parts}
|
||||
\end{multicols}
|
||||
\vfill
|
||||
|
||||
\question
|
||||
|
||||
|
||||
Dans un sac, il y a 20 bonbons à la menthe, 8 bonbons à la fraise et 6 au chocolat. On choisit un bonbon au hasard dans ce sac.
|
||||
\begin{parts}
|
||||
\part Combien y a-t-il d'issues en tout?
|
||||
\begin{solution}
|
||||
Il y a 34 bonbons.
|
||||
\end{solution}
|
||||
\part Calculer la probabilité de tirer un bonbon à la fraise.
|
||||
\begin{solution}
|
||||
$T($ tirer un bonbon à la fraise $) = \dfrac{20}{34}$
|
||||
\end{solution}
|
||||
\part Calculer la probabilité de tirer un bonbon qui n'est pas au chocolat.
|
||||
\begin{solution}
|
||||
|
||||
$T($ tirer un bonbon à la fraise $) = \dfrac{28}{34}$
|
||||
\end{solution}
|
||||
\part Calculer la probabilité de tirer un bonbon au réglisse.
|
||||
\begin{solution}
|
||||
$T($ tirer un bonbon au réglisse $) = \dfrac{0}{34} = 0$
|
||||
\end{solution}
|
||||
\part Dans un autre sac, on place 25 bonbons à la menthe et 34 bonbons à la fraise. Lise préfère les bonbons à la menthe. Dans quel sac doit-elle tirer un bonbon pour avoir le plus de chance d'avoir un bonbon qu'elle préfère?
|
||||
\end{parts}
|
||||
|
||||
\vfill
|
||||
\end{questions}
|
||||
|
||||
\end{document}
|
||||
|
||||
%%% Local Variables:
|
||||
%%% mode: latex
|
||||
%%% TeX-master: "master"
|
||||
%%% End:
|
78
3e/DM/DM_16_09_15/19_DM_16_09_15.tex
Normal file
78
3e/DM/DM_16_09_15/19_DM_16_09_15.tex
Normal file
@@ -0,0 +1,78 @@
|
||||
\documentclass[a5paper,12pt, table]{/media/documents/Cours/Prof/Enseignements/2016-2017/tools/style/classDS}
|
||||
\usepackage{/media/documents/Cours/Prof/Enseignements/2016-2017/theme}
|
||||
\geometry{left=10mm,right=10mm, top=10mm, bottom=10mm}
|
||||
|
||||
% Title Page
|
||||
\titre{1}
|
||||
% \seconde \premiereS \PSTMG \TSTMG
|
||||
\classe{Troisième}
|
||||
\date{15 septembre 2016}
|
||||
%\duree{1 heure}
|
||||
\sujet{19}
|
||||
% DS DSCorr DM DMCorr Corr
|
||||
\typedoc{DM}
|
||||
|
||||
%\printanswers
|
||||
|
||||
\begin{document}
|
||||
|
||||
\maketitle
|
||||
|
||||
Vous devez rendre le sujet avec la copie.
|
||||
|
||||
\begin{questions}
|
||||
|
||||
\question
|
||||
Faire les calculs suivants en détaillant des étapes.
|
||||
\begin{multicols}{2}
|
||||
\begin{parts}
|
||||
|
||||
\part $-9 - 8 \times 4$
|
||||
|
||||
\part $6 - 5 \times ( -4 )$
|
||||
|
||||
\part $4 \times 5 + 9 \times ( -3 )$
|
||||
|
||||
\part $( -10 + 10 ) \times 4 - 9$
|
||||
|
||||
\part $1 ( 7 - 1 ) \times ( -2 )$
|
||||
|
||||
\part $-7 ( 2 - 9 ) + 2$
|
||||
\end{parts}
|
||||
\end{multicols}
|
||||
\vfill
|
||||
|
||||
\question
|
||||
|
||||
|
||||
Dans un sac, il y a 80 bonbons à la menthe, 90 bonbons à la fraise et 4 au chocolat. On choisit un bonbon au hasard dans ce sac.
|
||||
\begin{parts}
|
||||
\part Combien y a-t-il d'issues en tout?
|
||||
\begin{solution}
|
||||
Il y a 174 bonbons.
|
||||
\end{solution}
|
||||
\part Calculer la probabilité de tirer un bonbon à la fraise.
|
||||
\begin{solution}
|
||||
$T($ tirer un bonbon à la fraise $) = \dfrac{80}{174}$
|
||||
\end{solution}
|
||||
\part Calculer la probabilité de tirer un bonbon qui n'est pas au chocolat.
|
||||
\begin{solution}
|
||||
|
||||
$T($ tirer un bonbon à la fraise $) = \dfrac{170}{174}$
|
||||
\end{solution}
|
||||
\part Calculer la probabilité de tirer un bonbon au réglisse.
|
||||
\begin{solution}
|
||||
$T($ tirer un bonbon au réglisse $) = \dfrac{0}{174} = 0$
|
||||
\end{solution}
|
||||
\part Dans un autre sac, on place 25 bonbons à la menthe et 34 bonbons à la fraise. Lise préfère les bonbons à la menthe. Dans quel sac doit-elle tirer un bonbon pour avoir le plus de chance d'avoir un bonbon qu'elle préfère?
|
||||
\end{parts}
|
||||
|
||||
\vfill
|
||||
\end{questions}
|
||||
|
||||
\end{document}
|
||||
|
||||
%%% Local Variables:
|
||||
%%% mode: latex
|
||||
%%% TeX-master: "master"
|
||||
%%% End:
|
78
3e/DM/DM_16_09_15/20_DM_16_09_15.tex
Normal file
78
3e/DM/DM_16_09_15/20_DM_16_09_15.tex
Normal file
@@ -0,0 +1,78 @@
|
||||
\documentclass[a5paper,12pt, table]{/media/documents/Cours/Prof/Enseignements/2016-2017/tools/style/classDS}
|
||||
\usepackage{/media/documents/Cours/Prof/Enseignements/2016-2017/theme}
|
||||
\geometry{left=10mm,right=10mm, top=10mm, bottom=10mm}
|
||||
|
||||
% Title Page
|
||||
\titre{1}
|
||||
% \seconde \premiereS \PSTMG \TSTMG
|
||||
\classe{Troisième}
|
||||
\date{15 septembre 2016}
|
||||
%\duree{1 heure}
|
||||
\sujet{20}
|
||||
% DS DSCorr DM DMCorr Corr
|
||||
\typedoc{DM}
|
||||
|
||||
%\printanswers
|
||||
|
||||
\begin{document}
|
||||
|
||||
\maketitle
|
||||
|
||||
Vous devez rendre le sujet avec la copie.
|
||||
|
||||
\begin{questions}
|
||||
|
||||
\question
|
||||
Faire les calculs suivants en détaillant des étapes.
|
||||
\begin{multicols}{2}
|
||||
\begin{parts}
|
||||
|
||||
\part $7 + 1 \times 7$
|
||||
|
||||
\part $2 - 5 \times ( -5 )$
|
||||
|
||||
\part $-7 \times 3 + 3 \times 6$
|
||||
|
||||
\part $( 2 - 6 ) \times 3 - 7$
|
||||
|
||||
\part $2 ( 10 + 9 ) \times ( -4 )$
|
||||
|
||||
\part $-6 ( 7 - 9 ) - 8$
|
||||
\end{parts}
|
||||
\end{multicols}
|
||||
\vfill
|
||||
|
||||
\question
|
||||
|
||||
|
||||
Dans un sac, il y a 20 bonbons à la menthe, 36 bonbons à la fraise et 6 au chocolat. On choisit un bonbon au hasard dans ce sac.
|
||||
\begin{parts}
|
||||
\part Combien y a-t-il d'issues en tout?
|
||||
\begin{solution}
|
||||
Il y a 62 bonbons.
|
||||
\end{solution}
|
||||
\part Calculer la probabilité de tirer un bonbon à la fraise.
|
||||
\begin{solution}
|
||||
$T($ tirer un bonbon à la fraise $) = \dfrac{20}{62}$
|
||||
\end{solution}
|
||||
\part Calculer la probabilité de tirer un bonbon qui n'est pas au chocolat.
|
||||
\begin{solution}
|
||||
|
||||
$T($ tirer un bonbon à la fraise $) = \dfrac{56}{62}$
|
||||
\end{solution}
|
||||
\part Calculer la probabilité de tirer un bonbon au réglisse.
|
||||
\begin{solution}
|
||||
$T($ tirer un bonbon au réglisse $) = \dfrac{0}{62} = 0$
|
||||
\end{solution}
|
||||
\part Dans un autre sac, on place 25 bonbons à la menthe et 34 bonbons à la fraise. Lise préfère les bonbons à la menthe. Dans quel sac doit-elle tirer un bonbon pour avoir le plus de chance d'avoir un bonbon qu'elle préfère?
|
||||
\end{parts}
|
||||
|
||||
\vfill
|
||||
\end{questions}
|
||||
|
||||
\end{document}
|
||||
|
||||
%%% Local Variables:
|
||||
%%% mode: latex
|
||||
%%% TeX-master: "master"
|
||||
%%% End:
|
78
3e/DM/DM_16_09_15/21_DM_16_09_15.tex
Normal file
78
3e/DM/DM_16_09_15/21_DM_16_09_15.tex
Normal file
@@ -0,0 +1,78 @@
|
||||
\documentclass[a5paper,12pt, table]{/media/documents/Cours/Prof/Enseignements/2016-2017/tools/style/classDS}
|
||||
\usepackage{/media/documents/Cours/Prof/Enseignements/2016-2017/theme}
|
||||
\geometry{left=10mm,right=10mm, top=10mm, bottom=10mm}
|
||||
|
||||
% Title Page
|
||||
\titre{1}
|
||||
% \seconde \premiereS \PSTMG \TSTMG
|
||||
\classe{Troisième}
|
||||
\date{15 septembre 2016}
|
||||
%\duree{1 heure}
|
||||
\sujet{21}
|
||||
% DS DSCorr DM DMCorr Corr
|
||||
\typedoc{DM}
|
||||
|
||||
%\printanswers
|
||||
|
||||
\begin{document}
|
||||
|
||||
\maketitle
|
||||
|
||||
Vous devez rendre le sujet avec la copie.
|
||||
|
||||
\begin{questions}
|
||||
|
||||
\question
|
||||
Faire les calculs suivants en détaillant des étapes.
|
||||
\begin{multicols}{2}
|
||||
\begin{parts}
|
||||
|
||||
\part $-9 - 7 \times 5$
|
||||
|
||||
\part $5 - 9 \times ( -8 )$
|
||||
|
||||
\part $4 \times 5 + 4 \times ( -8 )$
|
||||
|
||||
\part $( -6 + 10 ) \times 9 - 9$
|
||||
|
||||
\part $6 ( 10 - 4 ) \times ( -10 )$
|
||||
|
||||
\part $-7 ( -8 - 7 ) + 5$
|
||||
\end{parts}
|
||||
\end{multicols}
|
||||
\vfill
|
||||
|
||||
\question
|
||||
|
||||
|
||||
Dans un sac, il y a 45 bonbons à la menthe, 27 bonbons à la fraise et 6 au chocolat. On choisit un bonbon au hasard dans ce sac.
|
||||
\begin{parts}
|
||||
\part Combien y a-t-il d'issues en tout?
|
||||
\begin{solution}
|
||||
Il y a 78 bonbons.
|
||||
\end{solution}
|
||||
\part Calculer la probabilité de tirer un bonbon à la fraise.
|
||||
\begin{solution}
|
||||
$T($ tirer un bonbon à la fraise $) = \dfrac{45}{78}$
|
||||
\end{solution}
|
||||
\part Calculer la probabilité de tirer un bonbon qui n'est pas au chocolat.
|
||||
\begin{solution}
|
||||
|
||||
$T($ tirer un bonbon à la fraise $) = \dfrac{72}{78}$
|
||||
\end{solution}
|
||||
\part Calculer la probabilité de tirer un bonbon au réglisse.
|
||||
\begin{solution}
|
||||
$T($ tirer un bonbon au réglisse $) = \dfrac{0}{78} = 0$
|
||||
\end{solution}
|
||||
\part Dans un autre sac, on place 25 bonbons à la menthe et 34 bonbons à la fraise. Lise préfère les bonbons à la menthe. Dans quel sac doit-elle tirer un bonbon pour avoir le plus de chance d'avoir un bonbon qu'elle préfère?
|
||||
\end{parts}
|
||||
|
||||
\vfill
|
||||
\end{questions}
|
||||
|
||||
\end{document}
|
||||
|
||||
%%% Local Variables:
|
||||
%%% mode: latex
|
||||
%%% TeX-master: "master"
|
||||
%%% End:
|
78
3e/DM/DM_16_09_15/22_DM_16_09_15.tex
Normal file
78
3e/DM/DM_16_09_15/22_DM_16_09_15.tex
Normal file
@@ -0,0 +1,78 @@
|
||||
\documentclass[a5paper,12pt, table]{/media/documents/Cours/Prof/Enseignements/2016-2017/tools/style/classDS}
|
||||
\usepackage{/media/documents/Cours/Prof/Enseignements/2016-2017/theme}
|
||||
\geometry{left=10mm,right=10mm, top=10mm, bottom=10mm}
|
||||
|
||||
% Title Page
|
||||
\titre{1}
|
||||
% \seconde \premiereS \PSTMG \TSTMG
|
||||
\classe{Troisième}
|
||||
\date{15 septembre 2016}
|
||||
%\duree{1 heure}
|
||||
\sujet{22}
|
||||
% DS DSCorr DM DMCorr Corr
|
||||
\typedoc{DM}
|
||||
|
||||
%\printanswers
|
||||
|
||||
\begin{document}
|
||||
|
||||
\maketitle
|
||||
|
||||
Vous devez rendre le sujet avec la copie.
|
||||
|
||||
\begin{questions}
|
||||
|
||||
\question
|
||||
Faire les calculs suivants en détaillant des étapes.
|
||||
\begin{multicols}{2}
|
||||
\begin{parts}
|
||||
|
||||
\part $-7 - 8 \times ( -7 )$
|
||||
|
||||
\part $1 - 8 \times ( -1 )$
|
||||
|
||||
\part $1 \times 10 + 5 \times 3$
|
||||
|
||||
\part $( 6 + 1 ) \times ( -1 ) + 6$
|
||||
|
||||
\part $3 ( -2 + 4 ) \times 2$
|
||||
|
||||
\part $3 ( 2 - ( -1 ) ) + 1$
|
||||
\end{parts}
|
||||
\end{multicols}
|
||||
\vfill
|
||||
|
||||
\question
|
||||
|
||||
|
||||
Dans un sac, il y a 6 bonbons à la menthe, 30 bonbons à la fraise et 4 au chocolat. On choisit un bonbon au hasard dans ce sac.
|
||||
\begin{parts}
|
||||
\part Combien y a-t-il d'issues en tout?
|
||||
\begin{solution}
|
||||
Il y a 40 bonbons.
|
||||
\end{solution}
|
||||
\part Calculer la probabilité de tirer un bonbon à la fraise.
|
||||
\begin{solution}
|
||||
$T($ tirer un bonbon à la fraise $) = \dfrac{6}{40}$
|
||||
\end{solution}
|
||||
\part Calculer la probabilité de tirer un bonbon qui n'est pas au chocolat.
|
||||
\begin{solution}
|
||||
|
||||
$T($ tirer un bonbon à la fraise $) = \dfrac{36}{40}$
|
||||
\end{solution}
|
||||
\part Calculer la probabilité de tirer un bonbon au réglisse.
|
||||
\begin{solution}
|
||||
$T($ tirer un bonbon au réglisse $) = \dfrac{0}{40} = 0$
|
||||
\end{solution}
|
||||
\part Dans un autre sac, on place 25 bonbons à la menthe et 34 bonbons à la fraise. Lise préfère les bonbons à la menthe. Dans quel sac doit-elle tirer un bonbon pour avoir le plus de chance d'avoir un bonbon qu'elle préfère?
|
||||
\end{parts}
|
||||
|
||||
\vfill
|
||||
\end{questions}
|
||||
|
||||
\end{document}
|
||||
|
||||
%%% Local Variables:
|
||||
%%% mode: latex
|
||||
%%% TeX-master: "master"
|
||||
%%% End:
|
78
3e/DM/DM_16_09_15/23_DM_16_09_15.tex
Normal file
78
3e/DM/DM_16_09_15/23_DM_16_09_15.tex
Normal file
@@ -0,0 +1,78 @@
|
||||
\documentclass[a5paper,12pt, table]{/media/documents/Cours/Prof/Enseignements/2016-2017/tools/style/classDS}
|
||||
\usepackage{/media/documents/Cours/Prof/Enseignements/2016-2017/theme}
|
||||
\geometry{left=10mm,right=10mm, top=10mm, bottom=10mm}
|
||||
|
||||
% Title Page
|
||||
\titre{1}
|
||||
% \seconde \premiereS \PSTMG \TSTMG
|
||||
\classe{Troisième}
|
||||
\date{15 septembre 2016}
|
||||
%\duree{1 heure}
|
||||
\sujet{23}
|
||||
% DS DSCorr DM DMCorr Corr
|
||||
\typedoc{DM}
|
||||
|
||||
%\printanswers
|
||||
|
||||
\begin{document}
|
||||
|
||||
\maketitle
|
||||
|
||||
Vous devez rendre le sujet avec la copie.
|
||||
|
||||
\begin{questions}
|
||||
|
||||
\question
|
||||
Faire les calculs suivants en détaillant des étapes.
|
||||
\begin{multicols}{2}
|
||||
\begin{parts}
|
||||
|
||||
\part $8 + 3 \times 1$
|
||||
|
||||
\part $-8 - ( -5 \times ( -4 ) )$
|
||||
|
||||
\part $-2 \times 2 + 2 \times ( -3 )$
|
||||
|
||||
\part $( -8 - 1 ) \times 6 - 6$
|
||||
|
||||
\part $-3 ( -6 - 1 ) \times 1$
|
||||
|
||||
\part $9 ( 1 - 5 ) + 9$
|
||||
\end{parts}
|
||||
\end{multicols}
|
||||
\vfill
|
||||
|
||||
\question
|
||||
|
||||
|
||||
Dans un sac, il y a 60 bonbons à la menthe, 42 bonbons à la fraise et 10 au chocolat. On choisit un bonbon au hasard dans ce sac.
|
||||
\begin{parts}
|
||||
\part Combien y a-t-il d'issues en tout?
|
||||
\begin{solution}
|
||||
Il y a 112 bonbons.
|
||||
\end{solution}
|
||||
\part Calculer la probabilité de tirer un bonbon à la fraise.
|
||||
\begin{solution}
|
||||
$T($ tirer un bonbon à la fraise $) = \dfrac{60}{112}$
|
||||
\end{solution}
|
||||
\part Calculer la probabilité de tirer un bonbon qui n'est pas au chocolat.
|
||||
\begin{solution}
|
||||
|
||||
$T($ tirer un bonbon à la fraise $) = \dfrac{102}{112}$
|
||||
\end{solution}
|
||||
\part Calculer la probabilité de tirer un bonbon au réglisse.
|
||||
\begin{solution}
|
||||
$T($ tirer un bonbon au réglisse $) = \dfrac{0}{112} = 0$
|
||||
\end{solution}
|
||||
\part Dans un autre sac, on place 25 bonbons à la menthe et 34 bonbons à la fraise. Lise préfère les bonbons à la menthe. Dans quel sac doit-elle tirer un bonbon pour avoir le plus de chance d'avoir un bonbon qu'elle préfère?
|
||||
\end{parts}
|
||||
|
||||
\vfill
|
||||
\end{questions}
|
||||
|
||||
\end{document}
|
||||
|
||||
%%% Local Variables:
|
||||
%%% mode: latex
|
||||
%%% TeX-master: "master"
|
||||
%%% End:
|
78
3e/DM/DM_16_09_15/24_DM_16_09_15.tex
Normal file
78
3e/DM/DM_16_09_15/24_DM_16_09_15.tex
Normal file
@@ -0,0 +1,78 @@
|
||||
\documentclass[a5paper,12pt, table]{/media/documents/Cours/Prof/Enseignements/2016-2017/tools/style/classDS}
|
||||
\usepackage{/media/documents/Cours/Prof/Enseignements/2016-2017/theme}
|
||||
\geometry{left=10mm,right=10mm, top=10mm, bottom=10mm}
|
||||
|
||||
% Title Page
|
||||
\titre{1}
|
||||
% \seconde \premiereS \PSTMG \TSTMG
|
||||
\classe{Troisième}
|
||||
\date{15 septembre 2016}
|
||||
%\duree{1 heure}
|
||||
\sujet{24}
|
||||
% DS DSCorr DM DMCorr Corr
|
||||
\typedoc{DM}
|
||||
|
||||
%\printanswers
|
||||
|
||||
\begin{document}
|
||||
|
||||
\maketitle
|
||||
|
||||
Vous devez rendre le sujet avec la copie.
|
||||
|
||||
\begin{questions}
|
||||
|
||||
\question
|
||||
Faire les calculs suivants en détaillant des étapes.
|
||||
\begin{multicols}{2}
|
||||
\begin{parts}
|
||||
|
||||
\part $-8 + 5 \times 3$
|
||||
|
||||
\part $-6 - 5 \times 10$
|
||||
|
||||
\part $4 \times 6 + 3 \times 10$
|
||||
|
||||
\part $( 10 + 4 ) \times 3 - 1$
|
||||
|
||||
\part $-8 ( 2 - 10 ) \times ( -7 )$
|
||||
|
||||
\part $4 ( -8 - ( -7 ) ) + 10$
|
||||
\end{parts}
|
||||
\end{multicols}
|
||||
\vfill
|
||||
|
||||
\question
|
||||
|
||||
|
||||
Dans un sac, il y a 15 bonbons à la menthe, 27 bonbons à la fraise et 10 au chocolat. On choisit un bonbon au hasard dans ce sac.
|
||||
\begin{parts}
|
||||
\part Combien y a-t-il d'issues en tout?
|
||||
\begin{solution}
|
||||
Il y a 52 bonbons.
|
||||
\end{solution}
|
||||
\part Calculer la probabilité de tirer un bonbon à la fraise.
|
||||
\begin{solution}
|
||||
$T($ tirer un bonbon à la fraise $) = \dfrac{15}{52}$
|
||||
\end{solution}
|
||||
\part Calculer la probabilité de tirer un bonbon qui n'est pas au chocolat.
|
||||
\begin{solution}
|
||||
|
||||
$T($ tirer un bonbon à la fraise $) = \dfrac{42}{52}$
|
||||
\end{solution}
|
||||
\part Calculer la probabilité de tirer un bonbon au réglisse.
|
||||
\begin{solution}
|
||||
$T($ tirer un bonbon au réglisse $) = \dfrac{0}{52} = 0$
|
||||
\end{solution}
|
||||
\part Dans un autre sac, on place 25 bonbons à la menthe et 34 bonbons à la fraise. Lise préfère les bonbons à la menthe. Dans quel sac doit-elle tirer un bonbon pour avoir le plus de chance d'avoir un bonbon qu'elle préfère?
|
||||
\end{parts}
|
||||
|
||||
\vfill
|
||||
\end{questions}
|
||||
|
||||
\end{document}
|
||||
|
||||
%%% Local Variables:
|
||||
%%% mode: latex
|
||||
%%% TeX-master: "master"
|
||||
%%% End:
|
78
3e/DM/DM_16_09_15/25_DM_16_09_15.tex
Normal file
78
3e/DM/DM_16_09_15/25_DM_16_09_15.tex
Normal file
@@ -0,0 +1,78 @@
|
||||
\documentclass[a5paper,12pt, table]{/media/documents/Cours/Prof/Enseignements/2016-2017/tools/style/classDS}
|
||||
\usepackage{/media/documents/Cours/Prof/Enseignements/2016-2017/theme}
|
||||
\geometry{left=10mm,right=10mm, top=10mm, bottom=10mm}
|
||||
|
||||
% Title Page
|
||||
\titre{1}
|
||||
% \seconde \premiereS \PSTMG \TSTMG
|
||||
\classe{Troisième}
|
||||
\date{15 septembre 2016}
|
||||
%\duree{1 heure}
|
||||
\sujet{25}
|
||||
% DS DSCorr DM DMCorr Corr
|
||||
\typedoc{DM}
|
||||
|
||||
%\printanswers
|
||||
|
||||
\begin{document}
|
||||
|
||||
\maketitle
|
||||
|
||||
Vous devez rendre le sujet avec la copie.
|
||||
|
||||
\begin{questions}
|
||||
|
||||
\question
|
||||
Faire les calculs suivants en détaillant des étapes.
|
||||
\begin{multicols}{2}
|
||||
\begin{parts}
|
||||
|
||||
\part $9 + 2 \times ( -7 )$
|
||||
|
||||
\part $-5 - ( -6 \times 5 )$
|
||||
|
||||
\part $1 \times 5 + 10 \times ( -2 )$
|
||||
|
||||
\part $( -9 + 9 ) \times 9 - 5$
|
||||
|
||||
\part $-4 ( -6 - 10 ) \times 10$
|
||||
|
||||
\part $-7 ( 6 - ( -9 ) ) - 1$
|
||||
\end{parts}
|
||||
\end{multicols}
|
||||
\vfill
|
||||
|
||||
\question
|
||||
|
||||
|
||||
Dans un sac, il y a 8 bonbons à la menthe, 18 bonbons à la fraise et 7 au chocolat. On choisit un bonbon au hasard dans ce sac.
|
||||
\begin{parts}
|
||||
\part Combien y a-t-il d'issues en tout?
|
||||
\begin{solution}
|
||||
Il y a 33 bonbons.
|
||||
\end{solution}
|
||||
\part Calculer la probabilité de tirer un bonbon à la fraise.
|
||||
\begin{solution}
|
||||
$T($ tirer un bonbon à la fraise $) = \dfrac{8}{33}$
|
||||
\end{solution}
|
||||
\part Calculer la probabilité de tirer un bonbon qui n'est pas au chocolat.
|
||||
\begin{solution}
|
||||
|
||||
$T($ tirer un bonbon à la fraise $) = \dfrac{26}{33}$
|
||||
\end{solution}
|
||||
\part Calculer la probabilité de tirer un bonbon au réglisse.
|
||||
\begin{solution}
|
||||
$T($ tirer un bonbon au réglisse $) = \dfrac{0}{33} = 0$
|
||||
\end{solution}
|
||||
\part Dans un autre sac, on place 25 bonbons à la menthe et 34 bonbons à la fraise. Lise préfère les bonbons à la menthe. Dans quel sac doit-elle tirer un bonbon pour avoir le plus de chance d'avoir un bonbon qu'elle préfère?
|
||||
\end{parts}
|
||||
|
||||
\vfill
|
||||
\end{questions}
|
||||
|
||||
\end{document}
|
||||
|
||||
%%% Local Variables:
|
||||
%%% mode: latex
|
||||
%%% TeX-master: "master"
|
||||
%%% End:
|
78
3e/DM/DM_16_09_15/26_DM_16_09_15.tex
Normal file
78
3e/DM/DM_16_09_15/26_DM_16_09_15.tex
Normal file
@@ -0,0 +1,78 @@
|
||||
\documentclass[a5paper,12pt, table]{/media/documents/Cours/Prof/Enseignements/2016-2017/tools/style/classDS}
|
||||
\usepackage{/media/documents/Cours/Prof/Enseignements/2016-2017/theme}
|
||||
\geometry{left=10mm,right=10mm, top=10mm, bottom=10mm}
|
||||
|
||||
% Title Page
|
||||
\titre{1}
|
||||
% \seconde \premiereS \PSTMG \TSTMG
|
||||
\classe{Troisième}
|
||||
\date{15 septembre 2016}
|
||||
%\duree{1 heure}
|
||||
\sujet{26}
|
||||
% DS DSCorr DM DMCorr Corr
|
||||
\typedoc{DM}
|
||||
|
||||
%\printanswers
|
||||
|
||||
\begin{document}
|
||||
|
||||
\maketitle
|
||||
|
||||
Vous devez rendre le sujet avec la copie.
|
||||
|
||||
\begin{questions}
|
||||
|
||||
\question
|
||||
Faire les calculs suivants en détaillant des étapes.
|
||||
\begin{multicols}{2}
|
||||
\begin{parts}
|
||||
|
||||
\part $-2 + 4 \times ( -9 )$
|
||||
|
||||
\part $-9 - ( -10 \times ( -10 ) )$
|
||||
|
||||
\part $6 \times ( -1 ) - 10 \times 5$
|
||||
|
||||
\part $( -1 - 8 ) \times 8 + 1$
|
||||
|
||||
\part $-6 ( -6 - 4 ) \times 6$
|
||||
|
||||
\part $-1 ( 5 - ( -8 ) ) - 6$
|
||||
\end{parts}
|
||||
\end{multicols}
|
||||
\vfill
|
||||
|
||||
\question
|
||||
|
||||
|
||||
Dans un sac, il y a 16 bonbons à la menthe, 36 bonbons à la fraise et 9 au chocolat. On choisit un bonbon au hasard dans ce sac.
|
||||
\begin{parts}
|
||||
\part Combien y a-t-il d'issues en tout?
|
||||
\begin{solution}
|
||||
Il y a 61 bonbons.
|
||||
\end{solution}
|
||||
\part Calculer la probabilité de tirer un bonbon à la fraise.
|
||||
\begin{solution}
|
||||
$T($ tirer un bonbon à la fraise $) = \dfrac{16}{61}$
|
||||
\end{solution}
|
||||
\part Calculer la probabilité de tirer un bonbon qui n'est pas au chocolat.
|
||||
\begin{solution}
|
||||
|
||||
$T($ tirer un bonbon à la fraise $) = \dfrac{52}{61}$
|
||||
\end{solution}
|
||||
\part Calculer la probabilité de tirer un bonbon au réglisse.
|
||||
\begin{solution}
|
||||
$T($ tirer un bonbon au réglisse $) = \dfrac{0}{61} = 0$
|
||||
\end{solution}
|
||||
\part Dans un autre sac, on place 25 bonbons à la menthe et 34 bonbons à la fraise. Lise préfère les bonbons à la menthe. Dans quel sac doit-elle tirer un bonbon pour avoir le plus de chance d'avoir un bonbon qu'elle préfère?
|
||||
\end{parts}
|
||||
|
||||
\vfill
|
||||
\end{questions}
|
||||
|
||||
\end{document}
|
||||
|
||||
%%% Local Variables:
|
||||
%%% mode: latex
|
||||
%%% TeX-master: "master"
|
||||
%%% End:
|
78
3e/DM/DM_16_09_15/27_DM_16_09_15.tex
Normal file
78
3e/DM/DM_16_09_15/27_DM_16_09_15.tex
Normal file
@@ -0,0 +1,78 @@
|
||||
\documentclass[a5paper,12pt, table]{/media/documents/Cours/Prof/Enseignements/2016-2017/tools/style/classDS}
|
||||
\usepackage{/media/documents/Cours/Prof/Enseignements/2016-2017/theme}
|
||||
\geometry{left=10mm,right=10mm, top=10mm, bottom=10mm}
|
||||
|
||||
% Title Page
|
||||
\titre{1}
|
||||
% \seconde \premiereS \PSTMG \TSTMG
|
||||
\classe{Troisième}
|
||||
\date{15 septembre 2016}
|
||||
%\duree{1 heure}
|
||||
\sujet{27}
|
||||
% DS DSCorr DM DMCorr Corr
|
||||
\typedoc{DM}
|
||||
|
||||
%\printanswers
|
||||
|
||||
\begin{document}
|
||||
|
||||
\maketitle
|
||||
|
||||
Vous devez rendre le sujet avec la copie.
|
||||
|
||||
\begin{questions}
|
||||
|
||||
\question
|
||||
Faire les calculs suivants en détaillant des étapes.
|
||||
\begin{multicols}{2}
|
||||
\begin{parts}
|
||||
|
||||
\part $-4 - 8 \times 7$
|
||||
|
||||
\part $-10 - ( -3 \times 3 )$
|
||||
|
||||
\part $1 \times 4 - 3 \times 2$
|
||||
|
||||
\part $( -5 + 7 ) \times 6 - 2$
|
||||
|
||||
\part $-4 ( -4 - 10 ) \times ( -2 )$
|
||||
|
||||
\part $9 ( 9 - ( -4 ) ) - 8$
|
||||
\end{parts}
|
||||
\end{multicols}
|
||||
\vfill
|
||||
|
||||
\question
|
||||
|
||||
|
||||
Dans un sac, il y a 40 bonbons à la menthe, 20 bonbons à la fraise et 4 au chocolat. On choisit un bonbon au hasard dans ce sac.
|
||||
\begin{parts}
|
||||
\part Combien y a-t-il d'issues en tout?
|
||||
\begin{solution}
|
||||
Il y a 64 bonbons.
|
||||
\end{solution}
|
||||
\part Calculer la probabilité de tirer un bonbon à la fraise.
|
||||
\begin{solution}
|
||||
$T($ tirer un bonbon à la fraise $) = \dfrac{40}{64}$
|
||||
\end{solution}
|
||||
\part Calculer la probabilité de tirer un bonbon qui n'est pas au chocolat.
|
||||
\begin{solution}
|
||||
|
||||
$T($ tirer un bonbon à la fraise $) = \dfrac{60}{64}$
|
||||
\end{solution}
|
||||
\part Calculer la probabilité de tirer un bonbon au réglisse.
|
||||
\begin{solution}
|
||||
$T($ tirer un bonbon au réglisse $) = \dfrac{0}{64} = 0$
|
||||
\end{solution}
|
||||
\part Dans un autre sac, on place 25 bonbons à la menthe et 34 bonbons à la fraise. Lise préfère les bonbons à la menthe. Dans quel sac doit-elle tirer un bonbon pour avoir le plus de chance d'avoir un bonbon qu'elle préfère?
|
||||
\end{parts}
|
||||
|
||||
\vfill
|
||||
\end{questions}
|
||||
|
||||
\end{document}
|
||||
|
||||
%%% Local Variables:
|
||||
%%% mode: latex
|
||||
%%% TeX-master: "master"
|
||||
%%% End:
|
78
3e/DM/DM_16_09_15/28_DM_16_09_15.tex
Normal file
78
3e/DM/DM_16_09_15/28_DM_16_09_15.tex
Normal file
@@ -0,0 +1,78 @@
|
||||
\documentclass[a5paper,12pt, table]{/media/documents/Cours/Prof/Enseignements/2016-2017/tools/style/classDS}
|
||||
\usepackage{/media/documents/Cours/Prof/Enseignements/2016-2017/theme}
|
||||
\geometry{left=10mm,right=10mm, top=10mm, bottom=10mm}
|
||||
|
||||
% Title Page
|
||||
\titre{1}
|
||||
% \seconde \premiereS \PSTMG \TSTMG
|
||||
\classe{Troisième}
|
||||
\date{15 septembre 2016}
|
||||
%\duree{1 heure}
|
||||
\sujet{28}
|
||||
% DS DSCorr DM DMCorr Corr
|
||||
\typedoc{DM}
|
||||
|
||||
%\printanswers
|
||||
|
||||
\begin{document}
|
||||
|
||||
\maketitle
|
||||
|
||||
Vous devez rendre le sujet avec la copie.
|
||||
|
||||
\begin{questions}
|
||||
|
||||
\question
|
||||
Faire les calculs suivants en détaillant des étapes.
|
||||
\begin{multicols}{2}
|
||||
\begin{parts}
|
||||
|
||||
\part $6 - 3 \times ( -1 )$
|
||||
|
||||
\part $6 - 2 \times ( -1 )$
|
||||
|
||||
\part $-7 \times 10 + 5 \times 3$
|
||||
|
||||
\part $( -4 + 6 ) \times ( -2 ) - 4$
|
||||
|
||||
\part $3 ( 2 + 8 ) \times ( -5 )$
|
||||
|
||||
\part $-5 ( 9 - ( -10 ) ) - 9$
|
||||
\end{parts}
|
||||
\end{multicols}
|
||||
\vfill
|
||||
|
||||
\question
|
||||
|
||||
|
||||
Dans un sac, il y a 81 bonbons à la menthe, 63 bonbons à la fraise et 6 au chocolat. On choisit un bonbon au hasard dans ce sac.
|
||||
\begin{parts}
|
||||
\part Combien y a-t-il d'issues en tout?
|
||||
\begin{solution}
|
||||
Il y a 150 bonbons.
|
||||
\end{solution}
|
||||
\part Calculer la probabilité de tirer un bonbon à la fraise.
|
||||
\begin{solution}
|
||||
$T($ tirer un bonbon à la fraise $) = \dfrac{81}{150}$
|
||||
\end{solution}
|
||||
\part Calculer la probabilité de tirer un bonbon qui n'est pas au chocolat.
|
||||
\begin{solution}
|
||||
|
||||
$T($ tirer un bonbon à la fraise $) = \dfrac{144}{150}$
|
||||
\end{solution}
|
||||
\part Calculer la probabilité de tirer un bonbon au réglisse.
|
||||
\begin{solution}
|
||||
$T($ tirer un bonbon au réglisse $) = \dfrac{0}{150} = 0$
|
||||
\end{solution}
|
||||
\part Dans un autre sac, on place 25 bonbons à la menthe et 34 bonbons à la fraise. Lise préfère les bonbons à la menthe. Dans quel sac doit-elle tirer un bonbon pour avoir le plus de chance d'avoir un bonbon qu'elle préfère?
|
||||
\end{parts}
|
||||
|
||||
\vfill
|
||||
\end{questions}
|
||||
|
||||
\end{document}
|
||||
|
||||
%%% Local Variables:
|
||||
%%% mode: latex
|
||||
%%% TeX-master: "master"
|
||||
%%% End:
|
78
3e/DM/DM_16_09_15/29_DM_16_09_15.tex
Normal file
78
3e/DM/DM_16_09_15/29_DM_16_09_15.tex
Normal file
@@ -0,0 +1,78 @@
|
||||
\documentclass[a5paper,12pt, table]{/media/documents/Cours/Prof/Enseignements/2016-2017/tools/style/classDS}
|
||||
\usepackage{/media/documents/Cours/Prof/Enseignements/2016-2017/theme}
|
||||
\geometry{left=10mm,right=10mm, top=10mm, bottom=10mm}
|
||||
|
||||
% Title Page
|
||||
\titre{1}
|
||||
% \seconde \premiereS \PSTMG \TSTMG
|
||||
\classe{Troisième}
|
||||
\date{15 septembre 2016}
|
||||
%\duree{1 heure}
|
||||
\sujet{29}
|
||||
% DS DSCorr DM DMCorr Corr
|
||||
\typedoc{DM}
|
||||
|
||||
%\printanswers
|
||||
|
||||
\begin{document}
|
||||
|
||||
\maketitle
|
||||
|
||||
Vous devez rendre le sujet avec la copie.
|
||||
|
||||
\begin{questions}
|
||||
|
||||
\question
|
||||
Faire les calculs suivants en détaillant des étapes.
|
||||
\begin{multicols}{2}
|
||||
\begin{parts}
|
||||
|
||||
\part $8 + 5 \times 8$
|
||||
|
||||
\part $8 - ( -4 \times ( -7 ) )$
|
||||
|
||||
\part $8 \times 7 + 3 \times ( -8 )$
|
||||
|
||||
\part $( -4 - 3 ) \times ( -8 ) + 9$
|
||||
|
||||
\part $-7 ( 3 - 8 ) \times 1$
|
||||
|
||||
\part $-8 ( 3 - 1 ) - 8$
|
||||
\end{parts}
|
||||
\end{multicols}
|
||||
\vfill
|
||||
|
||||
\question
|
||||
|
||||
|
||||
Dans un sac, il y a 27 bonbons à la menthe, 9 bonbons à la fraise et 8 au chocolat. On choisit un bonbon au hasard dans ce sac.
|
||||
\begin{parts}
|
||||
\part Combien y a-t-il d'issues en tout?
|
||||
\begin{solution}
|
||||
Il y a 44 bonbons.
|
||||
\end{solution}
|
||||
\part Calculer la probabilité de tirer un bonbon à la fraise.
|
||||
\begin{solution}
|
||||
$T($ tirer un bonbon à la fraise $) = \dfrac{27}{44}$
|
||||
\end{solution}
|
||||
\part Calculer la probabilité de tirer un bonbon qui n'est pas au chocolat.
|
||||
\begin{solution}
|
||||
|
||||
$T($ tirer un bonbon à la fraise $) = \dfrac{36}{44}$
|
||||
\end{solution}
|
||||
\part Calculer la probabilité de tirer un bonbon au réglisse.
|
||||
\begin{solution}
|
||||
$T($ tirer un bonbon au réglisse $) = \dfrac{0}{44} = 0$
|
||||
\end{solution}
|
||||
\part Dans un autre sac, on place 25 bonbons à la menthe et 34 bonbons à la fraise. Lise préfère les bonbons à la menthe. Dans quel sac doit-elle tirer un bonbon pour avoir le plus de chance d'avoir un bonbon qu'elle préfère?
|
||||
\end{parts}
|
||||
|
||||
\vfill
|
||||
\end{questions}
|
||||
|
||||
\end{document}
|
||||
|
||||
%%% Local Variables:
|
||||
%%% mode: latex
|
||||
%%% TeX-master: "master"
|
||||
%%% End:
|
78
3e/DM/DM_16_09_15/30_DM_16_09_15.tex
Normal file
78
3e/DM/DM_16_09_15/30_DM_16_09_15.tex
Normal file
@@ -0,0 +1,78 @@
|
||||
\documentclass[a5paper,12pt, table]{/media/documents/Cours/Prof/Enseignements/2016-2017/tools/style/classDS}
|
||||
\usepackage{/media/documents/Cours/Prof/Enseignements/2016-2017/theme}
|
||||
\geometry{left=10mm,right=10mm, top=10mm, bottom=10mm}
|
||||
|
||||
% Title Page
|
||||
\titre{1}
|
||||
% \seconde \premiereS \PSTMG \TSTMG
|
||||
\classe{Troisième}
|
||||
\date{15 septembre 2016}
|
||||
%\duree{1 heure}
|
||||
\sujet{30}
|
||||
% DS DSCorr DM DMCorr Corr
|
||||
\typedoc{DM}
|
||||
|
||||
%\printanswers
|
||||
|
||||
\begin{document}
|
||||
|
||||
\maketitle
|
||||
|
||||
Vous devez rendre le sujet avec la copie.
|
||||
|
||||
\begin{questions}
|
||||
|
||||
\question
|
||||
Faire les calculs suivants en détaillant des étapes.
|
||||
\begin{multicols}{2}
|
||||
\begin{parts}
|
||||
|
||||
\part $-10 + 9 \times ( -5 )$
|
||||
|
||||
\part $4 - ( -7 \times 9 )$
|
||||
|
||||
\part $5 \times ( -3 ) - 1 \times ( -6 )$
|
||||
|
||||
\part $( 5 + 1 ) \times 9 - 3$
|
||||
|
||||
\part $9 ( -9 + 2 ) \times 1$
|
||||
|
||||
\part $2 ( -4 - ( -6 ) ) - 4$
|
||||
\end{parts}
|
||||
\end{multicols}
|
||||
\vfill
|
||||
|
||||
\question
|
||||
|
||||
|
||||
Dans un sac, il y a 6 bonbons à la menthe, 9 bonbons à la fraise et 7 au chocolat. On choisit un bonbon au hasard dans ce sac.
|
||||
\begin{parts}
|
||||
\part Combien y a-t-il d'issues en tout?
|
||||
\begin{solution}
|
||||
Il y a 22 bonbons.
|
||||
\end{solution}
|
||||
\part Calculer la probabilité de tirer un bonbon à la fraise.
|
||||
\begin{solution}
|
||||
$T($ tirer un bonbon à la fraise $) = \dfrac{6}{22}$
|
||||
\end{solution}
|
||||
\part Calculer la probabilité de tirer un bonbon qui n'est pas au chocolat.
|
||||
\begin{solution}
|
||||
|
||||
$T($ tirer un bonbon à la fraise $) = \dfrac{15}{22}$
|
||||
\end{solution}
|
||||
\part Calculer la probabilité de tirer un bonbon au réglisse.
|
||||
\begin{solution}
|
||||
$T($ tirer un bonbon au réglisse $) = \dfrac{0}{22} = 0$
|
||||
\end{solution}
|
||||
\part Dans un autre sac, on place 25 bonbons à la menthe et 34 bonbons à la fraise. Lise préfère les bonbons à la menthe. Dans quel sac doit-elle tirer un bonbon pour avoir le plus de chance d'avoir un bonbon qu'elle préfère?
|
||||
\end{parts}
|
||||
|
||||
\vfill
|
||||
\end{questions}
|
||||
|
||||
\end{document}
|
||||
|
||||
%%% Local Variables:
|
||||
%%% mode: latex
|
||||
%%% TeX-master: "master"
|
||||
%%% End:
|
78
3e/DM/DM_16_09_15/31_DM_16_09_15.tex
Normal file
78
3e/DM/DM_16_09_15/31_DM_16_09_15.tex
Normal file
@@ -0,0 +1,78 @@
|
||||
\documentclass[a5paper,12pt, table]{/media/documents/Cours/Prof/Enseignements/2016-2017/tools/style/classDS}
|
||||
\usepackage{/media/documents/Cours/Prof/Enseignements/2016-2017/theme}
|
||||
\geometry{left=10mm,right=10mm, top=10mm, bottom=10mm}
|
||||
|
||||
% Title Page
|
||||
\titre{1}
|
||||
% \seconde \premiereS \PSTMG \TSTMG
|
||||
\classe{Troisième}
|
||||
\date{15 septembre 2016}
|
||||
%\duree{1 heure}
|
||||
\sujet{31}
|
||||
% DS DSCorr DM DMCorr Corr
|
||||
\typedoc{DM}
|
||||
|
||||
%\printanswers
|
||||
|
||||
\begin{document}
|
||||
|
||||
\maketitle
|
||||
|
||||
Vous devez rendre le sujet avec la copie.
|
||||
|
||||
\begin{questions}
|
||||
|
||||
\question
|
||||
Faire les calculs suivants en détaillant des étapes.
|
||||
\begin{multicols}{2}
|
||||
\begin{parts}
|
||||
|
||||
\part $1 - 8 \times ( -7 )$
|
||||
|
||||
\part $10 - 2 \times ( -8 )$
|
||||
|
||||
\part $-10 \times ( -8 ) - 5 \times 4$
|
||||
|
||||
\part $( -6 + 7 ) \times ( -9 ) - 4$
|
||||
|
||||
\part $-8 ( -5 - 7 ) \times 2$
|
||||
|
||||
\part $4 ( -1 - 5 ) + 7$
|
||||
\end{parts}
|
||||
\end{multicols}
|
||||
\vfill
|
||||
|
||||
\question
|
||||
|
||||
|
||||
Dans un sac, il y a 100 bonbons à la menthe, 90 bonbons à la fraise et 9 au chocolat. On choisit un bonbon au hasard dans ce sac.
|
||||
\begin{parts}
|
||||
\part Combien y a-t-il d'issues en tout?
|
||||
\begin{solution}
|
||||
Il y a 199 bonbons.
|
||||
\end{solution}
|
||||
\part Calculer la probabilité de tirer un bonbon à la fraise.
|
||||
\begin{solution}
|
||||
$T($ tirer un bonbon à la fraise $) = \dfrac{100}{199}$
|
||||
\end{solution}
|
||||
\part Calculer la probabilité de tirer un bonbon qui n'est pas au chocolat.
|
||||
\begin{solution}
|
||||
|
||||
$T($ tirer un bonbon à la fraise $) = \dfrac{190}{199}$
|
||||
\end{solution}
|
||||
\part Calculer la probabilité de tirer un bonbon au réglisse.
|
||||
\begin{solution}
|
||||
$T($ tirer un bonbon au réglisse $) = \dfrac{0}{199} = 0$
|
||||
\end{solution}
|
||||
\part Dans un autre sac, on place 25 bonbons à la menthe et 34 bonbons à la fraise. Lise préfère les bonbons à la menthe. Dans quel sac doit-elle tirer un bonbon pour avoir le plus de chance d'avoir un bonbon qu'elle préfère?
|
||||
\end{parts}
|
||||
|
||||
\vfill
|
||||
\end{questions}
|
||||
|
||||
\end{document}
|
||||
|
||||
%%% Local Variables:
|
||||
%%% mode: latex
|
||||
%%% TeX-master: "master"
|
||||
%%% End:
|
78
3e/DM/DM_16_09_15/32_DM_16_09_15.tex
Normal file
78
3e/DM/DM_16_09_15/32_DM_16_09_15.tex
Normal file
@@ -0,0 +1,78 @@
|
||||
\documentclass[a5paper,12pt, table]{/media/documents/Cours/Prof/Enseignements/2016-2017/tools/style/classDS}
|
||||
\usepackage{/media/documents/Cours/Prof/Enseignements/2016-2017/theme}
|
||||
\geometry{left=10mm,right=10mm, top=10mm, bottom=10mm}
|
||||
|
||||
% Title Page
|
||||
\titre{1}
|
||||
% \seconde \premiereS \PSTMG \TSTMG
|
||||
\classe{Troisième}
|
||||
\date{15 septembre 2016}
|
||||
%\duree{1 heure}
|
||||
\sujet{32}
|
||||
% DS DSCorr DM DMCorr Corr
|
||||
\typedoc{DM}
|
||||
|
||||
%\printanswers
|
||||
|
||||
\begin{document}
|
||||
|
||||
\maketitle
|
||||
|
||||
Vous devez rendre le sujet avec la copie.
|
||||
|
||||
\begin{questions}
|
||||
|
||||
\question
|
||||
Faire les calculs suivants en détaillant des étapes.
|
||||
\begin{multicols}{2}
|
||||
\begin{parts}
|
||||
|
||||
\part $7 + 5 \times ( -4 )$
|
||||
|
||||
\part $-4 - 2 \times 8$
|
||||
|
||||
\part $-2 \times ( -9 ) - 8 \times 6$
|
||||
|
||||
\part $( -1 + 6 ) \times 1 - 1$
|
||||
|
||||
\part $1 ( -2 + 3 ) \times 10$
|
||||
|
||||
\part $-7 ( -3 - ( -6 ) ) - 4$
|
||||
\end{parts}
|
||||
\end{multicols}
|
||||
\vfill
|
||||
|
||||
\question
|
||||
|
||||
|
||||
Dans un sac, il y a 16 bonbons à la menthe, 40 bonbons à la fraise et 2 au chocolat. On choisit un bonbon au hasard dans ce sac.
|
||||
\begin{parts}
|
||||
\part Combien y a-t-il d'issues en tout?
|
||||
\begin{solution}
|
||||
Il y a 58 bonbons.
|
||||
\end{solution}
|
||||
\part Calculer la probabilité de tirer un bonbon à la fraise.
|
||||
\begin{solution}
|
||||
$T($ tirer un bonbon à la fraise $) = \dfrac{16}{58}$
|
||||
\end{solution}
|
||||
\part Calculer la probabilité de tirer un bonbon qui n'est pas au chocolat.
|
||||
\begin{solution}
|
||||
|
||||
$T($ tirer un bonbon à la fraise $) = \dfrac{56}{58}$
|
||||
\end{solution}
|
||||
\part Calculer la probabilité de tirer un bonbon au réglisse.
|
||||
\begin{solution}
|
||||
$T($ tirer un bonbon au réglisse $) = \dfrac{0}{58} = 0$
|
||||
\end{solution}
|
||||
\part Dans un autre sac, on place 25 bonbons à la menthe et 34 bonbons à la fraise. Lise préfère les bonbons à la menthe. Dans quel sac doit-elle tirer un bonbon pour avoir le plus de chance d'avoir un bonbon qu'elle préfère?
|
||||
\end{parts}
|
||||
|
||||
\vfill
|
||||
\end{questions}
|
||||
|
||||
\end{document}
|
||||
|
||||
%%% Local Variables:
|
||||
%%% mode: latex
|
||||
%%% TeX-master: "master"
|
||||
%%% End:
|
78
3e/DM/DM_16_09_15/33_DM_16_09_15.tex
Normal file
78
3e/DM/DM_16_09_15/33_DM_16_09_15.tex
Normal file
@@ -0,0 +1,78 @@
|
||||
\documentclass[a5paper,12pt, table]{/media/documents/Cours/Prof/Enseignements/2016-2017/tools/style/classDS}
|
||||
\usepackage{/media/documents/Cours/Prof/Enseignements/2016-2017/theme}
|
||||
\geometry{left=10mm,right=10mm, top=10mm, bottom=10mm}
|
||||
|
||||
% Title Page
|
||||
\titre{1}
|
||||
% \seconde \premiereS \PSTMG \TSTMG
|
||||
\classe{Troisième}
|
||||
\date{15 septembre 2016}
|
||||
%\duree{1 heure}
|
||||
\sujet{33}
|
||||
% DS DSCorr DM DMCorr Corr
|
||||
\typedoc{DM}
|
||||
|
||||
%\printanswers
|
||||
|
||||
\begin{document}
|
||||
|
||||
\maketitle
|
||||
|
||||
Vous devez rendre le sujet avec la copie.
|
||||
|
||||
\begin{questions}
|
||||
|
||||
\question
|
||||
Faire les calculs suivants en détaillant des étapes.
|
||||
\begin{multicols}{2}
|
||||
\begin{parts}
|
||||
|
||||
\part $-3 + 9 \times ( -5 )$
|
||||
|
||||
\part $-4 - 2 \times 8$
|
||||
|
||||
\part $-7 \times ( -4 ) - 4 \times ( -4 )$
|
||||
|
||||
\part $( -5 - 9 ) \times 8 + 9$
|
||||
|
||||
\part $-9 ( 2 + 5 ) \times ( -7 )$
|
||||
|
||||
\part $7 ( -1 - ( -2 ) ) - 5$
|
||||
\end{parts}
|
||||
\end{multicols}
|
||||
\vfill
|
||||
|
||||
\question
|
||||
|
||||
|
||||
Dans un sac, il y a 18 bonbons à la menthe, 30 bonbons à la fraise et 10 au chocolat. On choisit un bonbon au hasard dans ce sac.
|
||||
\begin{parts}
|
||||
\part Combien y a-t-il d'issues en tout?
|
||||
\begin{solution}
|
||||
Il y a 58 bonbons.
|
||||
\end{solution}
|
||||
\part Calculer la probabilité de tirer un bonbon à la fraise.
|
||||
\begin{solution}
|
||||
$T($ tirer un bonbon à la fraise $) = \dfrac{18}{58}$
|
||||
\end{solution}
|
||||
\part Calculer la probabilité de tirer un bonbon qui n'est pas au chocolat.
|
||||
\begin{solution}
|
||||
|
||||
$T($ tirer un bonbon à la fraise $) = \dfrac{48}{58}$
|
||||
\end{solution}
|
||||
\part Calculer la probabilité de tirer un bonbon au réglisse.
|
||||
\begin{solution}
|
||||
$T($ tirer un bonbon au réglisse $) = \dfrac{0}{58} = 0$
|
||||
\end{solution}
|
||||
\part Dans un autre sac, on place 25 bonbons à la menthe et 34 bonbons à la fraise. Lise préfère les bonbons à la menthe. Dans quel sac doit-elle tirer un bonbon pour avoir le plus de chance d'avoir un bonbon qu'elle préfère?
|
||||
\end{parts}
|
||||
|
||||
\vfill
|
||||
\end{questions}
|
||||
|
||||
\end{document}
|
||||
|
||||
%%% Local Variables:
|
||||
%%% mode: latex
|
||||
%%% TeX-master: "master"
|
||||
%%% End:
|
78
3e/DM/DM_16_09_15/34_DM_16_09_15.tex
Normal file
78
3e/DM/DM_16_09_15/34_DM_16_09_15.tex
Normal file
@@ -0,0 +1,78 @@
|
||||
\documentclass[a5paper,12pt, table]{/media/documents/Cours/Prof/Enseignements/2016-2017/tools/style/classDS}
|
||||
\usepackage{/media/documents/Cours/Prof/Enseignements/2016-2017/theme}
|
||||
\geometry{left=10mm,right=10mm, top=10mm, bottom=10mm}
|
||||
|
||||
% Title Page
|
||||
\titre{1}
|
||||
% \seconde \premiereS \PSTMG \TSTMG
|
||||
\classe{Troisième}
|
||||
\date{15 septembre 2016}
|
||||
%\duree{1 heure}
|
||||
\sujet{34}
|
||||
% DS DSCorr DM DMCorr Corr
|
||||
\typedoc{DM}
|
||||
|
||||
%\printanswers
|
||||
|
||||
\begin{document}
|
||||
|
||||
\maketitle
|
||||
|
||||
Vous devez rendre le sujet avec la copie.
|
||||
|
||||
\begin{questions}
|
||||
|
||||
\question
|
||||
Faire les calculs suivants en détaillant des étapes.
|
||||
\begin{multicols}{2}
|
||||
\begin{parts}
|
||||
|
||||
\part $9 - 7 \times ( -1 )$
|
||||
|
||||
\part $-2 - ( -1 \times 2 )$
|
||||
|
||||
\part $-4 \times 10 - 4 \times ( -5 )$
|
||||
|
||||
\part $( 7 + 8 ) \times ( -5 ) - 7$
|
||||
|
||||
\part $1 ( 6 + 8 ) \times ( -6 )$
|
||||
|
||||
\part $-6 ( 9 - 5 ) + 1$
|
||||
\end{parts}
|
||||
\end{multicols}
|
||||
\vfill
|
||||
|
||||
\question
|
||||
|
||||
|
||||
Dans un sac, il y a 18 bonbons à la menthe, 45 bonbons à la fraise et 7 au chocolat. On choisit un bonbon au hasard dans ce sac.
|
||||
\begin{parts}
|
||||
\part Combien y a-t-il d'issues en tout?
|
||||
\begin{solution}
|
||||
Il y a 70 bonbons.
|
||||
\end{solution}
|
||||
\part Calculer la probabilité de tirer un bonbon à la fraise.
|
||||
\begin{solution}
|
||||
$T($ tirer un bonbon à la fraise $) = \dfrac{18}{70}$
|
||||
\end{solution}
|
||||
\part Calculer la probabilité de tirer un bonbon qui n'est pas au chocolat.
|
||||
\begin{solution}
|
||||
|
||||
$T($ tirer un bonbon à la fraise $) = \dfrac{63}{70}$
|
||||
\end{solution}
|
||||
\part Calculer la probabilité de tirer un bonbon au réglisse.
|
||||
\begin{solution}
|
||||
$T($ tirer un bonbon au réglisse $) = \dfrac{0}{70} = 0$
|
||||
\end{solution}
|
||||
\part Dans un autre sac, on place 25 bonbons à la menthe et 34 bonbons à la fraise. Lise préfère les bonbons à la menthe. Dans quel sac doit-elle tirer un bonbon pour avoir le plus de chance d'avoir un bonbon qu'elle préfère?
|
||||
\end{parts}
|
||||
|
||||
\vfill
|
||||
\end{questions}
|
||||
|
||||
\end{document}
|
||||
|
||||
%%% Local Variables:
|
||||
%%% mode: latex
|
||||
%%% TeX-master: "master"
|
||||
%%% End:
|
78
3e/DM/DM_16_09_15/35_DM_16_09_15.tex
Normal file
78
3e/DM/DM_16_09_15/35_DM_16_09_15.tex
Normal file
@@ -0,0 +1,78 @@
|
||||
\documentclass[a5paper,12pt, table]{/media/documents/Cours/Prof/Enseignements/2016-2017/tools/style/classDS}
|
||||
\usepackage{/media/documents/Cours/Prof/Enseignements/2016-2017/theme}
|
||||
\geometry{left=10mm,right=10mm, top=10mm, bottom=10mm}
|
||||
|
||||
% Title Page
|
||||
\titre{1}
|
||||
% \seconde \premiereS \PSTMG \TSTMG
|
||||
\classe{Troisième}
|
||||
\date{15 septembre 2016}
|
||||
%\duree{1 heure}
|
||||
\sujet{35}
|
||||
% DS DSCorr DM DMCorr Corr
|
||||
\typedoc{DM}
|
||||
|
||||
%\printanswers
|
||||
|
||||
\begin{document}
|
||||
|
||||
\maketitle
|
||||
|
||||
Vous devez rendre le sujet avec la copie.
|
||||
|
||||
\begin{questions}
|
||||
|
||||
\question
|
||||
Faire les calculs suivants en détaillant des étapes.
|
||||
\begin{multicols}{2}
|
||||
\begin{parts}
|
||||
|
||||
\part $-7 - 3 \times ( -9 )$
|
||||
|
||||
\part $-5 - ( -7 \times ( -9 ) )$
|
||||
|
||||
\part $8 \times 9 - 8 \times 9$
|
||||
|
||||
\part $( 10 + 2 ) \times ( -9 ) + 9$
|
||||
|
||||
\part $-3 ( 7 + 5 ) \times ( -8 )$
|
||||
|
||||
\part $-4 ( 2 - 10 ) + 2$
|
||||
\end{parts}
|
||||
\end{multicols}
|
||||
\vfill
|
||||
|
||||
\question
|
||||
|
||||
|
||||
Dans un sac, il y a 16 bonbons à la menthe, 12 bonbons à la fraise et 9 au chocolat. On choisit un bonbon au hasard dans ce sac.
|
||||
\begin{parts}
|
||||
\part Combien y a-t-il d'issues en tout?
|
||||
\begin{solution}
|
||||
Il y a 37 bonbons.
|
||||
\end{solution}
|
||||
\part Calculer la probabilité de tirer un bonbon à la fraise.
|
||||
\begin{solution}
|
||||
$T($ tirer un bonbon à la fraise $) = \dfrac{16}{37}$
|
||||
\end{solution}
|
||||
\part Calculer la probabilité de tirer un bonbon qui n'est pas au chocolat.
|
||||
\begin{solution}
|
||||
|
||||
$T($ tirer un bonbon à la fraise $) = \dfrac{28}{37}$
|
||||
\end{solution}
|
||||
\part Calculer la probabilité de tirer un bonbon au réglisse.
|
||||
\begin{solution}
|
||||
$T($ tirer un bonbon au réglisse $) = \dfrac{0}{37} = 0$
|
||||
\end{solution}
|
||||
\part Dans un autre sac, on place 25 bonbons à la menthe et 34 bonbons à la fraise. Lise préfère les bonbons à la menthe. Dans quel sac doit-elle tirer un bonbon pour avoir le plus de chance d'avoir un bonbon qu'elle préfère?
|
||||
\end{parts}
|
||||
|
||||
\vfill
|
||||
\end{questions}
|
||||
|
||||
\end{document}
|
||||
|
||||
%%% Local Variables:
|
||||
%%% mode: latex
|
||||
%%% TeX-master: "master"
|
||||
%%% End:
|
78
3e/DM/DM_16_09_15/36_DM_16_09_15.tex
Normal file
78
3e/DM/DM_16_09_15/36_DM_16_09_15.tex
Normal file
@@ -0,0 +1,78 @@
|
||||
\documentclass[a5paper,12pt, table]{/media/documents/Cours/Prof/Enseignements/2016-2017/tools/style/classDS}
|
||||
\usepackage{/media/documents/Cours/Prof/Enseignements/2016-2017/theme}
|
||||
\geometry{left=10mm,right=10mm, top=10mm, bottom=10mm}
|
||||
|
||||
% Title Page
|
||||
\titre{1}
|
||||
% \seconde \premiereS \PSTMG \TSTMG
|
||||
\classe{Troisième}
|
||||
\date{15 septembre 2016}
|
||||
%\duree{1 heure}
|
||||
\sujet{36}
|
||||
% DS DSCorr DM DMCorr Corr
|
||||
\typedoc{DM}
|
||||
|
||||
%\printanswers
|
||||
|
||||
\begin{document}
|
||||
|
||||
\maketitle
|
||||
|
||||
Vous devez rendre le sujet avec la copie.
|
||||
|
||||
\begin{questions}
|
||||
|
||||
\question
|
||||
Faire les calculs suivants en détaillant des étapes.
|
||||
\begin{multicols}{2}
|
||||
\begin{parts}
|
||||
|
||||
\part $2 - 9 \times 2$
|
||||
|
||||
\part $4 - 1 \times 8$
|
||||
|
||||
\part $-7 \times ( -10 ) + 2 \times 8$
|
||||
|
||||
\part $( 1 + 6 ) \times ( -3 ) - 10$
|
||||
|
||||
\part $-9 ( 5 - 5 ) \times 5$
|
||||
|
||||
\part $8 ( 8 - 7 ) - 7$
|
||||
\end{parts}
|
||||
\end{multicols}
|
||||
\vfill
|
||||
|
||||
\question
|
||||
|
||||
|
||||
Dans un sac, il y a 30 bonbons à la menthe, 60 bonbons à la fraise et 10 au chocolat. On choisit un bonbon au hasard dans ce sac.
|
||||
\begin{parts}
|
||||
\part Combien y a-t-il d'issues en tout?
|
||||
\begin{solution}
|
||||
Il y a 100 bonbons.
|
||||
\end{solution}
|
||||
\part Calculer la probabilité de tirer un bonbon à la fraise.
|
||||
\begin{solution}
|
||||
$T($ tirer un bonbon à la fraise $) = \dfrac{30}{100}$
|
||||
\end{solution}
|
||||
\part Calculer la probabilité de tirer un bonbon qui n'est pas au chocolat.
|
||||
\begin{solution}
|
||||
|
||||
$T($ tirer un bonbon à la fraise $) = \dfrac{90}{100}$
|
||||
\end{solution}
|
||||
\part Calculer la probabilité de tirer un bonbon au réglisse.
|
||||
\begin{solution}
|
||||
$T($ tirer un bonbon au réglisse $) = \dfrac{0}{100} = 0$
|
||||
\end{solution}
|
||||
\part Dans un autre sac, on place 25 bonbons à la menthe et 34 bonbons à la fraise. Lise préfère les bonbons à la menthe. Dans quel sac doit-elle tirer un bonbon pour avoir le plus de chance d'avoir un bonbon qu'elle préfère?
|
||||
\end{parts}
|
||||
|
||||
\vfill
|
||||
\end{questions}
|
||||
|
||||
\end{document}
|
||||
|
||||
%%% Local Variables:
|
||||
%%% mode: latex
|
||||
%%% TeX-master: "master"
|
||||
%%% End:
|
78
3e/DM/DM_16_09_15/37_DM_16_09_15.tex
Normal file
78
3e/DM/DM_16_09_15/37_DM_16_09_15.tex
Normal file
@@ -0,0 +1,78 @@
|
||||
\documentclass[a5paper,12pt, table]{/media/documents/Cours/Prof/Enseignements/2016-2017/tools/style/classDS}
|
||||
\usepackage{/media/documents/Cours/Prof/Enseignements/2016-2017/theme}
|
||||
\geometry{left=10mm,right=10mm, top=10mm, bottom=10mm}
|
||||
|
||||
% Title Page
|
||||
\titre{1}
|
||||
% \seconde \premiereS \PSTMG \TSTMG
|
||||
\classe{Troisième}
|
||||
\date{15 septembre 2016}
|
||||
%\duree{1 heure}
|
||||
\sujet{37}
|
||||
% DS DSCorr DM DMCorr Corr
|
||||
\typedoc{DM}
|
||||
|
||||
%\printanswers
|
||||
|
||||
\begin{document}
|
||||
|
||||
\maketitle
|
||||
|
||||
Vous devez rendre le sujet avec la copie.
|
||||
|
||||
\begin{questions}
|
||||
|
||||
\question
|
||||
Faire les calculs suivants en détaillant des étapes.
|
||||
\begin{multicols}{2}
|
||||
\begin{parts}
|
||||
|
||||
\part $1 - 8 \times ( -7 )$
|
||||
|
||||
\part $5 - 3 \times ( -10 )$
|
||||
|
||||
\part $2 \times 9 - 4 \times 8$
|
||||
|
||||
\part $( -4 - 3 ) \times ( -5 ) + 9$
|
||||
|
||||
\part $2 ( 5 - 3 ) \times 9$
|
||||
|
||||
\part $-5 ( 4 - 1 ) - 8$
|
||||
\end{parts}
|
||||
\end{multicols}
|
||||
\vfill
|
||||
|
||||
\question
|
||||
|
||||
|
||||
Dans un sac, il y a 18 bonbons à la menthe, 10 bonbons à la fraise et 9 au chocolat. On choisit un bonbon au hasard dans ce sac.
|
||||
\begin{parts}
|
||||
\part Combien y a-t-il d'issues en tout?
|
||||
\begin{solution}
|
||||
Il y a 37 bonbons.
|
||||
\end{solution}
|
||||
\part Calculer la probabilité de tirer un bonbon à la fraise.
|
||||
\begin{solution}
|
||||
$T($ tirer un bonbon à la fraise $) = \dfrac{18}{37}$
|
||||
\end{solution}
|
||||
\part Calculer la probabilité de tirer un bonbon qui n'est pas au chocolat.
|
||||
\begin{solution}
|
||||
|
||||
$T($ tirer un bonbon à la fraise $) = \dfrac{28}{37}$
|
||||
\end{solution}
|
||||
\part Calculer la probabilité de tirer un bonbon au réglisse.
|
||||
\begin{solution}
|
||||
$T($ tirer un bonbon au réglisse $) = \dfrac{0}{37} = 0$
|
||||
\end{solution}
|
||||
\part Dans un autre sac, on place 25 bonbons à la menthe et 34 bonbons à la fraise. Lise préfère les bonbons à la menthe. Dans quel sac doit-elle tirer un bonbon pour avoir le plus de chance d'avoir un bonbon qu'elle préfère?
|
||||
\end{parts}
|
||||
|
||||
\vfill
|
||||
\end{questions}
|
||||
|
||||
\end{document}
|
||||
|
||||
%%% Local Variables:
|
||||
%%% mode: latex
|
||||
%%% TeX-master: "master"
|
||||
%%% End:
|
78
3e/DM/DM_16_09_15/38_DM_16_09_15.tex
Normal file
78
3e/DM/DM_16_09_15/38_DM_16_09_15.tex
Normal file
@@ -0,0 +1,78 @@
|
||||
\documentclass[a5paper,12pt, table]{/media/documents/Cours/Prof/Enseignements/2016-2017/tools/style/classDS}
|
||||
\usepackage{/media/documents/Cours/Prof/Enseignements/2016-2017/theme}
|
||||
\geometry{left=10mm,right=10mm, top=10mm, bottom=10mm}
|
||||
|
||||
% Title Page
|
||||
\titre{1}
|
||||
% \seconde \premiereS \PSTMG \TSTMG
|
||||
\classe{Troisième}
|
||||
\date{15 septembre 2016}
|
||||
%\duree{1 heure}
|
||||
\sujet{38}
|
||||
% DS DSCorr DM DMCorr Corr
|
||||
\typedoc{DM}
|
||||
|
||||
%\printanswers
|
||||
|
||||
\begin{document}
|
||||
|
||||
\maketitle
|
||||
|
||||
Vous devez rendre le sujet avec la copie.
|
||||
|
||||
\begin{questions}
|
||||
|
||||
\question
|
||||
Faire les calculs suivants en détaillant des étapes.
|
||||
\begin{multicols}{2}
|
||||
\begin{parts}
|
||||
|
||||
\part $-6 + 4 \times ( -5 )$
|
||||
|
||||
\part $-5 - ( -8 \times ( -5 ) )$
|
||||
|
||||
\part $8 \times ( -2 ) - 1 \times 2$
|
||||
|
||||
\part $( 3 - 3 ) \times 7 + 5$
|
||||
|
||||
\part $6 ( -5 - 9 ) \times ( -3 )$
|
||||
|
||||
\part $-8 ( -2 - 6 ) + 7$
|
||||
\end{parts}
|
||||
\end{multicols}
|
||||
\vfill
|
||||
|
||||
\question
|
||||
|
||||
|
||||
Dans un sac, il y a 35 bonbons à la menthe, 30 bonbons à la fraise et 3 au chocolat. On choisit un bonbon au hasard dans ce sac.
|
||||
\begin{parts}
|
||||
\part Combien y a-t-il d'issues en tout?
|
||||
\begin{solution}
|
||||
Il y a 68 bonbons.
|
||||
\end{solution}
|
||||
\part Calculer la probabilité de tirer un bonbon à la fraise.
|
||||
\begin{solution}
|
||||
$T($ tirer un bonbon à la fraise $) = \dfrac{35}{68}$
|
||||
\end{solution}
|
||||
\part Calculer la probabilité de tirer un bonbon qui n'est pas au chocolat.
|
||||
\begin{solution}
|
||||
|
||||
$T($ tirer un bonbon à la fraise $) = \dfrac{65}{68}$
|
||||
\end{solution}
|
||||
\part Calculer la probabilité de tirer un bonbon au réglisse.
|
||||
\begin{solution}
|
||||
$T($ tirer un bonbon au réglisse $) = \dfrac{0}{68} = 0$
|
||||
\end{solution}
|
||||
\part Dans un autre sac, on place 25 bonbons à la menthe et 34 bonbons à la fraise. Lise préfère les bonbons à la menthe. Dans quel sac doit-elle tirer un bonbon pour avoir le plus de chance d'avoir un bonbon qu'elle préfère?
|
||||
\end{parts}
|
||||
|
||||
\vfill
|
||||
\end{questions}
|
||||
|
||||
\end{document}
|
||||
|
||||
%%% Local Variables:
|
||||
%%% mode: latex
|
||||
%%% TeX-master: "master"
|
||||
%%% End:
|
78
3e/DM/DM_16_09_15/39_DM_16_09_15.tex
Normal file
78
3e/DM/DM_16_09_15/39_DM_16_09_15.tex
Normal file
@@ -0,0 +1,78 @@
|
||||
\documentclass[a5paper,12pt, table]{/media/documents/Cours/Prof/Enseignements/2016-2017/tools/style/classDS}
|
||||
\usepackage{/media/documents/Cours/Prof/Enseignements/2016-2017/theme}
|
||||
\geometry{left=10mm,right=10mm, top=10mm, bottom=10mm}
|
||||
|
||||
% Title Page
|
||||
\titre{1}
|
||||
% \seconde \premiereS \PSTMG \TSTMG
|
||||
\classe{Troisième}
|
||||
\date{15 septembre 2016}
|
||||
%\duree{1 heure}
|
||||
\sujet{39}
|
||||
% DS DSCorr DM DMCorr Corr
|
||||
\typedoc{DM}
|
||||
|
||||
%\printanswers
|
||||
|
||||
\begin{document}
|
||||
|
||||
\maketitle
|
||||
|
||||
Vous devez rendre le sujet avec la copie.
|
||||
|
||||
\begin{questions}
|
||||
|
||||
\question
|
||||
Faire les calculs suivants en détaillant des étapes.
|
||||
\begin{multicols}{2}
|
||||
\begin{parts}
|
||||
|
||||
\part $-3 + 9 \times 4$
|
||||
|
||||
\part $6 - 9 \times 2$
|
||||
|
||||
\part $-9 \times ( -1 ) + 1 \times ( -2 )$
|
||||
|
||||
\part $( 7 + 5 ) \times ( -3 ) + 2$
|
||||
|
||||
\part $7 ( -8 - 4 ) \times 4$
|
||||
|
||||
\part $6 ( -2 - ( -8 ) ) - 3$
|
||||
\end{parts}
|
||||
\end{multicols}
|
||||
\vfill
|
||||
|
||||
\question
|
||||
|
||||
|
||||
Dans un sac, il y a 64 bonbons à la menthe, 40 bonbons à la fraise et 3 au chocolat. On choisit un bonbon au hasard dans ce sac.
|
||||
\begin{parts}
|
||||
\part Combien y a-t-il d'issues en tout?
|
||||
\begin{solution}
|
||||
Il y a 107 bonbons.
|
||||
\end{solution}
|
||||
\part Calculer la probabilité de tirer un bonbon à la fraise.
|
||||
\begin{solution}
|
||||
$T($ tirer un bonbon à la fraise $) = \dfrac{64}{107}$
|
||||
\end{solution}
|
||||
\part Calculer la probabilité de tirer un bonbon qui n'est pas au chocolat.
|
||||
\begin{solution}
|
||||
|
||||
$T($ tirer un bonbon à la fraise $) = \dfrac{104}{107}$
|
||||
\end{solution}
|
||||
\part Calculer la probabilité de tirer un bonbon au réglisse.
|
||||
\begin{solution}
|
||||
$T($ tirer un bonbon au réglisse $) = \dfrac{0}{107} = 0$
|
||||
\end{solution}
|
||||
\part Dans un autre sac, on place 25 bonbons à la menthe et 34 bonbons à la fraise. Lise préfère les bonbons à la menthe. Dans quel sac doit-elle tirer un bonbon pour avoir le plus de chance d'avoir un bonbon qu'elle préfère?
|
||||
\end{parts}
|
||||
|
||||
\vfill
|
||||
\end{questions}
|
||||
|
||||
\end{document}
|
||||
|
||||
%%% Local Variables:
|
||||
%%% mode: latex
|
||||
%%% TeX-master: "master"
|
||||
%%% End:
|
78
3e/DM/DM_16_09_15/40_DM_16_09_15.tex
Normal file
78
3e/DM/DM_16_09_15/40_DM_16_09_15.tex
Normal file
@@ -0,0 +1,78 @@
|
||||
\documentclass[a5paper,12pt, table]{/media/documents/Cours/Prof/Enseignements/2016-2017/tools/style/classDS}
|
||||
\usepackage{/media/documents/Cours/Prof/Enseignements/2016-2017/theme}
|
||||
\geometry{left=10mm,right=10mm, top=10mm, bottom=10mm}
|
||||
|
||||
% Title Page
|
||||
\titre{1}
|
||||
% \seconde \premiereS \PSTMG \TSTMG
|
||||
\classe{Troisième}
|
||||
\date{15 septembre 2016}
|
||||
%\duree{1 heure}
|
||||
\sujet{40}
|
||||
% DS DSCorr DM DMCorr Corr
|
||||
\typedoc{DM}
|
||||
|
||||
%\printanswers
|
||||
|
||||
\begin{document}
|
||||
|
||||
\maketitle
|
||||
|
||||
Vous devez rendre le sujet avec la copie.
|
||||
|
||||
\begin{questions}
|
||||
|
||||
\question
|
||||
Faire les calculs suivants en détaillant des étapes.
|
||||
\begin{multicols}{2}
|
||||
\begin{parts}
|
||||
|
||||
\part $-8 - 9 \times 1$
|
||||
|
||||
\part $2 - ( -5 \times 2 )$
|
||||
|
||||
\part $-4 \times ( -4 ) + 5 \times ( -10 )$
|
||||
|
||||
\part $( 5 + 7 ) \times ( -6 ) + 4$
|
||||
|
||||
\part $1 ( -2 + 5 ) \times ( -3 )$
|
||||
|
||||
\part $5 ( -4 - 1 ) - 6$
|
||||
\end{parts}
|
||||
\end{multicols}
|
||||
\vfill
|
||||
|
||||
\question
|
||||
|
||||
|
||||
Dans un sac, il y a 70 bonbons à la menthe, 40 bonbons à la fraise et 10 au chocolat. On choisit un bonbon au hasard dans ce sac.
|
||||
\begin{parts}
|
||||
\part Combien y a-t-il d'issues en tout?
|
||||
\begin{solution}
|
||||
Il y a 120 bonbons.
|
||||
\end{solution}
|
||||
\part Calculer la probabilité de tirer un bonbon à la fraise.
|
||||
\begin{solution}
|
||||
$T($ tirer un bonbon à la fraise $) = \dfrac{70}{120}$
|
||||
\end{solution}
|
||||
\part Calculer la probabilité de tirer un bonbon qui n'est pas au chocolat.
|
||||
\begin{solution}
|
||||
|
||||
$T($ tirer un bonbon à la fraise $) = \dfrac{110}{120}$
|
||||
\end{solution}
|
||||
\part Calculer la probabilité de tirer un bonbon au réglisse.
|
||||
\begin{solution}
|
||||
$T($ tirer un bonbon au réglisse $) = \dfrac{0}{120} = 0$
|
||||
\end{solution}
|
||||
\part Dans un autre sac, on place 25 bonbons à la menthe et 34 bonbons à la fraise. Lise préfère les bonbons à la menthe. Dans quel sac doit-elle tirer un bonbon pour avoir le plus de chance d'avoir un bonbon qu'elle préfère?
|
||||
\end{parts}
|
||||
|
||||
\vfill
|
||||
\end{questions}
|
||||
|
||||
\end{document}
|
||||
|
||||
%%% Local Variables:
|
||||
%%% mode: latex
|
||||
%%% TeX-master: "master"
|
||||
%%% End:
|
78
3e/DM/DM_16_09_15/41_DM_16_09_15.tex
Normal file
78
3e/DM/DM_16_09_15/41_DM_16_09_15.tex
Normal file
@@ -0,0 +1,78 @@
|
||||
\documentclass[a5paper,12pt, table]{/media/documents/Cours/Prof/Enseignements/2016-2017/tools/style/classDS}
|
||||
\usepackage{/media/documents/Cours/Prof/Enseignements/2016-2017/theme}
|
||||
\geometry{left=10mm,right=10mm, top=10mm, bottom=10mm}
|
||||
|
||||
% Title Page
|
||||
\titre{1}
|
||||
% \seconde \premiereS \PSTMG \TSTMG
|
||||
\classe{Troisième}
|
||||
\date{15 septembre 2016}
|
||||
%\duree{1 heure}
|
||||
\sujet{41}
|
||||
% DS DSCorr DM DMCorr Corr
|
||||
\typedoc{DM}
|
||||
|
||||
%\printanswers
|
||||
|
||||
\begin{document}
|
||||
|
||||
\maketitle
|
||||
|
||||
Vous devez rendre le sujet avec la copie.
|
||||
|
||||
\begin{questions}
|
||||
|
||||
\question
|
||||
Faire les calculs suivants en détaillant des étapes.
|
||||
\begin{multicols}{2}
|
||||
\begin{parts}
|
||||
|
||||
\part $9 + 2 \times 5$
|
||||
|
||||
\part $9 - ( -2 \times ( -5 ) )$
|
||||
|
||||
\part $-4 \times ( -1 ) - 3 \times ( -8 )$
|
||||
|
||||
\part $( 5 - 5 ) \times ( -1 ) - 8$
|
||||
|
||||
\part $1 ( 7 + 6 ) \times 9$
|
||||
|
||||
\part $3 ( -4 - ( -4 ) ) - 4$
|
||||
\end{parts}
|
||||
\end{multicols}
|
||||
\vfill
|
||||
|
||||
\question
|
||||
|
||||
|
||||
Dans un sac, il y a 40 bonbons à la menthe, 20 bonbons à la fraise et 8 au chocolat. On choisit un bonbon au hasard dans ce sac.
|
||||
\begin{parts}
|
||||
\part Combien y a-t-il d'issues en tout?
|
||||
\begin{solution}
|
||||
Il y a 68 bonbons.
|
||||
\end{solution}
|
||||
\part Calculer la probabilité de tirer un bonbon à la fraise.
|
||||
\begin{solution}
|
||||
$T($ tirer un bonbon à la fraise $) = \dfrac{40}{68}$
|
||||
\end{solution}
|
||||
\part Calculer la probabilité de tirer un bonbon qui n'est pas au chocolat.
|
||||
\begin{solution}
|
||||
|
||||
$T($ tirer un bonbon à la fraise $) = \dfrac{60}{68}$
|
||||
\end{solution}
|
||||
\part Calculer la probabilité de tirer un bonbon au réglisse.
|
||||
\begin{solution}
|
||||
$T($ tirer un bonbon au réglisse $) = \dfrac{0}{68} = 0$
|
||||
\end{solution}
|
||||
\part Dans un autre sac, on place 25 bonbons à la menthe et 34 bonbons à la fraise. Lise préfère les bonbons à la menthe. Dans quel sac doit-elle tirer un bonbon pour avoir le plus de chance d'avoir un bonbon qu'elle préfère?
|
||||
\end{parts}
|
||||
|
||||
\vfill
|
||||
\end{questions}
|
||||
|
||||
\end{document}
|
||||
|
||||
%%% Local Variables:
|
||||
%%% mode: latex
|
||||
%%% TeX-master: "master"
|
||||
%%% End:
|
78
3e/DM/DM_16_09_15/42_DM_16_09_15.tex
Normal file
78
3e/DM/DM_16_09_15/42_DM_16_09_15.tex
Normal file
@@ -0,0 +1,78 @@
|
||||
\documentclass[a5paper,12pt, table]{/media/documents/Cours/Prof/Enseignements/2016-2017/tools/style/classDS}
|
||||
\usepackage{/media/documents/Cours/Prof/Enseignements/2016-2017/theme}
|
||||
\geometry{left=10mm,right=10mm, top=10mm, bottom=10mm}
|
||||
|
||||
% Title Page
|
||||
\titre{1}
|
||||
% \seconde \premiereS \PSTMG \TSTMG
|
||||
\classe{Troisième}
|
||||
\date{15 septembre 2016}
|
||||
%\duree{1 heure}
|
||||
\sujet{42}
|
||||
% DS DSCorr DM DMCorr Corr
|
||||
\typedoc{DM}
|
||||
|
||||
%\printanswers
|
||||
|
||||
\begin{document}
|
||||
|
||||
\maketitle
|
||||
|
||||
Vous devez rendre le sujet avec la copie.
|
||||
|
||||
\begin{questions}
|
||||
|
||||
\question
|
||||
Faire les calculs suivants en détaillant des étapes.
|
||||
\begin{multicols}{2}
|
||||
\begin{parts}
|
||||
|
||||
\part $5 - 2 \times ( -3 )$
|
||||
|
||||
\part $-2 - ( -4 \times 8 )$
|
||||
|
||||
\part $-5 \times 7 + 4 \times ( -8 )$
|
||||
|
||||
\part $( -8 + 4 ) \times ( -6 ) - 10$
|
||||
|
||||
\part $5 ( -1 - 5 ) \times ( -7 )$
|
||||
|
||||
\part $-2 ( -1 - 4 ) - 6$
|
||||
\end{parts}
|
||||
\end{multicols}
|
||||
\vfill
|
||||
|
||||
\question
|
||||
|
||||
|
||||
Dans un sac, il y a 90 bonbons à la menthe, 18 bonbons à la fraise et 7 au chocolat. On choisit un bonbon au hasard dans ce sac.
|
||||
\begin{parts}
|
||||
\part Combien y a-t-il d'issues en tout?
|
||||
\begin{solution}
|
||||
Il y a 115 bonbons.
|
||||
\end{solution}
|
||||
\part Calculer la probabilité de tirer un bonbon à la fraise.
|
||||
\begin{solution}
|
||||
$T($ tirer un bonbon à la fraise $) = \dfrac{90}{115}$
|
||||
\end{solution}
|
||||
\part Calculer la probabilité de tirer un bonbon qui n'est pas au chocolat.
|
||||
\begin{solution}
|
||||
|
||||
$T($ tirer un bonbon à la fraise $) = \dfrac{108}{115}$
|
||||
\end{solution}
|
||||
\part Calculer la probabilité de tirer un bonbon au réglisse.
|
||||
\begin{solution}
|
||||
$T($ tirer un bonbon au réglisse $) = \dfrac{0}{115} = 0$
|
||||
\end{solution}
|
||||
\part Dans un autre sac, on place 25 bonbons à la menthe et 34 bonbons à la fraise. Lise préfère les bonbons à la menthe. Dans quel sac doit-elle tirer un bonbon pour avoir le plus de chance d'avoir un bonbon qu'elle préfère?
|
||||
\end{parts}
|
||||
|
||||
\vfill
|
||||
\end{questions}
|
||||
|
||||
\end{document}
|
||||
|
||||
%%% Local Variables:
|
||||
%%% mode: latex
|
||||
%%% TeX-master: "master"
|
||||
%%% End:
|
78
3e/DM/DM_16_09_15/43_DM_16_09_15.tex
Normal file
78
3e/DM/DM_16_09_15/43_DM_16_09_15.tex
Normal file
@@ -0,0 +1,78 @@
|
||||
\documentclass[a5paper,12pt, table]{/media/documents/Cours/Prof/Enseignements/2016-2017/tools/style/classDS}
|
||||
\usepackage{/media/documents/Cours/Prof/Enseignements/2016-2017/theme}
|
||||
\geometry{left=10mm,right=10mm, top=10mm, bottom=10mm}
|
||||
|
||||
% Title Page
|
||||
\titre{1}
|
||||
% \seconde \premiereS \PSTMG \TSTMG
|
||||
\classe{Troisième}
|
||||
\date{15 septembre 2016}
|
||||
%\duree{1 heure}
|
||||
\sujet{43}
|
||||
% DS DSCorr DM DMCorr Corr
|
||||
\typedoc{DM}
|
||||
|
||||
%\printanswers
|
||||
|
||||
\begin{document}
|
||||
|
||||
\maketitle
|
||||
|
||||
Vous devez rendre le sujet avec la copie.
|
||||
|
||||
\begin{questions}
|
||||
|
||||
\question
|
||||
Faire les calculs suivants en détaillant des étapes.
|
||||
\begin{multicols}{2}
|
||||
\begin{parts}
|
||||
|
||||
\part $8 + 2 \times ( -6 )$
|
||||
|
||||
\part $7 - 9 \times ( -9 )$
|
||||
|
||||
\part $5 \times 8 + 2 \times 10$
|
||||
|
||||
\part $( 6 - 7 ) \times ( -4 ) - 5$
|
||||
|
||||
\part $9 ( -2 - 6 ) \times ( -7 )$
|
||||
|
||||
\part $4 ( 1 - ( -10 ) ) - 4$
|
||||
\end{parts}
|
||||
\end{multicols}
|
||||
\vfill
|
||||
|
||||
\question
|
||||
|
||||
|
||||
Dans un sac, il y a 40 bonbons à la menthe, 56 bonbons à la fraise et 2 au chocolat. On choisit un bonbon au hasard dans ce sac.
|
||||
\begin{parts}
|
||||
\part Combien y a-t-il d'issues en tout?
|
||||
\begin{solution}
|
||||
Il y a 98 bonbons.
|
||||
\end{solution}
|
||||
\part Calculer la probabilité de tirer un bonbon à la fraise.
|
||||
\begin{solution}
|
||||
$T($ tirer un bonbon à la fraise $) = \dfrac{40}{98}$
|
||||
\end{solution}
|
||||
\part Calculer la probabilité de tirer un bonbon qui n'est pas au chocolat.
|
||||
\begin{solution}
|
||||
|
||||
$T($ tirer un bonbon à la fraise $) = \dfrac{96}{98}$
|
||||
\end{solution}
|
||||
\part Calculer la probabilité de tirer un bonbon au réglisse.
|
||||
\begin{solution}
|
||||
$T($ tirer un bonbon au réglisse $) = \dfrac{0}{98} = 0$
|
||||
\end{solution}
|
||||
\part Dans un autre sac, on place 25 bonbons à la menthe et 34 bonbons à la fraise. Lise préfère les bonbons à la menthe. Dans quel sac doit-elle tirer un bonbon pour avoir le plus de chance d'avoir un bonbon qu'elle préfère?
|
||||
\end{parts}
|
||||
|
||||
\vfill
|
||||
\end{questions}
|
||||
|
||||
\end{document}
|
||||
|
||||
%%% Local Variables:
|
||||
%%% mode: latex
|
||||
%%% TeX-master: "master"
|
||||
%%% End:
|
78
3e/DM/DM_16_09_15/44_DM_16_09_15.tex
Normal file
78
3e/DM/DM_16_09_15/44_DM_16_09_15.tex
Normal file
@@ -0,0 +1,78 @@
|
||||
\documentclass[a5paper,12pt, table]{/media/documents/Cours/Prof/Enseignements/2016-2017/tools/style/classDS}
|
||||
\usepackage{/media/documents/Cours/Prof/Enseignements/2016-2017/theme}
|
||||
\geometry{left=10mm,right=10mm, top=10mm, bottom=10mm}
|
||||
|
||||
% Title Page
|
||||
\titre{1}
|
||||
% \seconde \premiereS \PSTMG \TSTMG
|
||||
\classe{Troisième}
|
||||
\date{15 septembre 2016}
|
||||
%\duree{1 heure}
|
||||
\sujet{44}
|
||||
% DS DSCorr DM DMCorr Corr
|
||||
\typedoc{DM}
|
||||
|
||||
%\printanswers
|
||||
|
||||
\begin{document}
|
||||
|
||||
\maketitle
|
||||
|
||||
Vous devez rendre le sujet avec la copie.
|
||||
|
||||
\begin{questions}
|
||||
|
||||
\question
|
||||
Faire les calculs suivants en détaillant des étapes.
|
||||
\begin{multicols}{2}
|
||||
\begin{parts}
|
||||
|
||||
\part $7 + 9 \times 8$
|
||||
|
||||
\part $4 - 7 \times 10$
|
||||
|
||||
\part $-1 \times 6 - 1 \times ( -4 )$
|
||||
|
||||
\part $( 4 - 1 ) \times ( -6 ) + 4$
|
||||
|
||||
\part $-7 ( 2 - 3 ) \times 8$
|
||||
|
||||
\part $7 ( -2 - ( -6 ) ) + 9$
|
||||
\end{parts}
|
||||
\end{multicols}
|
||||
\vfill
|
||||
|
||||
\question
|
||||
|
||||
|
||||
Dans un sac, il y a 20 bonbons à la menthe, 10 bonbons à la fraise et 7 au chocolat. On choisit un bonbon au hasard dans ce sac.
|
||||
\begin{parts}
|
||||
\part Combien y a-t-il d'issues en tout?
|
||||
\begin{solution}
|
||||
Il y a 37 bonbons.
|
||||
\end{solution}
|
||||
\part Calculer la probabilité de tirer un bonbon à la fraise.
|
||||
\begin{solution}
|
||||
$T($ tirer un bonbon à la fraise $) = \dfrac{20}{37}$
|
||||
\end{solution}
|
||||
\part Calculer la probabilité de tirer un bonbon qui n'est pas au chocolat.
|
||||
\begin{solution}
|
||||
|
||||
$T($ tirer un bonbon à la fraise $) = \dfrac{30}{37}$
|
||||
\end{solution}
|
||||
\part Calculer la probabilité de tirer un bonbon au réglisse.
|
||||
\begin{solution}
|
||||
$T($ tirer un bonbon au réglisse $) = \dfrac{0}{37} = 0$
|
||||
\end{solution}
|
||||
\part Dans un autre sac, on place 25 bonbons à la menthe et 34 bonbons à la fraise. Lise préfère les bonbons à la menthe. Dans quel sac doit-elle tirer un bonbon pour avoir le plus de chance d'avoir un bonbon qu'elle préfère?
|
||||
\end{parts}
|
||||
|
||||
\vfill
|
||||
\end{questions}
|
||||
|
||||
\end{document}
|
||||
|
||||
%%% Local Variables:
|
||||
%%% mode: latex
|
||||
%%% TeX-master: "master"
|
||||
%%% End:
|
78
3e/DM/DM_16_09_15/45_DM_16_09_15.tex
Normal file
78
3e/DM/DM_16_09_15/45_DM_16_09_15.tex
Normal file
@@ -0,0 +1,78 @@
|
||||
\documentclass[a5paper,12pt, table]{/media/documents/Cours/Prof/Enseignements/2016-2017/tools/style/classDS}
|
||||
\usepackage{/media/documents/Cours/Prof/Enseignements/2016-2017/theme}
|
||||
\geometry{left=10mm,right=10mm, top=10mm, bottom=10mm}
|
||||
|
||||
% Title Page
|
||||
\titre{1}
|
||||
% \seconde \premiereS \PSTMG \TSTMG
|
||||
\classe{Troisième}
|
||||
\date{15 septembre 2016}
|
||||
%\duree{1 heure}
|
||||
\sujet{45}
|
||||
% DS DSCorr DM DMCorr Corr
|
||||
\typedoc{DM}
|
||||
|
||||
%\printanswers
|
||||
|
||||
\begin{document}
|
||||
|
||||
\maketitle
|
||||
|
||||
Vous devez rendre le sujet avec la copie.
|
||||
|
||||
\begin{questions}
|
||||
|
||||
\question
|
||||
Faire les calculs suivants en détaillant des étapes.
|
||||
\begin{multicols}{2}
|
||||
\begin{parts}
|
||||
|
||||
\part $-5 - 2 \times 4$
|
||||
|
||||
\part $-3 - 10 \times ( -8 )$
|
||||
|
||||
\part $4 \times ( -4 ) - 6 \times ( -10 )$
|
||||
|
||||
\part $( -8 + 2 ) \times ( -6 ) - 4$
|
||||
|
||||
\part $4 ( 1 - 6 ) \times ( -1 )$
|
||||
|
||||
\part $-8 ( 5 - ( -8 ) ) - 9$
|
||||
\end{parts}
|
||||
\end{multicols}
|
||||
\vfill
|
||||
|
||||
\question
|
||||
|
||||
|
||||
Dans un sac, il y a 15 bonbons à la menthe, 6 bonbons à la fraise et 2 au chocolat. On choisit un bonbon au hasard dans ce sac.
|
||||
\begin{parts}
|
||||
\part Combien y a-t-il d'issues en tout?
|
||||
\begin{solution}
|
||||
Il y a 23 bonbons.
|
||||
\end{solution}
|
||||
\part Calculer la probabilité de tirer un bonbon à la fraise.
|
||||
\begin{solution}
|
||||
$T($ tirer un bonbon à la fraise $) = \dfrac{15}{23}$
|
||||
\end{solution}
|
||||
\part Calculer la probabilité de tirer un bonbon qui n'est pas au chocolat.
|
||||
\begin{solution}
|
||||
|
||||
$T($ tirer un bonbon à la fraise $) = \dfrac{21}{23}$
|
||||
\end{solution}
|
||||
\part Calculer la probabilité de tirer un bonbon au réglisse.
|
||||
\begin{solution}
|
||||
$T($ tirer un bonbon au réglisse $) = \dfrac{0}{23} = 0$
|
||||
\end{solution}
|
||||
\part Dans un autre sac, on place 25 bonbons à la menthe et 34 bonbons à la fraise. Lise préfère les bonbons à la menthe. Dans quel sac doit-elle tirer un bonbon pour avoir le plus de chance d'avoir un bonbon qu'elle préfère?
|
||||
\end{parts}
|
||||
|
||||
\vfill
|
||||
\end{questions}
|
||||
|
||||
\end{document}
|
||||
|
||||
%%% Local Variables:
|
||||
%%% mode: latex
|
||||
%%% TeX-master: "master"
|
||||
%%% End:
|
78
3e/DM/DM_16_09_15/46_DM_16_09_15.tex
Normal file
78
3e/DM/DM_16_09_15/46_DM_16_09_15.tex
Normal file
@@ -0,0 +1,78 @@
|
||||
\documentclass[a5paper,12pt, table]{/media/documents/Cours/Prof/Enseignements/2016-2017/tools/style/classDS}
|
||||
\usepackage{/media/documents/Cours/Prof/Enseignements/2016-2017/theme}
|
||||
\geometry{left=10mm,right=10mm, top=10mm, bottom=10mm}
|
||||
|
||||
% Title Page
|
||||
\titre{1}
|
||||
% \seconde \premiereS \PSTMG \TSTMG
|
||||
\classe{Troisième}
|
||||
\date{15 septembre 2016}
|
||||
%\duree{1 heure}
|
||||
\sujet{46}
|
||||
% DS DSCorr DM DMCorr Corr
|
||||
\typedoc{DM}
|
||||
|
||||
%\printanswers
|
||||
|
||||
\begin{document}
|
||||
|
||||
\maketitle
|
||||
|
||||
Vous devez rendre le sujet avec la copie.
|
||||
|
||||
\begin{questions}
|
||||
|
||||
\question
|
||||
Faire les calculs suivants en détaillant des étapes.
|
||||
\begin{multicols}{2}
|
||||
\begin{parts}
|
||||
|
||||
\part $-4 + 6 \times 6$
|
||||
|
||||
\part $-4 - ( -10 \times 9 )$
|
||||
|
||||
\part $3 \times 8 - 3 \times ( -3 )$
|
||||
|
||||
\part $( -7 - 6 ) \times 8 - 1$
|
||||
|
||||
\part $8 ( -5 - 4 ) \times 6$
|
||||
|
||||
\part $9 ( -8 - 5 ) - 4$
|
||||
\end{parts}
|
||||
\end{multicols}
|
||||
\vfill
|
||||
|
||||
\question
|
||||
|
||||
|
||||
Dans un sac, il y a 21 bonbons à la menthe, 28 bonbons à la fraise et 10 au chocolat. On choisit un bonbon au hasard dans ce sac.
|
||||
\begin{parts}
|
||||
\part Combien y a-t-il d'issues en tout?
|
||||
\begin{solution}
|
||||
Il y a 59 bonbons.
|
||||
\end{solution}
|
||||
\part Calculer la probabilité de tirer un bonbon à la fraise.
|
||||
\begin{solution}
|
||||
$T($ tirer un bonbon à la fraise $) = \dfrac{21}{59}$
|
||||
\end{solution}
|
||||
\part Calculer la probabilité de tirer un bonbon qui n'est pas au chocolat.
|
||||
\begin{solution}
|
||||
|
||||
$T($ tirer un bonbon à la fraise $) = \dfrac{49}{59}$
|
||||
\end{solution}
|
||||
\part Calculer la probabilité de tirer un bonbon au réglisse.
|
||||
\begin{solution}
|
||||
$T($ tirer un bonbon au réglisse $) = \dfrac{0}{59} = 0$
|
||||
\end{solution}
|
||||
\part Dans un autre sac, on place 25 bonbons à la menthe et 34 bonbons à la fraise. Lise préfère les bonbons à la menthe. Dans quel sac doit-elle tirer un bonbon pour avoir le plus de chance d'avoir un bonbon qu'elle préfère?
|
||||
\end{parts}
|
||||
|
||||
\vfill
|
||||
\end{questions}
|
||||
|
||||
\end{document}
|
||||
|
||||
%%% Local Variables:
|
||||
%%% mode: latex
|
||||
%%% TeX-master: "master"
|
||||
%%% End:
|
78
3e/DM/DM_16_09_15/47_DM_16_09_15.tex
Normal file
78
3e/DM/DM_16_09_15/47_DM_16_09_15.tex
Normal file
@@ -0,0 +1,78 @@
|
||||
\documentclass[a5paper,12pt, table]{/media/documents/Cours/Prof/Enseignements/2016-2017/tools/style/classDS}
|
||||
\usepackage{/media/documents/Cours/Prof/Enseignements/2016-2017/theme}
|
||||
\geometry{left=10mm,right=10mm, top=10mm, bottom=10mm}
|
||||
|
||||
% Title Page
|
||||
\titre{1}
|
||||
% \seconde \premiereS \PSTMG \TSTMG
|
||||
\classe{Troisième}
|
||||
\date{15 septembre 2016}
|
||||
%\duree{1 heure}
|
||||
\sujet{47}
|
||||
% DS DSCorr DM DMCorr Corr
|
||||
\typedoc{DM}
|
||||
|
||||
%\printanswers
|
||||
|
||||
\begin{document}
|
||||
|
||||
\maketitle
|
||||
|
||||
Vous devez rendre le sujet avec la copie.
|
||||
|
||||
\begin{questions}
|
||||
|
||||
\question
|
||||
Faire les calculs suivants en détaillant des étapes.
|
||||
\begin{multicols}{2}
|
||||
\begin{parts}
|
||||
|
||||
\part $1 - 6 \times ( -5 )$
|
||||
|
||||
\part $-5 - 2 \times 9$
|
||||
|
||||
\part $5 \times 5 - 2 \times 7$
|
||||
|
||||
\part $( -3 - 3 ) \times ( -5 ) + 10$
|
||||
|
||||
\part $-3 ( 5 - 4 ) \times 3$
|
||||
|
||||
\part $4 ( -5 - ( -5 ) ) + 5$
|
||||
\end{parts}
|
||||
\end{multicols}
|
||||
\vfill
|
||||
|
||||
\question
|
||||
|
||||
|
||||
Dans un sac, il y a 24 bonbons à la menthe, 32 bonbons à la fraise et 3 au chocolat. On choisit un bonbon au hasard dans ce sac.
|
||||
\begin{parts}
|
||||
\part Combien y a-t-il d'issues en tout?
|
||||
\begin{solution}
|
||||
Il y a 59 bonbons.
|
||||
\end{solution}
|
||||
\part Calculer la probabilité de tirer un bonbon à la fraise.
|
||||
\begin{solution}
|
||||
$T($ tirer un bonbon à la fraise $) = \dfrac{24}{59}$
|
||||
\end{solution}
|
||||
\part Calculer la probabilité de tirer un bonbon qui n'est pas au chocolat.
|
||||
\begin{solution}
|
||||
|
||||
$T($ tirer un bonbon à la fraise $) = \dfrac{56}{59}$
|
||||
\end{solution}
|
||||
\part Calculer la probabilité de tirer un bonbon au réglisse.
|
||||
\begin{solution}
|
||||
$T($ tirer un bonbon au réglisse $) = \dfrac{0}{59} = 0$
|
||||
\end{solution}
|
||||
\part Dans un autre sac, on place 25 bonbons à la menthe et 34 bonbons à la fraise. Lise préfère les bonbons à la menthe. Dans quel sac doit-elle tirer un bonbon pour avoir le plus de chance d'avoir un bonbon qu'elle préfère?
|
||||
\end{parts}
|
||||
|
||||
\vfill
|
||||
\end{questions}
|
||||
|
||||
\end{document}
|
||||
|
||||
%%% Local Variables:
|
||||
%%% mode: latex
|
||||
%%% TeX-master: "master"
|
||||
%%% End:
|
78
3e/DM/DM_16_09_15/48_DM_16_09_15.tex
Normal file
78
3e/DM/DM_16_09_15/48_DM_16_09_15.tex
Normal file
@@ -0,0 +1,78 @@
|
||||
\documentclass[a5paper,12pt, table]{/media/documents/Cours/Prof/Enseignements/2016-2017/tools/style/classDS}
|
||||
\usepackage{/media/documents/Cours/Prof/Enseignements/2016-2017/theme}
|
||||
\geometry{left=10mm,right=10mm, top=10mm, bottom=10mm}
|
||||
|
||||
% Title Page
|
||||
\titre{1}
|
||||
% \seconde \premiereS \PSTMG \TSTMG
|
||||
\classe{Troisième}
|
||||
\date{15 septembre 2016}
|
||||
%\duree{1 heure}
|
||||
\sujet{48}
|
||||
% DS DSCorr DM DMCorr Corr
|
||||
\typedoc{DM}
|
||||
|
||||
%\printanswers
|
||||
|
||||
\begin{document}
|
||||
|
||||
\maketitle
|
||||
|
||||
Vous devez rendre le sujet avec la copie.
|
||||
|
||||
\begin{questions}
|
||||
|
||||
\question
|
||||
Faire les calculs suivants en détaillant des étapes.
|
||||
\begin{multicols}{2}
|
||||
\begin{parts}
|
||||
|
||||
\part $-1 - 10 \times ( -1 )$
|
||||
|
||||
\part $-6 - 8 \times 10$
|
||||
|
||||
\part $-7 \times 2 - 5 \times 8$
|
||||
|
||||
\part $( -2 - 7 ) \times 10 - 2$
|
||||
|
||||
\part $6 ( -3 + 4 ) \times ( -1 )$
|
||||
|
||||
\part $1 ( 6 - ( -2 ) ) + 1$
|
||||
\end{parts}
|
||||
\end{multicols}
|
||||
\vfill
|
||||
|
||||
\question
|
||||
|
||||
|
||||
Dans un sac, il y a 60 bonbons à la menthe, 30 bonbons à la fraise et 3 au chocolat. On choisit un bonbon au hasard dans ce sac.
|
||||
\begin{parts}
|
||||
\part Combien y a-t-il d'issues en tout?
|
||||
\begin{solution}
|
||||
Il y a 93 bonbons.
|
||||
\end{solution}
|
||||
\part Calculer la probabilité de tirer un bonbon à la fraise.
|
||||
\begin{solution}
|
||||
$T($ tirer un bonbon à la fraise $) = \dfrac{60}{93}$
|
||||
\end{solution}
|
||||
\part Calculer la probabilité de tirer un bonbon qui n'est pas au chocolat.
|
||||
\begin{solution}
|
||||
|
||||
$T($ tirer un bonbon à la fraise $) = \dfrac{90}{93}$
|
||||
\end{solution}
|
||||
\part Calculer la probabilité de tirer un bonbon au réglisse.
|
||||
\begin{solution}
|
||||
$T($ tirer un bonbon au réglisse $) = \dfrac{0}{93} = 0$
|
||||
\end{solution}
|
||||
\part Dans un autre sac, on place 25 bonbons à la menthe et 34 bonbons à la fraise. Lise préfère les bonbons à la menthe. Dans quel sac doit-elle tirer un bonbon pour avoir le plus de chance d'avoir un bonbon qu'elle préfère?
|
||||
\end{parts}
|
||||
|
||||
\vfill
|
||||
\end{questions}
|
||||
|
||||
\end{document}
|
||||
|
||||
%%% Local Variables:
|
||||
%%% mode: latex
|
||||
%%% TeX-master: "master"
|
||||
%%% End:
|
78
3e/DM/DM_16_09_15/49_DM_16_09_15.tex
Normal file
78
3e/DM/DM_16_09_15/49_DM_16_09_15.tex
Normal file
@@ -0,0 +1,78 @@
|
||||
\documentclass[a5paper,12pt, table]{/media/documents/Cours/Prof/Enseignements/2016-2017/tools/style/classDS}
|
||||
\usepackage{/media/documents/Cours/Prof/Enseignements/2016-2017/theme}
|
||||
\geometry{left=10mm,right=10mm, top=10mm, bottom=10mm}
|
||||
|
||||
% Title Page
|
||||
\titre{1}
|
||||
% \seconde \premiereS \PSTMG \TSTMG
|
||||
\classe{Troisième}
|
||||
\date{15 septembre 2016}
|
||||
%\duree{1 heure}
|
||||
\sujet{49}
|
||||
% DS DSCorr DM DMCorr Corr
|
||||
\typedoc{DM}
|
||||
|
||||
%\printanswers
|
||||
|
||||
\begin{document}
|
||||
|
||||
\maketitle
|
||||
|
||||
Vous devez rendre le sujet avec la copie.
|
||||
|
||||
\begin{questions}
|
||||
|
||||
\question
|
||||
Faire les calculs suivants en détaillant des étapes.
|
||||
\begin{multicols}{2}
|
||||
\begin{parts}
|
||||
|
||||
\part $4 - 4 \times ( -7 )$
|
||||
|
||||
\part $-5 - ( -7 \times 4 )$
|
||||
|
||||
\part $-4 \times 3 + 8 \times 3$
|
||||
|
||||
\part $( -4 + 3 ) \times 1 + 6$
|
||||
|
||||
\part $9 ( 3 - 8 ) \times ( -8 )$
|
||||
|
||||
\part $6 ( 4 - 9 ) + 1$
|
||||
\end{parts}
|
||||
\end{multicols}
|
||||
\vfill
|
||||
|
||||
\question
|
||||
|
||||
|
||||
Dans un sac, il y a 80 bonbons à la menthe, 40 bonbons à la fraise et 9 au chocolat. On choisit un bonbon au hasard dans ce sac.
|
||||
\begin{parts}
|
||||
\part Combien y a-t-il d'issues en tout?
|
||||
\begin{solution}
|
||||
Il y a 129 bonbons.
|
||||
\end{solution}
|
||||
\part Calculer la probabilité de tirer un bonbon à la fraise.
|
||||
\begin{solution}
|
||||
$T($ tirer un bonbon à la fraise $) = \dfrac{80}{129}$
|
||||
\end{solution}
|
||||
\part Calculer la probabilité de tirer un bonbon qui n'est pas au chocolat.
|
||||
\begin{solution}
|
||||
|
||||
$T($ tirer un bonbon à la fraise $) = \dfrac{120}{129}$
|
||||
\end{solution}
|
||||
\part Calculer la probabilité de tirer un bonbon au réglisse.
|
||||
\begin{solution}
|
||||
$T($ tirer un bonbon au réglisse $) = \dfrac{0}{129} = 0$
|
||||
\end{solution}
|
||||
\part Dans un autre sac, on place 25 bonbons à la menthe et 34 bonbons à la fraise. Lise préfère les bonbons à la menthe. Dans quel sac doit-elle tirer un bonbon pour avoir le plus de chance d'avoir un bonbon qu'elle préfère?
|
||||
\end{parts}
|
||||
|
||||
\vfill
|
||||
\end{questions}
|
||||
|
||||
\end{document}
|
||||
|
||||
%%% Local Variables:
|
||||
%%% mode: latex
|
||||
%%% TeX-master: "master"
|
||||
%%% End:
|
78
3e/DM/DM_16_09_15/50_DM_16_09_15.tex
Normal file
78
3e/DM/DM_16_09_15/50_DM_16_09_15.tex
Normal file
@@ -0,0 +1,78 @@
|
||||
\documentclass[a5paper,12pt, table]{/media/documents/Cours/Prof/Enseignements/2016-2017/tools/style/classDS}
|
||||
\usepackage{/media/documents/Cours/Prof/Enseignements/2016-2017/theme}
|
||||
\geometry{left=10mm,right=10mm, top=10mm, bottom=10mm}
|
||||
|
||||
% Title Page
|
||||
\titre{1}
|
||||
% \seconde \premiereS \PSTMG \TSTMG
|
||||
\classe{Troisième}
|
||||
\date{15 septembre 2016}
|
||||
%\duree{1 heure}
|
||||
\sujet{50}
|
||||
% DS DSCorr DM DMCorr Corr
|
||||
\typedoc{DM}
|
||||
|
||||
%\printanswers
|
||||
|
||||
\begin{document}
|
||||
|
||||
\maketitle
|
||||
|
||||
Vous devez rendre le sujet avec la copie.
|
||||
|
||||
\begin{questions}
|
||||
|
||||
\question
|
||||
Faire les calculs suivants en détaillant des étapes.
|
||||
\begin{multicols}{2}
|
||||
\begin{parts}
|
||||
|
||||
\part $7 + 8 \times 2$
|
||||
|
||||
\part $5 - ( -8 \times ( -5 ) )$
|
||||
|
||||
\part $-5 \times ( -9 ) + 3 \times 7$
|
||||
|
||||
\part $( -1 - 9 ) \times ( -10 ) - 3$
|
||||
|
||||
\part $-2 ( 7 + 10 ) \times 2$
|
||||
|
||||
\part $-2 ( 5 - ( -10 ) ) + 6$
|
||||
\end{parts}
|
||||
\end{multicols}
|
||||
\vfill
|
||||
|
||||
\question
|
||||
|
||||
|
||||
Dans un sac, il y a 24 bonbons à la menthe, 80 bonbons à la fraise et 4 au chocolat. On choisit un bonbon au hasard dans ce sac.
|
||||
\begin{parts}
|
||||
\part Combien y a-t-il d'issues en tout?
|
||||
\begin{solution}
|
||||
Il y a 108 bonbons.
|
||||
\end{solution}
|
||||
\part Calculer la probabilité de tirer un bonbon à la fraise.
|
||||
\begin{solution}
|
||||
$T($ tirer un bonbon à la fraise $) = \dfrac{24}{108}$
|
||||
\end{solution}
|
||||
\part Calculer la probabilité de tirer un bonbon qui n'est pas au chocolat.
|
||||
\begin{solution}
|
||||
|
||||
$T($ tirer un bonbon à la fraise $) = \dfrac{104}{108}$
|
||||
\end{solution}
|
||||
\part Calculer la probabilité de tirer un bonbon au réglisse.
|
||||
\begin{solution}
|
||||
$T($ tirer un bonbon au réglisse $) = \dfrac{0}{108} = 0$
|
||||
\end{solution}
|
||||
\part Dans un autre sac, on place 25 bonbons à la menthe et 34 bonbons à la fraise. Lise préfère les bonbons à la menthe. Dans quel sac doit-elle tirer un bonbon pour avoir le plus de chance d'avoir un bonbon qu'elle préfère?
|
||||
\end{parts}
|
||||
|
||||
\vfill
|
||||
\end{questions}
|
||||
|
||||
\end{document}
|
||||
|
||||
%%% Local Variables:
|
||||
%%% mode: latex
|
||||
%%% TeX-master: "master"
|
||||
%%% End:
|
78
3e/DM/DM_16_09_15/51_DM_16_09_15.tex
Normal file
78
3e/DM/DM_16_09_15/51_DM_16_09_15.tex
Normal file
@@ -0,0 +1,78 @@
|
||||
\documentclass[a5paper,12pt, table]{/media/documents/Cours/Prof/Enseignements/2016-2017/tools/style/classDS}
|
||||
\usepackage{/media/documents/Cours/Prof/Enseignements/2016-2017/theme}
|
||||
\geometry{left=10mm,right=10mm, top=10mm, bottom=10mm}
|
||||
|
||||
% Title Page
|
||||
\titre{1}
|
||||
% \seconde \premiereS \PSTMG \TSTMG
|
||||
\classe{Troisième}
|
||||
\date{15 septembre 2016}
|
||||
%\duree{1 heure}
|
||||
\sujet{51}
|
||||
% DS DSCorr DM DMCorr Corr
|
||||
\typedoc{DM}
|
||||
|
||||
%\printanswers
|
||||
|
||||
\begin{document}
|
||||
|
||||
\maketitle
|
||||
|
||||
Vous devez rendre le sujet avec la copie.
|
||||
|
||||
\begin{questions}
|
||||
|
||||
\question
|
||||
Faire les calculs suivants en détaillant des étapes.
|
||||
\begin{multicols}{2}
|
||||
\begin{parts}
|
||||
|
||||
\part $-2 - 8 \times ( -5 )$
|
||||
|
||||
\part $1 - ( -10 \times ( -4 ) )$
|
||||
|
||||
\part $7 \times ( -4 ) + 9 \times 2$
|
||||
|
||||
\part $( 2 - 6 ) \times ( -5 ) - 6$
|
||||
|
||||
\part $-5 ( 8 - 9 ) \times 2$
|
||||
|
||||
\part $-4 ( 2 - 10 ) + 9$
|
||||
\end{parts}
|
||||
\end{multicols}
|
||||
\vfill
|
||||
|
||||
\question
|
||||
|
||||
|
||||
Dans un sac, il y a 4 bonbons à la menthe, 12 bonbons à la fraise et 6 au chocolat. On choisit un bonbon au hasard dans ce sac.
|
||||
\begin{parts}
|
||||
\part Combien y a-t-il d'issues en tout?
|
||||
\begin{solution}
|
||||
Il y a 22 bonbons.
|
||||
\end{solution}
|
||||
\part Calculer la probabilité de tirer un bonbon à la fraise.
|
||||
\begin{solution}
|
||||
$T($ tirer un bonbon à la fraise $) = \dfrac{4}{22}$
|
||||
\end{solution}
|
||||
\part Calculer la probabilité de tirer un bonbon qui n'est pas au chocolat.
|
||||
\begin{solution}
|
||||
|
||||
$T($ tirer un bonbon à la fraise $) = \dfrac{16}{22}$
|
||||
\end{solution}
|
||||
\part Calculer la probabilité de tirer un bonbon au réglisse.
|
||||
\begin{solution}
|
||||
$T($ tirer un bonbon au réglisse $) = \dfrac{0}{22} = 0$
|
||||
\end{solution}
|
||||
\part Dans un autre sac, on place 25 bonbons à la menthe et 34 bonbons à la fraise. Lise préfère les bonbons à la menthe. Dans quel sac doit-elle tirer un bonbon pour avoir le plus de chance d'avoir un bonbon qu'elle préfère?
|
||||
\end{parts}
|
||||
|
||||
\vfill
|
||||
\end{questions}
|
||||
|
||||
\end{document}
|
||||
|
||||
%%% Local Variables:
|
||||
%%% mode: latex
|
||||
%%% TeX-master: "master"
|
||||
%%% End:
|
78
3e/DM/DM_16_09_15/52_DM_16_09_15.tex
Normal file
78
3e/DM/DM_16_09_15/52_DM_16_09_15.tex
Normal file
@@ -0,0 +1,78 @@
|
||||
\documentclass[a5paper,12pt, table]{/media/documents/Cours/Prof/Enseignements/2016-2017/tools/style/classDS}
|
||||
\usepackage{/media/documents/Cours/Prof/Enseignements/2016-2017/theme}
|
||||
\geometry{left=10mm,right=10mm, top=10mm, bottom=10mm}
|
||||
|
||||
% Title Page
|
||||
\titre{1}
|
||||
% \seconde \premiereS \PSTMG \TSTMG
|
||||
\classe{Troisième}
|
||||
\date{15 septembre 2016}
|
||||
%\duree{1 heure}
|
||||
\sujet{52}
|
||||
% DS DSCorr DM DMCorr Corr
|
||||
\typedoc{DM}
|
||||
|
||||
%\printanswers
|
||||
|
||||
\begin{document}
|
||||
|
||||
\maketitle
|
||||
|
||||
Vous devez rendre le sujet avec la copie.
|
||||
|
||||
\begin{questions}
|
||||
|
||||
\question
|
||||
Faire les calculs suivants en détaillant des étapes.
|
||||
\begin{multicols}{2}
|
||||
\begin{parts}
|
||||
|
||||
\part $-8 + 4 \times ( -3 )$
|
||||
|
||||
\part $-7 - 2 \times 8$
|
||||
|
||||
\part $6 \times ( -5 ) + 6 \times 7$
|
||||
|
||||
\part $( 7 - 5 ) \times ( -8 ) - 3$
|
||||
|
||||
\part $-3 ( -8 - 2 ) \times 5$
|
||||
|
||||
\part $8 ( 3 - ( -2 ) ) + 2$
|
||||
\end{parts}
|
||||
\end{multicols}
|
||||
\vfill
|
||||
|
||||
\question
|
||||
|
||||
|
||||
Dans un sac, il y a 90 bonbons à la menthe, 20 bonbons à la fraise et 10 au chocolat. On choisit un bonbon au hasard dans ce sac.
|
||||
\begin{parts}
|
||||
\part Combien y a-t-il d'issues en tout?
|
||||
\begin{solution}
|
||||
Il y a 120 bonbons.
|
||||
\end{solution}
|
||||
\part Calculer la probabilité de tirer un bonbon à la fraise.
|
||||
\begin{solution}
|
||||
$T($ tirer un bonbon à la fraise $) = \dfrac{90}{120}$
|
||||
\end{solution}
|
||||
\part Calculer la probabilité de tirer un bonbon qui n'est pas au chocolat.
|
||||
\begin{solution}
|
||||
|
||||
$T($ tirer un bonbon à la fraise $) = \dfrac{110}{120}$
|
||||
\end{solution}
|
||||
\part Calculer la probabilité de tirer un bonbon au réglisse.
|
||||
\begin{solution}
|
||||
$T($ tirer un bonbon au réglisse $) = \dfrac{0}{120} = 0$
|
||||
\end{solution}
|
||||
\part Dans un autre sac, on place 25 bonbons à la menthe et 34 bonbons à la fraise. Lise préfère les bonbons à la menthe. Dans quel sac doit-elle tirer un bonbon pour avoir le plus de chance d'avoir un bonbon qu'elle préfère?
|
||||
\end{parts}
|
||||
|
||||
\vfill
|
||||
\end{questions}
|
||||
|
||||
\end{document}
|
||||
|
||||
%%% Local Variables:
|
||||
%%% mode: latex
|
||||
%%% TeX-master: "master"
|
||||
%%% End:
|
78
3e/DM/DM_16_09_15/53_DM_16_09_15.tex
Normal file
78
3e/DM/DM_16_09_15/53_DM_16_09_15.tex
Normal file
@@ -0,0 +1,78 @@
|
||||
\documentclass[a5paper,12pt, table]{/media/documents/Cours/Prof/Enseignements/2016-2017/tools/style/classDS}
|
||||
\usepackage{/media/documents/Cours/Prof/Enseignements/2016-2017/theme}
|
||||
\geometry{left=10mm,right=10mm, top=10mm, bottom=10mm}
|
||||
|
||||
% Title Page
|
||||
\titre{1}
|
||||
% \seconde \premiereS \PSTMG \TSTMG
|
||||
\classe{Troisième}
|
||||
\date{15 septembre 2016}
|
||||
%\duree{1 heure}
|
||||
\sujet{53}
|
||||
% DS DSCorr DM DMCorr Corr
|
||||
\typedoc{DM}
|
||||
|
||||
%\printanswers
|
||||
|
||||
\begin{document}
|
||||
|
||||
\maketitle
|
||||
|
||||
Vous devez rendre le sujet avec la copie.
|
||||
|
||||
\begin{questions}
|
||||
|
||||
\question
|
||||
Faire les calculs suivants en détaillant des étapes.
|
||||
\begin{multicols}{2}
|
||||
\begin{parts}
|
||||
|
||||
\part $-10 - 5 \times ( -2 )$
|
||||
|
||||
\part $2 - ( -4 \times 8 )$
|
||||
|
||||
\part $9 \times ( -10 ) + 6 \times ( -10 )$
|
||||
|
||||
\part $( 1 + 6 ) \times ( -6 ) - 1$
|
||||
|
||||
\part $-6 ( -8 - 7 ) \times ( -6 )$
|
||||
|
||||
\part $-4 ( 7 - 1 ) - 9$
|
||||
\end{parts}
|
||||
\end{multicols}
|
||||
\vfill
|
||||
|
||||
\question
|
||||
|
||||
|
||||
Dans un sac, il y a 30 bonbons à la menthe, 50 bonbons à la fraise et 6 au chocolat. On choisit un bonbon au hasard dans ce sac.
|
||||
\begin{parts}
|
||||
\part Combien y a-t-il d'issues en tout?
|
||||
\begin{solution}
|
||||
Il y a 86 bonbons.
|
||||
\end{solution}
|
||||
\part Calculer la probabilité de tirer un bonbon à la fraise.
|
||||
\begin{solution}
|
||||
$T($ tirer un bonbon à la fraise $) = \dfrac{30}{86}$
|
||||
\end{solution}
|
||||
\part Calculer la probabilité de tirer un bonbon qui n'est pas au chocolat.
|
||||
\begin{solution}
|
||||
|
||||
$T($ tirer un bonbon à la fraise $) = \dfrac{80}{86}$
|
||||
\end{solution}
|
||||
\part Calculer la probabilité de tirer un bonbon au réglisse.
|
||||
\begin{solution}
|
||||
$T($ tirer un bonbon au réglisse $) = \dfrac{0}{86} = 0$
|
||||
\end{solution}
|
||||
\part Dans un autre sac, on place 25 bonbons à la menthe et 34 bonbons à la fraise. Lise préfère les bonbons à la menthe. Dans quel sac doit-elle tirer un bonbon pour avoir le plus de chance d'avoir un bonbon qu'elle préfère?
|
||||
\end{parts}
|
||||
|
||||
\vfill
|
||||
\end{questions}
|
||||
|
||||
\end{document}
|
||||
|
||||
%%% Local Variables:
|
||||
%%% mode: latex
|
||||
%%% TeX-master: "master"
|
||||
%%% End:
|
78
3e/DM/DM_16_09_15/54_DM_16_09_15.tex
Normal file
78
3e/DM/DM_16_09_15/54_DM_16_09_15.tex
Normal file
@@ -0,0 +1,78 @@
|
||||
\documentclass[a5paper,12pt, table]{/media/documents/Cours/Prof/Enseignements/2016-2017/tools/style/classDS}
|
||||
\usepackage{/media/documents/Cours/Prof/Enseignements/2016-2017/theme}
|
||||
\geometry{left=10mm,right=10mm, top=10mm, bottom=10mm}
|
||||
|
||||
% Title Page
|
||||
\titre{1}
|
||||
% \seconde \premiereS \PSTMG \TSTMG
|
||||
\classe{Troisième}
|
||||
\date{15 septembre 2016}
|
||||
%\duree{1 heure}
|
||||
\sujet{54}
|
||||
% DS DSCorr DM DMCorr Corr
|
||||
\typedoc{DM}
|
||||
|
||||
%\printanswers
|
||||
|
||||
\begin{document}
|
||||
|
||||
\maketitle
|
||||
|
||||
Vous devez rendre le sujet avec la copie.
|
||||
|
||||
\begin{questions}
|
||||
|
||||
\question
|
||||
Faire les calculs suivants en détaillant des étapes.
|
||||
\begin{multicols}{2}
|
||||
\begin{parts}
|
||||
|
||||
\part $2 + 1 \times ( -5 )$
|
||||
|
||||
\part $-9 - ( -5 \times ( -9 ) )$
|
||||
|
||||
\part $-7 \times 5 + 7 \times 3$
|
||||
|
||||
\part $( 8 + 6 ) \times 4 + 1$
|
||||
|
||||
\part $-10 ( 2 - 4 ) \times 7$
|
||||
|
||||
\part $-7 ( -5 - 7 ) - 3$
|
||||
\end{parts}
|
||||
\end{multicols}
|
||||
\vfill
|
||||
|
||||
\question
|
||||
|
||||
|
||||
Dans un sac, il y a 12 bonbons à la menthe, 9 bonbons à la fraise et 5 au chocolat. On choisit un bonbon au hasard dans ce sac.
|
||||
\begin{parts}
|
||||
\part Combien y a-t-il d'issues en tout?
|
||||
\begin{solution}
|
||||
Il y a 26 bonbons.
|
||||
\end{solution}
|
||||
\part Calculer la probabilité de tirer un bonbon à la fraise.
|
||||
\begin{solution}
|
||||
$T($ tirer un bonbon à la fraise $) = \dfrac{12}{26}$
|
||||
\end{solution}
|
||||
\part Calculer la probabilité de tirer un bonbon qui n'est pas au chocolat.
|
||||
\begin{solution}
|
||||
|
||||
$T($ tirer un bonbon à la fraise $) = \dfrac{21}{26}$
|
||||
\end{solution}
|
||||
\part Calculer la probabilité de tirer un bonbon au réglisse.
|
||||
\begin{solution}
|
||||
$T($ tirer un bonbon au réglisse $) = \dfrac{0}{26} = 0$
|
||||
\end{solution}
|
||||
\part Dans un autre sac, on place 25 bonbons à la menthe et 34 bonbons à la fraise. Lise préfère les bonbons à la menthe. Dans quel sac doit-elle tirer un bonbon pour avoir le plus de chance d'avoir un bonbon qu'elle préfère?
|
||||
\end{parts}
|
||||
|
||||
\vfill
|
||||
\end{questions}
|
||||
|
||||
\end{document}
|
||||
|
||||
%%% Local Variables:
|
||||
%%% mode: latex
|
||||
%%% TeX-master: "master"
|
||||
%%% End:
|
78
3e/DM/DM_16_09_15/55_DM_16_09_15.tex
Normal file
78
3e/DM/DM_16_09_15/55_DM_16_09_15.tex
Normal file
@@ -0,0 +1,78 @@
|
||||
\documentclass[a5paper,12pt, table]{/media/documents/Cours/Prof/Enseignements/2016-2017/tools/style/classDS}
|
||||
\usepackage{/media/documents/Cours/Prof/Enseignements/2016-2017/theme}
|
||||
\geometry{left=10mm,right=10mm, top=10mm, bottom=10mm}
|
||||
|
||||
% Title Page
|
||||
\titre{1}
|
||||
% \seconde \premiereS \PSTMG \TSTMG
|
||||
\classe{Troisième}
|
||||
\date{15 septembre 2016}
|
||||
%\duree{1 heure}
|
||||
\sujet{55}
|
||||
% DS DSCorr DM DMCorr Corr
|
||||
\typedoc{DM}
|
||||
|
||||
%\printanswers
|
||||
|
||||
\begin{document}
|
||||
|
||||
\maketitle
|
||||
|
||||
Vous devez rendre le sujet avec la copie.
|
||||
|
||||
\begin{questions}
|
||||
|
||||
\question
|
||||
Faire les calculs suivants en détaillant des étapes.
|
||||
\begin{multicols}{2}
|
||||
\begin{parts}
|
||||
|
||||
\part $10 - 9 \times 6$
|
||||
|
||||
\part $-6 - ( -8 \times 6 )$
|
||||
|
||||
\part $10 \times 7 + 4 \times 3$
|
||||
|
||||
\part $( -8 - 1 ) \times ( -9 ) - 2$
|
||||
|
||||
\part $-4 ( 6 - 5 ) \times 7$
|
||||
|
||||
\part $-10 ( -10 - ( -6 ) ) + 9$
|
||||
\end{parts}
|
||||
\end{multicols}
|
||||
\vfill
|
||||
|
||||
\question
|
||||
|
||||
|
||||
Dans un sac, il y a 70 bonbons à la menthe, 90 bonbons à la fraise et 7 au chocolat. On choisit un bonbon au hasard dans ce sac.
|
||||
\begin{parts}
|
||||
\part Combien y a-t-il d'issues en tout?
|
||||
\begin{solution}
|
||||
Il y a 167 bonbons.
|
||||
\end{solution}
|
||||
\part Calculer la probabilité de tirer un bonbon à la fraise.
|
||||
\begin{solution}
|
||||
$T($ tirer un bonbon à la fraise $) = \dfrac{70}{167}$
|
||||
\end{solution}
|
||||
\part Calculer la probabilité de tirer un bonbon qui n'est pas au chocolat.
|
||||
\begin{solution}
|
||||
|
||||
$T($ tirer un bonbon à la fraise $) = \dfrac{160}{167}$
|
||||
\end{solution}
|
||||
\part Calculer la probabilité de tirer un bonbon au réglisse.
|
||||
\begin{solution}
|
||||
$T($ tirer un bonbon au réglisse $) = \dfrac{0}{167} = 0$
|
||||
\end{solution}
|
||||
\part Dans un autre sac, on place 25 bonbons à la menthe et 34 bonbons à la fraise. Lise préfère les bonbons à la menthe. Dans quel sac doit-elle tirer un bonbon pour avoir le plus de chance d'avoir un bonbon qu'elle préfère?
|
||||
\end{parts}
|
||||
|
||||
\vfill
|
||||
\end{questions}
|
||||
|
||||
\end{document}
|
||||
|
||||
%%% Local Variables:
|
||||
%%% mode: latex
|
||||
%%% TeX-master: "master"
|
||||
%%% End:
|
78
3e/DM/DM_16_09_15/56_DM_16_09_15.tex
Normal file
78
3e/DM/DM_16_09_15/56_DM_16_09_15.tex
Normal file
@@ -0,0 +1,78 @@
|
||||
\documentclass[a5paper,12pt, table]{/media/documents/Cours/Prof/Enseignements/2016-2017/tools/style/classDS}
|
||||
\usepackage{/media/documents/Cours/Prof/Enseignements/2016-2017/theme}
|
||||
\geometry{left=10mm,right=10mm, top=10mm, bottom=10mm}
|
||||
|
||||
% Title Page
|
||||
\titre{1}
|
||||
% \seconde \premiereS \PSTMG \TSTMG
|
||||
\classe{Troisième}
|
||||
\date{15 septembre 2016}
|
||||
%\duree{1 heure}
|
||||
\sujet{56}
|
||||
% DS DSCorr DM DMCorr Corr
|
||||
\typedoc{DM}
|
||||
|
||||
%\printanswers
|
||||
|
||||
\begin{document}
|
||||
|
||||
\maketitle
|
||||
|
||||
Vous devez rendre le sujet avec la copie.
|
||||
|
||||
\begin{questions}
|
||||
|
||||
\question
|
||||
Faire les calculs suivants en détaillant des étapes.
|
||||
\begin{multicols}{2}
|
||||
\begin{parts}
|
||||
|
||||
\part $6 - 10 \times ( -9 )$
|
||||
|
||||
\part $1 - 4 \times ( -7 )$
|
||||
|
||||
\part $-8 \times ( -5 ) + 10 \times 4$
|
||||
|
||||
\part $( -10 - 7 ) \times ( -10 ) - 10$
|
||||
|
||||
\part $-6 ( 1 - 8 ) \times 10$
|
||||
|
||||
\part $4 ( 1 - ( -8 ) ) - 4$
|
||||
\end{parts}
|
||||
\end{multicols}
|
||||
\vfill
|
||||
|
||||
\question
|
||||
|
||||
|
||||
Dans un sac, il y a 18 bonbons à la menthe, 14 bonbons à la fraise et 4 au chocolat. On choisit un bonbon au hasard dans ce sac.
|
||||
\begin{parts}
|
||||
\part Combien y a-t-il d'issues en tout?
|
||||
\begin{solution}
|
||||
Il y a 36 bonbons.
|
||||
\end{solution}
|
||||
\part Calculer la probabilité de tirer un bonbon à la fraise.
|
||||
\begin{solution}
|
||||
$T($ tirer un bonbon à la fraise $) = \dfrac{18}{36}$
|
||||
\end{solution}
|
||||
\part Calculer la probabilité de tirer un bonbon qui n'est pas au chocolat.
|
||||
\begin{solution}
|
||||
|
||||
$T($ tirer un bonbon à la fraise $) = \dfrac{32}{36}$
|
||||
\end{solution}
|
||||
\part Calculer la probabilité de tirer un bonbon au réglisse.
|
||||
\begin{solution}
|
||||
$T($ tirer un bonbon au réglisse $) = \dfrac{0}{36} = 0$
|
||||
\end{solution}
|
||||
\part Dans un autre sac, on place 25 bonbons à la menthe et 34 bonbons à la fraise. Lise préfère les bonbons à la menthe. Dans quel sac doit-elle tirer un bonbon pour avoir le plus de chance d'avoir un bonbon qu'elle préfère?
|
||||
\end{parts}
|
||||
|
||||
\vfill
|
||||
\end{questions}
|
||||
|
||||
\end{document}
|
||||
|
||||
%%% Local Variables:
|
||||
%%% mode: latex
|
||||
%%% TeX-master: "master"
|
||||
%%% End:
|
78
3e/DM/DM_16_09_15/57_DM_16_09_15.tex
Normal file
78
3e/DM/DM_16_09_15/57_DM_16_09_15.tex
Normal file
@@ -0,0 +1,78 @@
|
||||
\documentclass[a5paper,12pt, table]{/media/documents/Cours/Prof/Enseignements/2016-2017/tools/style/classDS}
|
||||
\usepackage{/media/documents/Cours/Prof/Enseignements/2016-2017/theme}
|
||||
\geometry{left=10mm,right=10mm, top=10mm, bottom=10mm}
|
||||
|
||||
% Title Page
|
||||
\titre{1}
|
||||
% \seconde \premiereS \PSTMG \TSTMG
|
||||
\classe{Troisième}
|
||||
\date{15 septembre 2016}
|
||||
%\duree{1 heure}
|
||||
\sujet{57}
|
||||
% DS DSCorr DM DMCorr Corr
|
||||
\typedoc{DM}
|
||||
|
||||
%\printanswers
|
||||
|
||||
\begin{document}
|
||||
|
||||
\maketitle
|
||||
|
||||
Vous devez rendre le sujet avec la copie.
|
||||
|
||||
\begin{questions}
|
||||
|
||||
\question
|
||||
Faire les calculs suivants en détaillant des étapes.
|
||||
\begin{multicols}{2}
|
||||
\begin{parts}
|
||||
|
||||
\part $6 + 3 \times ( -6 )$
|
||||
|
||||
\part $-7 - 4 \times ( -5 )$
|
||||
|
||||
\part $3 \times 7 - 4 \times 7$
|
||||
|
||||
\part $( -1 + 4 ) \times ( -5 ) + 1$
|
||||
|
||||
\part $-1 ( 1 + 9 ) \times 5$
|
||||
|
||||
\part $6 ( -6 - 7 ) + 9$
|
||||
\end{parts}
|
||||
\end{multicols}
|
||||
\vfill
|
||||
|
||||
\question
|
||||
|
||||
|
||||
Dans un sac, il y a 18 bonbons à la menthe, 12 bonbons à la fraise et 7 au chocolat. On choisit un bonbon au hasard dans ce sac.
|
||||
\begin{parts}
|
||||
\part Combien y a-t-il d'issues en tout?
|
||||
\begin{solution}
|
||||
Il y a 37 bonbons.
|
||||
\end{solution}
|
||||
\part Calculer la probabilité de tirer un bonbon à la fraise.
|
||||
\begin{solution}
|
||||
$T($ tirer un bonbon à la fraise $) = \dfrac{18}{37}$
|
||||
\end{solution}
|
||||
\part Calculer la probabilité de tirer un bonbon qui n'est pas au chocolat.
|
||||
\begin{solution}
|
||||
|
||||
$T($ tirer un bonbon à la fraise $) = \dfrac{30}{37}$
|
||||
\end{solution}
|
||||
\part Calculer la probabilité de tirer un bonbon au réglisse.
|
||||
\begin{solution}
|
||||
$T($ tirer un bonbon au réglisse $) = \dfrac{0}{37} = 0$
|
||||
\end{solution}
|
||||
\part Dans un autre sac, on place 25 bonbons à la menthe et 34 bonbons à la fraise. Lise préfère les bonbons à la menthe. Dans quel sac doit-elle tirer un bonbon pour avoir le plus de chance d'avoir un bonbon qu'elle préfère?
|
||||
\end{parts}
|
||||
|
||||
\vfill
|
||||
\end{questions}
|
||||
|
||||
\end{document}
|
||||
|
||||
%%% Local Variables:
|
||||
%%% mode: latex
|
||||
%%% TeX-master: "master"
|
||||
%%% End:
|
78
3e/DM/DM_16_09_15/58_DM_16_09_15.tex
Normal file
78
3e/DM/DM_16_09_15/58_DM_16_09_15.tex
Normal file
@@ -0,0 +1,78 @@
|
||||
\documentclass[a5paper,12pt, table]{/media/documents/Cours/Prof/Enseignements/2016-2017/tools/style/classDS}
|
||||
\usepackage{/media/documents/Cours/Prof/Enseignements/2016-2017/theme}
|
||||
\geometry{left=10mm,right=10mm, top=10mm, bottom=10mm}
|
||||
|
||||
% Title Page
|
||||
\titre{1}
|
||||
% \seconde \premiereS \PSTMG \TSTMG
|
||||
\classe{Troisième}
|
||||
\date{15 septembre 2016}
|
||||
%\duree{1 heure}
|
||||
\sujet{58}
|
||||
% DS DSCorr DM DMCorr Corr
|
||||
\typedoc{DM}
|
||||
|
||||
%\printanswers
|
||||
|
||||
\begin{document}
|
||||
|
||||
\maketitle
|
||||
|
||||
Vous devez rendre le sujet avec la copie.
|
||||
|
||||
\begin{questions}
|
||||
|
||||
\question
|
||||
Faire les calculs suivants en détaillant des étapes.
|
||||
\begin{multicols}{2}
|
||||
\begin{parts}
|
||||
|
||||
\part $-8 - 10 \times 6$
|
||||
|
||||
\part $-3 - ( -8 \times ( -3 ) )$
|
||||
|
||||
\part $-10 \times 4 + 7 \times 5$
|
||||
|
||||
\part $( -10 + 7 ) \times 5 + 8$
|
||||
|
||||
\part $4 ( -8 + 4 ) \times 1$
|
||||
|
||||
\part $-8 ( 9 - ( -5 ) ) - 3$
|
||||
\end{parts}
|
||||
\end{multicols}
|
||||
\vfill
|
||||
|
||||
\question
|
||||
|
||||
|
||||
Dans un sac, il y a 70 bonbons à la menthe, 80 bonbons à la fraise et 3 au chocolat. On choisit un bonbon au hasard dans ce sac.
|
||||
\begin{parts}
|
||||
\part Combien y a-t-il d'issues en tout?
|
||||
\begin{solution}
|
||||
Il y a 153 bonbons.
|
||||
\end{solution}
|
||||
\part Calculer la probabilité de tirer un bonbon à la fraise.
|
||||
\begin{solution}
|
||||
$T($ tirer un bonbon à la fraise $) = \dfrac{70}{153}$
|
||||
\end{solution}
|
||||
\part Calculer la probabilité de tirer un bonbon qui n'est pas au chocolat.
|
||||
\begin{solution}
|
||||
|
||||
$T($ tirer un bonbon à la fraise $) = \dfrac{150}{153}$
|
||||
\end{solution}
|
||||
\part Calculer la probabilité de tirer un bonbon au réglisse.
|
||||
\begin{solution}
|
||||
$T($ tirer un bonbon au réglisse $) = \dfrac{0}{153} = 0$
|
||||
\end{solution}
|
||||
\part Dans un autre sac, on place 25 bonbons à la menthe et 34 bonbons à la fraise. Lise préfère les bonbons à la menthe. Dans quel sac doit-elle tirer un bonbon pour avoir le plus de chance d'avoir un bonbon qu'elle préfère?
|
||||
\end{parts}
|
||||
|
||||
\vfill
|
||||
\end{questions}
|
||||
|
||||
\end{document}
|
||||
|
||||
%%% Local Variables:
|
||||
%%% mode: latex
|
||||
%%% TeX-master: "master"
|
||||
%%% End:
|
78
3e/DM/DM_16_09_15/59_DM_16_09_15.tex
Normal file
78
3e/DM/DM_16_09_15/59_DM_16_09_15.tex
Normal file
@@ -0,0 +1,78 @@
|
||||
\documentclass[a5paper,12pt, table]{/media/documents/Cours/Prof/Enseignements/2016-2017/tools/style/classDS}
|
||||
\usepackage{/media/documents/Cours/Prof/Enseignements/2016-2017/theme}
|
||||
\geometry{left=10mm,right=10mm, top=10mm, bottom=10mm}
|
||||
|
||||
% Title Page
|
||||
\titre{1}
|
||||
% \seconde \premiereS \PSTMG \TSTMG
|
||||
\classe{Troisième}
|
||||
\date{15 septembre 2016}
|
||||
%\duree{1 heure}
|
||||
\sujet{59}
|
||||
% DS DSCorr DM DMCorr Corr
|
||||
\typedoc{DM}
|
||||
|
||||
%\printanswers
|
||||
|
||||
\begin{document}
|
||||
|
||||
\maketitle
|
||||
|
||||
Vous devez rendre le sujet avec la copie.
|
||||
|
||||
\begin{questions}
|
||||
|
||||
\question
|
||||
Faire les calculs suivants en détaillant des étapes.
|
||||
\begin{multicols}{2}
|
||||
\begin{parts}
|
||||
|
||||
\part $-3 - 9 \times 1$
|
||||
|
||||
\part $-4 - ( -3 \times ( -2 ) )$
|
||||
|
||||
\part $-8 \times 2 + 1 \times 7$
|
||||
|
||||
\part $( 8 - 5 ) \times ( -4 ) - 1$
|
||||
|
||||
\part $-4 ( 10 + 9 ) \times 4$
|
||||
|
||||
\part $-1 ( 4 - 1 ) - 10$
|
||||
\end{parts}
|
||||
\end{multicols}
|
||||
\vfill
|
||||
|
||||
\question
|
||||
|
||||
|
||||
Dans un sac, il y a 18 bonbons à la menthe, 42 bonbons à la fraise et 9 au chocolat. On choisit un bonbon au hasard dans ce sac.
|
||||
\begin{parts}
|
||||
\part Combien y a-t-il d'issues en tout?
|
||||
\begin{solution}
|
||||
Il y a 69 bonbons.
|
||||
\end{solution}
|
||||
\part Calculer la probabilité de tirer un bonbon à la fraise.
|
||||
\begin{solution}
|
||||
$T($ tirer un bonbon à la fraise $) = \dfrac{18}{69}$
|
||||
\end{solution}
|
||||
\part Calculer la probabilité de tirer un bonbon qui n'est pas au chocolat.
|
||||
\begin{solution}
|
||||
|
||||
$T($ tirer un bonbon à la fraise $) = \dfrac{60}{69}$
|
||||
\end{solution}
|
||||
\part Calculer la probabilité de tirer un bonbon au réglisse.
|
||||
\begin{solution}
|
||||
$T($ tirer un bonbon au réglisse $) = \dfrac{0}{69} = 0$
|
||||
\end{solution}
|
||||
\part Dans un autre sac, on place 25 bonbons à la menthe et 34 bonbons à la fraise. Lise préfère les bonbons à la menthe. Dans quel sac doit-elle tirer un bonbon pour avoir le plus de chance d'avoir un bonbon qu'elle préfère?
|
||||
\end{parts}
|
||||
|
||||
\vfill
|
||||
\end{questions}
|
||||
|
||||
\end{document}
|
||||
|
||||
%%% Local Variables:
|
||||
%%% mode: latex
|
||||
%%% TeX-master: "master"
|
||||
%%% End:
|
78
3e/DM/DM_16_09_15/60_DM_16_09_15.tex
Normal file
78
3e/DM/DM_16_09_15/60_DM_16_09_15.tex
Normal file
@@ -0,0 +1,78 @@
|
||||
\documentclass[a5paper,12pt, table]{/media/documents/Cours/Prof/Enseignements/2016-2017/tools/style/classDS}
|
||||
\usepackage{/media/documents/Cours/Prof/Enseignements/2016-2017/theme}
|
||||
\geometry{left=10mm,right=10mm, top=10mm, bottom=10mm}
|
||||
|
||||
% Title Page
|
||||
\titre{1}
|
||||
% \seconde \premiereS \PSTMG \TSTMG
|
||||
\classe{Troisième}
|
||||
\date{15 septembre 2016}
|
||||
%\duree{1 heure}
|
||||
\sujet{60}
|
||||
% DS DSCorr DM DMCorr Corr
|
||||
\typedoc{DM}
|
||||
|
||||
%\printanswers
|
||||
|
||||
\begin{document}
|
||||
|
||||
\maketitle
|
||||
|
||||
Vous devez rendre le sujet avec la copie.
|
||||
|
||||
\begin{questions}
|
||||
|
||||
\question
|
||||
Faire les calculs suivants en détaillant des étapes.
|
||||
\begin{multicols}{2}
|
||||
\begin{parts}
|
||||
|
||||
\part $-3 + 2 \times 6$
|
||||
|
||||
\part $10 - ( -8 \times ( -4 ) )$
|
||||
|
||||
\part $7 \times 7 + 2 \times 5$
|
||||
|
||||
\part $( -6 + 10 ) \times 6 + 10$
|
||||
|
||||
\part $-10 ( 5 - 1 ) \times 4$
|
||||
|
||||
\part $-7 ( -3 - ( -5 ) ) + 6$
|
||||
\end{parts}
|
||||
\end{multicols}
|
||||
\vfill
|
||||
|
||||
\question
|
||||
|
||||
|
||||
Dans un sac, il y a 40 bonbons à la menthe, 10 bonbons à la fraise et 2 au chocolat. On choisit un bonbon au hasard dans ce sac.
|
||||
\begin{parts}
|
||||
\part Combien y a-t-il d'issues en tout?
|
||||
\begin{solution}
|
||||
Il y a 52 bonbons.
|
||||
\end{solution}
|
||||
\part Calculer la probabilité de tirer un bonbon à la fraise.
|
||||
\begin{solution}
|
||||
$T($ tirer un bonbon à la fraise $) = \dfrac{40}{52}$
|
||||
\end{solution}
|
||||
\part Calculer la probabilité de tirer un bonbon qui n'est pas au chocolat.
|
||||
\begin{solution}
|
||||
|
||||
$T($ tirer un bonbon à la fraise $) = \dfrac{50}{52}$
|
||||
\end{solution}
|
||||
\part Calculer la probabilité de tirer un bonbon au réglisse.
|
||||
\begin{solution}
|
||||
$T($ tirer un bonbon au réglisse $) = \dfrac{0}{52} = 0$
|
||||
\end{solution}
|
||||
\part Dans un autre sac, on place 25 bonbons à la menthe et 34 bonbons à la fraise. Lise préfère les bonbons à la menthe. Dans quel sac doit-elle tirer un bonbon pour avoir le plus de chance d'avoir un bonbon qu'elle préfère?
|
||||
\end{parts}
|
||||
|
||||
\vfill
|
||||
\end{questions}
|
||||
|
||||
\end{document}
|
||||
|
||||
%%% Local Variables:
|
||||
%%% mode: latex
|
||||
%%% TeX-master: "master"
|
||||
%%% End:
|
BIN
3e/DM/DM_16_09_15/all_DM_16_09_15.pdf
Normal file
BIN
3e/DM/DM_16_09_15/all_DM_16_09_15.pdf
Normal file
Binary file not shown.
BIN
3e/DM/DM_16_09_15/corr_DM_16_09_15.pdf
Normal file
BIN
3e/DM/DM_16_09_15/corr_DM_16_09_15.pdf
Normal file
Binary file not shown.
96
3e/DM/DM_16_09_15/remediation/01_DM_16_09_15_remediation.tex
Normal file
96
3e/DM/DM_16_09_15/remediation/01_DM_16_09_15_remediation.tex
Normal file
@@ -0,0 +1,96 @@
|
||||
\documentclass[a5paper,12pt, table]{/media/documents/Cours/Prof/Enseignements/2016-2017/tools/style/classExo}
|
||||
\usepackage{/media/documents/Cours/Prof/Enseignements/2016-2017/theme}
|
||||
\geometry{left=10mm,right=10mm, top=10mm, bottom=10mm}
|
||||
|
||||
% Title Page
|
||||
\titre{Remediation priorité calculs}
|
||||
% \seconde \premiereS \PSTMG \TSTMG
|
||||
\classe{Troisième}
|
||||
\date{21 septembre 2016}
|
||||
%\duree{1 heure}
|
||||
%\sujet{}
|
||||
% DS DSCorr DM DMCorr Corr
|
||||
\thispagestyle{empty}
|
||||
|
||||
%\printanswers
|
||||
|
||||
\newcommand{\lexo}{%
|
||||
Faire les calculs suivants en détaillant des étapes.
|
||||
\ifprintanswers
|
||||
\else
|
||||
\begin{multicols}{2}
|
||||
\fi
|
||||
\begin{parts}
|
||||
|
||||
\part $4 + 9 \times ( -7 )$
|
||||
\begin{solution}
|
||||
\begin{align*}
|
||||
4 + 9 \times ( -7 )=4 - 63=-59
|
||||
\end{align*}
|
||||
\end{solution}
|
||||
|
||||
\part $-7 - ( -4 \times ( -3 ) )$
|
||||
\begin{solution}
|
||||
\begin{align*}
|
||||
-7 - ( -4 \times ( -3 ) )=-7 - 12=-19
|
||||
\end{align*}
|
||||
\end{solution}
|
||||
|
||||
\part $-5 \times ( -5 ) - 2 \times ( -4 )$
|
||||
\begin{solution}
|
||||
\begin{align*}
|
||||
-5 \times ( -5 ) - 2 \times ( -4 )=25 + 8=33
|
||||
\end{align*}
|
||||
\end{solution}
|
||||
|
||||
\part $( -9 + 9 ) \times 8 + 8$
|
||||
\begin{solution}
|
||||
\begin{align*}
|
||||
( -9 + 9 ) \times 8 + 8=0 \times 8 + 8=0 + 8=8
|
||||
\end{align*}
|
||||
\end{solution}
|
||||
|
||||
\part $7 ( -1 - 9 ) \times ( -8 )$
|
||||
\begin{solution}
|
||||
\begin{align*}
|
||||
7 ( -1 - 9 ) \times ( -8 )=7 \times ( -10 ) \times ( -8 )=-70 \times ( -8 )=560
|
||||
\end{align*}
|
||||
\end{solution}
|
||||
|
||||
\part $2 ( -3 - 10 ) - 6$
|
||||
\begin{solution}
|
||||
\begin{align*}
|
||||
2 ( -3 - 10 ) - 6=2 \times ( -13 ) - 6=-26 - 6=-32
|
||||
\end{align*}
|
||||
\end{solution}
|
||||
\end{parts}
|
||||
\ifprintanswers
|
||||
\else
|
||||
\end{multicols}
|
||||
\fi
|
||||
\vfill
|
||||
|
||||
}
|
||||
|
||||
\begin{document}
|
||||
|
||||
\begin{questions}
|
||||
|
||||
\question
|
||||
\lexo
|
||||
|
||||
\ifprintanswers
|
||||
\else
|
||||
\lexo
|
||||
\lexo
|
||||
\lexo
|
||||
\fi
|
||||
|
||||
\end{questions}
|
||||
|
||||
\end{document}
|
||||
|
||||
%%% Local Variables:
|
||||
%%% mode: latex
|
||||
%%% TeX-master: "master"
|
||||
%%% End:
|
BIN
3e/DM/DM_16_09_15/remediation/all_DM_16_09_15_remediation.pdf
Normal file
BIN
3e/DM/DM_16_09_15/remediation/all_DM_16_09_15_remediation.pdf
Normal file
Binary file not shown.
BIN
3e/DM/DM_16_09_15/remediation/corr_DM_16_09_15_remediation.pdf
Normal file
BIN
3e/DM/DM_16_09_15/remediation/corr_DM_16_09_15_remediation.pdf
Normal file
Binary file not shown.
@@ -0,0 +1,97 @@
|
||||
\documentclass[a5paper,12pt, table]{/media/documents/Cours/Prof/Enseignements/2016-2017/tools/style/classExo}
|
||||
\usepackage{/media/documents/Cours/Prof/Enseignements/2016-2017/theme}
|
||||
\geometry{left=10mm,right=10mm, top=10mm, bottom=10mm}
|
||||
|
||||
% Title Page
|
||||
\titre{Remediation priorité calculs}
|
||||
% \seconde \premiereS \PSTMG \TSTMG
|
||||
\classe{Troisième}
|
||||
\date{21 septembre 2016}
|
||||
%\duree{1 heure}
|
||||
%\sujet{}
|
||||
% DS DSCorr DM DMCorr Corr
|
||||
\thispagestyle{empty}
|
||||
|
||||
%\printanswers
|
||||
|
||||
\newcommand{\lexo}{%
|
||||
Faire les calculs suivants en détaillant des étapes.
|
||||
\ifprintanswers
|
||||
\else
|
||||
\begin{multicols}{2}
|
||||
\fi
|
||||
\begin{parts}
|
||||
\Block{set e = Expression.random("{a} + {b}*{c}")}
|
||||
\part $\Var{e}$
|
||||
\begin{solution}
|
||||
\begin{align*}
|
||||
\Var{e.simplify().explain() | join("=")}
|
||||
\end{align*}
|
||||
\end{solution}
|
||||
\Block{set e = Expression.random("{a} - {b}*{c}")}
|
||||
\part $\Var{e}$
|
||||
\begin{solution}
|
||||
\begin{align*}
|
||||
\Var{e.simplify().explain() | join("=")}
|
||||
\end{align*}
|
||||
\end{solution}
|
||||
\Block{set e = Expression.random("{a} * {b} + {c} * {d}")}
|
||||
\part $\Var{e}$
|
||||
\begin{solution}
|
||||
\begin{align*}
|
||||
\Var{e.simplify().explain() | join("=")}
|
||||
\end{align*}
|
||||
\end{solution}
|
||||
\Block{set e = Expression.random("({a} + {b})*{c} + {d}")}
|
||||
\part $\Var{e}$
|
||||
\begin{solution}
|
||||
\begin{align*}
|
||||
\Var{e.simplify().explain() | join("=")}
|
||||
\end{align*}
|
||||
\end{solution}
|
||||
\Block{set e = Expression.random("{a} * ({b} + {c}) * {d}")}
|
||||
\part $\Var{e}$
|
||||
\begin{solution}
|
||||
\begin{align*}
|
||||
\Var{e.simplify().explain() | join("=")}
|
||||
\end{align*}
|
||||
\end{solution}
|
||||
\Block{set e = Expression.random("{c}({a} - {b}) + {d}")}
|
||||
\part $\Var{e}$
|
||||
\begin{solution}
|
||||
\begin{align*}
|
||||
\Var{e.simplify().explain() | join("=")}
|
||||
\end{align*}
|
||||
\end{solution}
|
||||
\end{parts}
|
||||
\ifprintanswers
|
||||
\else
|
||||
\end{multicols}
|
||||
\fi
|
||||
\vfill
|
||||
|
||||
}
|
||||
|
||||
\begin{document}
|
||||
|
||||
\begin{questions}
|
||||
|
||||
\question
|
||||
\lexo
|
||||
|
||||
\ifprintanswers
|
||||
\else
|
||||
\lexo
|
||||
\lexo
|
||||
\lexo
|
||||
\fi
|
||||
|
||||
\end{questions}
|
||||
|
||||
\end{document}
|
||||
|
||||
%%% Local Variables:
|
||||
%%% mode: latex
|
||||
%%% TeX-master: "master"
|
||||
%%% End:
|
||||
|
79
3e/DM/DM_16_09_15/tpl_DM_16_09_15.tex
Normal file
79
3e/DM/DM_16_09_15/tpl_DM_16_09_15.tex
Normal file
@@ -0,0 +1,79 @@
|
||||
\documentclass[a5paper,12pt, table]{/media/documents/Cours/Prof/Enseignements/2016-2017/tools/style/classDS}
|
||||
\usepackage{/media/documents/Cours/Prof/Enseignements/2016-2017/theme}
|
||||
\geometry{left=10mm,right=10mm, top=10mm, bottom=10mm}
|
||||
|
||||
% Title Page
|
||||
\titre{1}
|
||||
% \seconde \premiereS \PSTMG \TSTMG
|
||||
\classe{Troisième}
|
||||
\date{15 septembre 2016}
|
||||
%\duree{1 heure}
|
||||
\sujet{\Var{infos.num}}
|
||||
% DS DSCorr DM DMCorr Corr
|
||||
\typedoc{DM}
|
||||
|
||||
%\printanswers
|
||||
|
||||
\begin{document}
|
||||
|
||||
\maketitle
|
||||
|
||||
Vous devez rendre le sujet avec la copie.
|
||||
|
||||
\begin{questions}
|
||||
|
||||
\question
|
||||
Faire les calculs suivants en détaillant des étapes.
|
||||
\begin{multicols}{2}
|
||||
\begin{parts}
|
||||
\Block{set e = Expression.random("{a} + {b}*{c}")}
|
||||
\part $\Var{e}$
|
||||
\Block{set e = Expression.random("{a} - {b}*{c}")}
|
||||
\part $\Var{e}$
|
||||
\Block{set e = Expression.random("{a} * {b} + {c} * {d}")}
|
||||
\part $\Var{e}$
|
||||
\Block{set e = Expression.random("({a} + {b})*{c} + {d}")}
|
||||
\part $\Var{e}$
|
||||
\Block{set e = Expression.random("{a} * ({b} + {c}) * {d}")}
|
||||
\part $\Var{e}$
|
||||
\Block{set e = Expression.random("{c}({a} - {b}) + {d}")}
|
||||
\part $\Var{e}$
|
||||
\end{parts}
|
||||
\end{multicols}
|
||||
\vfill
|
||||
|
||||
\question
|
||||
\Block{set a,b,c = random_str("{a*d},{b*d},{c}", conditions = ["{a} != {b}", "{a} > 1", "{b}>1", "{c} > 1", "{d}>1"]).split(',')}
|
||||
\Block{set total = int(a) + int(b) + int(c)}
|
||||
Dans un sac, il y a \Var{a} bonbons à la menthe, \Var{b} bonbons à la fraise et \Var{c} au chocolat. On choisit un bonbon au hasard dans ce sac.
|
||||
\begin{parts}
|
||||
\part Combien y a-t-il d'issues en tout?
|
||||
\begin{solution}
|
||||
Il y a \Var{total} bonbons.
|
||||
\end{solution}
|
||||
\part Calculer la probabilité de tirer un bonbon à la fraise.
|
||||
\begin{solution}
|
||||
$T($ tirer un bonbon à la fraise $) = \dfrac{\Var{a}}{\Var{total}}$
|
||||
\end{solution}
|
||||
\part Calculer la probabilité de tirer un bonbon qui n'est pas au chocolat.
|
||||
\begin{solution}
|
||||
\Block{set nonChoco = int(a) + int(b)}
|
||||
$T($ tirer un bonbon à la fraise $) = \dfrac{\Var{nonChoco}}{\Var{total}}$
|
||||
\end{solution}
|
||||
\part Calculer la probabilité de tirer un bonbon au réglisse.
|
||||
\begin{solution}
|
||||
$T($ tirer un bonbon au réglisse $) = \dfrac{0}{\Var{total}} = 0$
|
||||
\end{solution}
|
||||
\part Dans un autre sac, on place 25 bonbons à la menthe et 34 bonbons à la fraise. Lise préfère les bonbons à la menthe. Dans quel sac doit-elle tirer un bonbon pour avoir le plus de chance d'avoir un bonbon qu'elle préfère?
|
||||
\end{parts}
|
||||
|
||||
\vfill
|
||||
\end{questions}
|
||||
|
||||
\end{document}
|
||||
|
||||
%%% Local Variables:
|
||||
%%% mode: latex
|
||||
%%% TeX-master: "master"
|
||||
%%% End:
|
||||
|
147
3e/DM/DM_16_11_14/308/01_DM_15_11_12_308.tex
Normal file
147
3e/DM/DM_16_11_14/308/01_DM_15_11_12_308.tex
Normal file
@@ -0,0 +1,147 @@
|
||||
\documentclass[a5paper,12pt, table]{/media/documents/Cours/Prof/Enseignements/2016-2017/tools/style/classDS}
|
||||
\usepackage{/media/documents/Cours/Prof/Enseignements/2016-2017/theme}
|
||||
%\geometry{left=10mm,right=10mm, top=10mm, bottom=10mm}
|
||||
|
||||
% Title Page
|
||||
\titre{1}
|
||||
% \seconde \premiereS \PSTMG \TSTMG
|
||||
\classe{Troisième}
|
||||
\date{lundi 14 novembre 2016}
|
||||
%\duree{1 heure}
|
||||
\sujet{01}
|
||||
% DS DSCorr DM DMCorr Corr
|
||||
\typedoc{DM}
|
||||
|
||||
\geometry{left=10mm,right=10mm, bottom= 10mm, top=10mm}
|
||||
%\printanswers
|
||||
|
||||
\begin{document}
|
||||
|
||||
\maketitle
|
||||
|
||||
\vspace{-1cm}
|
||||
Vous devez rendre le sujet avec la copie.
|
||||
|
||||
\begin{questions}
|
||||
|
||||
\vfill
|
||||
\question
|
||||
\begin{parts}
|
||||
\part Compléter les pointillés pour qu'il y est bien égalité.
|
||||
\hspace{-1cm}
|
||||
\begin{center}
|
||||
%
|
||||
$\dfrac{3}{8} = \dfrac{\ldots}{32}$
|
||||
\hfill
|
||||
%
|
||||
$\dfrac{5}{8} = \dfrac{\ldots}{32}$
|
||||
\hfill
|
||||
%
|
||||
$\dfrac{\cdots}{18} = \dfrac{2}{9}$
|
||||
\hfill
|
||||
%
|
||||
$\dfrac{8}{6} = \dfrac{32}{\cdots}$
|
||||
\end{center}
|
||||
|
||||
\vfill
|
||||
\part Faire les calculs suivants en détaillant les étapes (penser à simplifier les fractions quand c'est possible).
|
||||
\begin{multicols}{3}
|
||||
\begin{subparts}
|
||||
|
||||
\subpart $A = \frac{ 10 }{ 8 } + \frac{ 6 }{ 8 }$
|
||||
\begin{solution}
|
||||
\begin{eqnarray*}
|
||||
A & = & \frac{ 10 }{ 8 } + \frac{ 6 }{ 8 } \\
|
||||
A & = & \frac{ 10 + 6 }{ 8 } \\
|
||||
A & = & \frac{ 16 }{ 8 } \\
|
||||
A & = & 2
|
||||
\end{eqnarray*}
|
||||
\end{solution}
|
||||
|
||||
\subpart $B = \frac{ 1 }{ 4 } + \frac{ -9 }{ 4 }$
|
||||
\begin{solution}
|
||||
\begin{eqnarray*}
|
||||
B & = & \frac{ 1 }{ 4 } + \frac{ -9 }{ 4 } \\
|
||||
B & = & \frac{ 1 - 9 }{ 4 } \\
|
||||
B & = & \frac{ -8 }{ 4 } \\
|
||||
B & = & -2
|
||||
\end{eqnarray*}
|
||||
\end{solution}
|
||||
|
||||
\subpart $C = \frac{ -4 }{ 4 } + \frac{ 2 }{ 40 }$
|
||||
\begin{solution}
|
||||
\begin{eqnarray*}
|
||||
C & = & \frac{ -4 }{ 4 } + \frac{ 2 }{ 40 } \\
|
||||
C & = & \frac{ -4 \times 10 }{ 4 \times 10 } + \frac{ 2 \times 1 }{ 40 \times 1 } \\
|
||||
C & = & \frac{ -40 }{ 40 } + \frac{ 2 }{ 40 } \\
|
||||
C & = & \frac{ -40 + 2 }{ 40 } \\
|
||||
C & = & \frac{ -38 }{ 40 } \\
|
||||
C & = & \frac{ -19 \times 2 }{ 20 \times 2 } \\
|
||||
C & = & \frac{ -19 }{ 20 }
|
||||
\end{eqnarray*}
|
||||
\end{solution}
|
||||
\end{subparts}
|
||||
\end{multicols}
|
||||
|
||||
\end{parts}
|
||||
|
||||
|
||||
\vfill
|
||||
|
||||
\question
|
||||
|
||||
|
||||
Dans la figure suivante, $(AB)$ et $(CD)$ sont parallèles, $AO = 15$, $OD = 19$, $CD = 15$ et $OB = 8$.
|
||||
|
||||
|
||||
\begin{minipage}{0.5\textwidth}
|
||||
\includegraphics[scale=0.4]{./fig/thales2}
|
||||
\end{minipage}
|
||||
\begin{minipage}{0.5\textwidth}
|
||||
Calculer les longueurs $AB$ et $BC$.
|
||||
\end{minipage}
|
||||
|
||||
|
||||
\vfill
|
||||
|
||||
\question
|
||||
|
||||
|
||||
|
||||
|
||||
|
||||
|
||||
Dans une urne, on a placé des boules colorées indiscernables au touché. Il y a 2 boules bleu, 5 boules jaunes, 4 boules vertes et 8 boules rouges.
|
||||
|
||||
\begin{parts}
|
||||
\part Quelle est la probabilité de tirer une boule bleu?
|
||||
\begin{solution}
|
||||
$\dfrac{2}{19} \approx 0.11$
|
||||
\end{solution}
|
||||
\part Quelle est la probabilté de tirer une boule jaune ou bleu?
|
||||
\begin{solution}
|
||||
$\dfrac{7}{19} \approx 0.37$
|
||||
\end{solution}
|
||||
\part A-t-on plus de chance de tirer une boule verte ou une boule rouge?
|
||||
\begin{solution}
|
||||
Boules vertes: $\dfrac{4}{19} \approx 0.21$
|
||||
|
||||
Boules rouges: $\dfrac{8}{19} \approx 0.42$
|
||||
|
||||
|
||||
Une boule rouge
|
||||
|
||||
\end{solution}
|
||||
\end{parts}
|
||||
|
||||
|
||||
|
||||
|
||||
\end{questions}
|
||||
|
||||
\end{document}
|
||||
|
||||
%%% Local Variables:
|
||||
%%% mode: latex
|
||||
%%% TeX-master: "master"
|
||||
%%% End:
|
143
3e/DM/DM_16_11_14/308/02_DM_15_11_12_308.tex
Normal file
143
3e/DM/DM_16_11_14/308/02_DM_15_11_12_308.tex
Normal file
@@ -0,0 +1,143 @@
|
||||
\documentclass[a5paper,12pt, table]{/media/documents/Cours/Prof/Enseignements/2016-2017/tools/style/classDS}
|
||||
\usepackage{/media/documents/Cours/Prof/Enseignements/2016-2017/theme}
|
||||
%\geometry{left=10mm,right=10mm, top=10mm, bottom=10mm}
|
||||
|
||||
% Title Page
|
||||
\titre{1}
|
||||
% \seconde \premiereS \PSTMG \TSTMG
|
||||
\classe{Troisième}
|
||||
\date{lundi 14 novembre 2016}
|
||||
%\duree{1 heure}
|
||||
\sujet{02}
|
||||
% DS DSCorr DM DMCorr Corr
|
||||
\typedoc{DM}
|
||||
|
||||
\geometry{left=10mm,right=10mm, bottom= 10mm, top=10mm}
|
||||
%\printanswers
|
||||
|
||||
\begin{document}
|
||||
|
||||
\maketitle
|
||||
|
||||
\vspace{-1cm}
|
||||
Vous devez rendre le sujet avec la copie.
|
||||
|
||||
\begin{questions}
|
||||
|
||||
\vfill
|
||||
\question
|
||||
\begin{parts}
|
||||
\part Compléter les pointillés pour qu'il y est bien égalité.
|
||||
\hspace{-1cm}
|
||||
\begin{center}
|
||||
%
|
||||
$\dfrac{2}{8} = \dfrac{\ldots}{40}$
|
||||
\hfill
|
||||
%
|
||||
$\dfrac{2}{5} = \dfrac{\ldots}{30}$
|
||||
\hfill
|
||||
%
|
||||
$\dfrac{\cdots}{24} = \dfrac{10}{3}$
|
||||
\hfill
|
||||
%
|
||||
$\dfrac{4}{8} = \dfrac{40}{\cdots}$
|
||||
\end{center}
|
||||
|
||||
\vfill
|
||||
\part Faire les calculs suivants en détaillant les étapes (penser à simplifier les fractions quand c'est possible).
|
||||
\begin{multicols}{3}
|
||||
\begin{subparts}
|
||||
|
||||
\subpart $A = \frac{ 7 }{ 5 } + \frac{ 1 }{ 5 }$
|
||||
\begin{solution}
|
||||
\begin{eqnarray*}
|
||||
A & = & \frac{ 7 }{ 5 } + \frac{ 1 }{ 5 } \\
|
||||
A & = & \frac{ 7 + 1 }{ 5 } \\
|
||||
A & = & \frac{ 8 }{ 5 }
|
||||
\end{eqnarray*}
|
||||
\end{solution}
|
||||
|
||||
\subpart $B = \frac{ -1 }{ 3 } + \frac{ -1 }{ 3 }$
|
||||
\begin{solution}
|
||||
\begin{eqnarray*}
|
||||
B & = & \frac{ -1 }{ 3 } + \frac{ -1 }{ 3 } \\
|
||||
B & = & \frac{ -1 - 1 }{ 3 } \\
|
||||
B & = & \frac{ -2 }{ 3 }
|
||||
\end{eqnarray*}
|
||||
\end{solution}
|
||||
|
||||
\subpart $C = \frac{ -3 }{ 10 } + \frac{ 8 }{ 30 }$
|
||||
\begin{solution}
|
||||
\begin{eqnarray*}
|
||||
C & = & \frac{ -3 }{ 10 } + \frac{ 8 }{ 30 } \\
|
||||
C & = & \frac{ -3 \times 3 }{ 10 \times 3 } + \frac{ 8 \times 1 }{ 30 \times 1 } \\
|
||||
C & = & \frac{ -9 }{ 30 } + \frac{ 8 }{ 30 } \\
|
||||
C & = & \frac{ -9 + 8 }{ 30 } \\
|
||||
C & = & \frac{ -1 }{ 30 }
|
||||
\end{eqnarray*}
|
||||
\end{solution}
|
||||
\end{subparts}
|
||||
\end{multicols}
|
||||
|
||||
\end{parts}
|
||||
|
||||
|
||||
\vfill
|
||||
|
||||
\question
|
||||
|
||||
|
||||
Dans la figure suivante, $(AB)$ et $(CD)$ sont parallèles, $AO = 5$, $OD = 7$, $CD = 8$ et $OB = 20$.
|
||||
|
||||
|
||||
\begin{minipage}{0.5\textwidth}
|
||||
\includegraphics[scale=0.4]{./fig/thales2}
|
||||
\end{minipage}
|
||||
\begin{minipage}{0.5\textwidth}
|
||||
Calculer les longueurs $AB$ et $BC$.
|
||||
\end{minipage}
|
||||
|
||||
|
||||
\vfill
|
||||
|
||||
\question
|
||||
|
||||
|
||||
|
||||
|
||||
|
||||
|
||||
Dans une urne, on a placé des boules colorées indiscernables au touché. Il y a 10 boules bleu, 8 boules jaunes, 5 boules vertes et 4 boules rouges.
|
||||
|
||||
\begin{parts}
|
||||
\part Quelle est la probabilité de tirer une boule bleu?
|
||||
\begin{solution}
|
||||
$\dfrac{10}{27} \approx 0.37$
|
||||
\end{solution}
|
||||
\part Quelle est la probabilté de tirer une boule jaune ou bleu?
|
||||
\begin{solution}
|
||||
$\dfrac{18}{27} \approx 0.67$
|
||||
\end{solution}
|
||||
\part A-t-on plus de chance de tirer une boule verte ou une boule rouge?
|
||||
\begin{solution}
|
||||
Boules vertes: $\dfrac{5}{27} \approx 0.19$
|
||||
|
||||
Boules rouges: $\dfrac{4}{27} \approx 0.15$
|
||||
|
||||
|
||||
Une boule verte
|
||||
|
||||
\end{solution}
|
||||
\end{parts}
|
||||
|
||||
|
||||
|
||||
|
||||
\end{questions}
|
||||
|
||||
\end{document}
|
||||
|
||||
%%% Local Variables:
|
||||
%%% mode: latex
|
||||
%%% TeX-master: "master"
|
||||
%%% End:
|
146
3e/DM/DM_16_11_14/308/03_DM_15_11_12_308.tex
Normal file
146
3e/DM/DM_16_11_14/308/03_DM_15_11_12_308.tex
Normal file
@@ -0,0 +1,146 @@
|
||||
\documentclass[a5paper,12pt, table]{/media/documents/Cours/Prof/Enseignements/2016-2017/tools/style/classDS}
|
||||
\usepackage{/media/documents/Cours/Prof/Enseignements/2016-2017/theme}
|
||||
%\geometry{left=10mm,right=10mm, top=10mm, bottom=10mm}
|
||||
|
||||
% Title Page
|
||||
\titre{1}
|
||||
% \seconde \premiereS \PSTMG \TSTMG
|
||||
\classe{Troisième}
|
||||
\date{lundi 14 novembre 2016}
|
||||
%\duree{1 heure}
|
||||
\sujet{03}
|
||||
% DS DSCorr DM DMCorr Corr
|
||||
\typedoc{DM}
|
||||
|
||||
\geometry{left=10mm,right=10mm, bottom= 10mm, top=10mm}
|
||||
%\printanswers
|
||||
|
||||
\begin{document}
|
||||
|
||||
\maketitle
|
||||
|
||||
\vspace{-1cm}
|
||||
Vous devez rendre le sujet avec la copie.
|
||||
|
||||
\begin{questions}
|
||||
|
||||
\vfill
|
||||
\question
|
||||
\begin{parts}
|
||||
\part Compléter les pointillés pour qu'il y est bien égalité.
|
||||
\hspace{-1cm}
|
||||
\begin{center}
|
||||
%
|
||||
$\dfrac{3}{10} = \dfrac{\ldots}{100}$
|
||||
\hfill
|
||||
%
|
||||
$\dfrac{9}{10} = \dfrac{\ldots}{80}$
|
||||
\hfill
|
||||
%
|
||||
$\dfrac{\cdots}{49} = \dfrac{3}{7}$
|
||||
\hfill
|
||||
%
|
||||
$\dfrac{8}{7} = \dfrac{80}{\cdots}$
|
||||
\end{center}
|
||||
|
||||
\vfill
|
||||
\part Faire les calculs suivants en détaillant les étapes (penser à simplifier les fractions quand c'est possible).
|
||||
\begin{multicols}{3}
|
||||
\begin{subparts}
|
||||
|
||||
\subpart $A = \frac{ 6 }{ 7 } + \frac{ 6 }{ 7 }$
|
||||
\begin{solution}
|
||||
\begin{eqnarray*}
|
||||
A & = & \frac{ 6 }{ 7 } + \frac{ 6 }{ 7 } \\
|
||||
A & = & \frac{ 6 + 6 }{ 7 } \\
|
||||
A & = & \frac{ 12 }{ 7 }
|
||||
\end{eqnarray*}
|
||||
\end{solution}
|
||||
|
||||
\subpart $B = \frac{ -5 }{ 6 } + \frac{ 3 }{ 6 }$
|
||||
\begin{solution}
|
||||
\begin{eqnarray*}
|
||||
B & = & \frac{ -5 }{ 6 } + \frac{ 3 }{ 6 } \\
|
||||
B & = & \frac{ -5 + 3 }{ 6 } \\
|
||||
B & = & \frac{ -2 }{ 6 } \\
|
||||
B & = & \frac{ -1 \times 2 }{ 3 \times 2 } \\
|
||||
B & = & \frac{ -1 }{ 3 }
|
||||
\end{eqnarray*}
|
||||
\end{solution}
|
||||
|
||||
\subpart $C = \frac{ -5 }{ 2 } + \frac{ 5 }{ 10 }$
|
||||
\begin{solution}
|
||||
\begin{eqnarray*}
|
||||
C & = & \frac{ -5 }{ 2 } + \frac{ 5 }{ 10 } \\
|
||||
C & = & \frac{ -5 \times 5 }{ 2 \times 5 } + \frac{ 5 \times 1 }{ 10 \times 1 } \\
|
||||
C & = & \frac{ -25 }{ 10 } + \frac{ 5 }{ 10 } \\
|
||||
C & = & \frac{ -25 + 5 }{ 10 } \\
|
||||
C & = & \frac{ -20 }{ 10 } \\
|
||||
C & = & -2
|
||||
\end{eqnarray*}
|
||||
\end{solution}
|
||||
\end{subparts}
|
||||
\end{multicols}
|
||||
|
||||
\end{parts}
|
||||
|
||||
|
||||
\vfill
|
||||
|
||||
\question
|
||||
|
||||
|
||||
Dans la figure suivante, $(AB)$ et $(CD)$ sont parallèles, $AO = 9$, $OD = 17$, $CD = 11$ et $OB = 5$.
|
||||
|
||||
|
||||
\begin{minipage}{0.5\textwidth}
|
||||
\includegraphics[scale=0.4]{./fig/thales1}
|
||||
\end{minipage}
|
||||
\begin{minipage}{0.5\textwidth}
|
||||
Calculer les longueurs $AB$ et $BC$.
|
||||
\end{minipage}
|
||||
|
||||
|
||||
\vfill
|
||||
|
||||
\question
|
||||
|
||||
|
||||
|
||||
|
||||
|
||||
|
||||
Dans une urne, on a placé des boules colorées indiscernables au touché. Il y a 7 boules bleu, 4 boules jaunes, 10 boules vertes et 3 boules rouges.
|
||||
|
||||
\begin{parts}
|
||||
\part Quelle est la probabilité de tirer une boule bleu?
|
||||
\begin{solution}
|
||||
$\dfrac{7}{24} \approx 0.29$
|
||||
\end{solution}
|
||||
\part Quelle est la probabilté de tirer une boule jaune ou bleu?
|
||||
\begin{solution}
|
||||
$\dfrac{11}{24} \approx 0.46$
|
||||
\end{solution}
|
||||
\part A-t-on plus de chance de tirer une boule verte ou une boule rouge?
|
||||
\begin{solution}
|
||||
Boules vertes: $\dfrac{10}{24} \approx 0.42$
|
||||
|
||||
Boules rouges: $\dfrac{3}{24} \approx 0.12$
|
||||
|
||||
|
||||
Une boule verte
|
||||
|
||||
\end{solution}
|
||||
\end{parts}
|
||||
|
||||
|
||||
|
||||
|
||||
\end{questions}
|
||||
|
||||
\end{document}
|
||||
|
||||
%%% Local Variables:
|
||||
%%% mode: latex
|
||||
%%% TeX-master: "master"
|
||||
%%% End:
|
145
3e/DM/DM_16_11_14/308/04_DM_15_11_12_308.tex
Normal file
145
3e/DM/DM_16_11_14/308/04_DM_15_11_12_308.tex
Normal file
@@ -0,0 +1,145 @@
|
||||
\documentclass[a5paper,12pt, table]{/media/documents/Cours/Prof/Enseignements/2016-2017/tools/style/classDS}
|
||||
\usepackage{/media/documents/Cours/Prof/Enseignements/2016-2017/theme}
|
||||
%\geometry{left=10mm,right=10mm, top=10mm, bottom=10mm}
|
||||
|
||||
% Title Page
|
||||
\titre{1}
|
||||
% \seconde \premiereS \PSTMG \TSTMG
|
||||
\classe{Troisième}
|
||||
\date{lundi 14 novembre 2016}
|
||||
%\duree{1 heure}
|
||||
\sujet{04}
|
||||
% DS DSCorr DM DMCorr Corr
|
||||
\typedoc{DM}
|
||||
|
||||
\geometry{left=10mm,right=10mm, bottom= 10mm, top=10mm}
|
||||
%\printanswers
|
||||
|
||||
\begin{document}
|
||||
|
||||
\maketitle
|
||||
|
||||
\vspace{-1cm}
|
||||
Vous devez rendre le sujet avec la copie.
|
||||
|
||||
\begin{questions}
|
||||
|
||||
\vfill
|
||||
\question
|
||||
\begin{parts}
|
||||
\part Compléter les pointillés pour qu'il y est bien égalité.
|
||||
\hspace{-1cm}
|
||||
\begin{center}
|
||||
%
|
||||
$\dfrac{4}{10} = \dfrac{\ldots}{100}$
|
||||
\hfill
|
||||
%
|
||||
$\dfrac{6}{3} = \dfrac{\ldots}{15}$
|
||||
\hfill
|
||||
%
|
||||
$\dfrac{\cdots}{20} = \dfrac{5}{2}$
|
||||
\hfill
|
||||
%
|
||||
$\dfrac{9}{4} = \dfrac{36}{\cdots}$
|
||||
\end{center}
|
||||
|
||||
\vfill
|
||||
\part Faire les calculs suivants en détaillant les étapes (penser à simplifier les fractions quand c'est possible).
|
||||
\begin{multicols}{3}
|
||||
\begin{subparts}
|
||||
|
||||
\subpart $A = \frac{ 10 }{ 2 } + \frac{ 5 }{ 2 }$
|
||||
\begin{solution}
|
||||
\begin{eqnarray*}
|
||||
A & = & \frac{ 10 }{ 2 } + \frac{ 5 }{ 2 } \\
|
||||
A & = & \frac{ 10 + 5 }{ 2 } \\
|
||||
A & = & \frac{ 15 }{ 2 }
|
||||
\end{eqnarray*}
|
||||
\end{solution}
|
||||
|
||||
\subpart $B = \frac{ -9 }{ 7 } + \frac{ 6 }{ 7 }$
|
||||
\begin{solution}
|
||||
\begin{eqnarray*}
|
||||
B & = & \frac{ -9 }{ 7 } + \frac{ 6 }{ 7 } \\
|
||||
B & = & \frac{ -9 + 6 }{ 7 } \\
|
||||
B & = & \frac{ -3 }{ 7 }
|
||||
\end{eqnarray*}
|
||||
\end{solution}
|
||||
|
||||
\subpart $C = \frac{ -10 }{ 8 } + \frac{ 2 }{ 48 }$
|
||||
\begin{solution}
|
||||
\begin{eqnarray*}
|
||||
C & = & \frac{ -10 }{ 8 } + \frac{ 2 }{ 48 } \\
|
||||
C & = & \frac{ -10 \times 6 }{ 8 \times 6 } + \frac{ 2 \times 1 }{ 48 \times 1 } \\
|
||||
C & = & \frac{ -60 }{ 48 } + \frac{ 2 }{ 48 } \\
|
||||
C & = & \frac{ -60 + 2 }{ 48 } \\
|
||||
C & = & \frac{ -58 }{ 48 } \\
|
||||
C & = & \frac{ -29 \times 2 }{ 24 \times 2 } \\
|
||||
C & = & \frac{ -29 }{ 24 }
|
||||
\end{eqnarray*}
|
||||
\end{solution}
|
||||
\end{subparts}
|
||||
\end{multicols}
|
||||
|
||||
\end{parts}
|
||||
|
||||
|
||||
\vfill
|
||||
|
||||
\question
|
||||
|
||||
|
||||
Dans la figure suivante, $(AB)$ et $(CD)$ sont parallèles, $AO = 7$, $OD = 14$, $CD = 15$ et $OB = 13$.
|
||||
|
||||
|
||||
\begin{minipage}{0.5\textwidth}
|
||||
\includegraphics[scale=0.4]{./fig/thales1}
|
||||
\end{minipage}
|
||||
\begin{minipage}{0.5\textwidth}
|
||||
Calculer les longueurs $AB$ et $BC$.
|
||||
\end{minipage}
|
||||
|
||||
|
||||
\vfill
|
||||
|
||||
\question
|
||||
|
||||
|
||||
|
||||
|
||||
|
||||
|
||||
Dans une urne, on a placé des boules colorées indiscernables au touché. Il y a 8 boules bleu, 7 boules jaunes, 7 boules vertes et 9 boules rouges.
|
||||
|
||||
\begin{parts}
|
||||
\part Quelle est la probabilité de tirer une boule bleu?
|
||||
\begin{solution}
|
||||
$\dfrac{8}{31} \approx 0.26$
|
||||
\end{solution}
|
||||
\part Quelle est la probabilté de tirer une boule jaune ou bleu?
|
||||
\begin{solution}
|
||||
$\dfrac{15}{31} \approx 0.48$
|
||||
\end{solution}
|
||||
\part A-t-on plus de chance de tirer une boule verte ou une boule rouge?
|
||||
\begin{solution}
|
||||
Boules vertes: $\dfrac{7}{31} \approx 0.23$
|
||||
|
||||
Boules rouges: $\dfrac{9}{31} \approx 0.29$
|
||||
|
||||
|
||||
Une boule rouge
|
||||
|
||||
\end{solution}
|
||||
\end{parts}
|
||||
|
||||
|
||||
|
||||
|
||||
\end{questions}
|
||||
|
||||
\end{document}
|
||||
|
||||
%%% Local Variables:
|
||||
%%% mode: latex
|
||||
%%% TeX-master: "master"
|
||||
%%% End:
|
144
3e/DM/DM_16_11_14/308/05_DM_15_11_12_308.tex
Normal file
144
3e/DM/DM_16_11_14/308/05_DM_15_11_12_308.tex
Normal file
@@ -0,0 +1,144 @@
|
||||
\documentclass[a5paper,12pt, table]{/media/documents/Cours/Prof/Enseignements/2016-2017/tools/style/classDS}
|
||||
\usepackage{/media/documents/Cours/Prof/Enseignements/2016-2017/theme}
|
||||
%\geometry{left=10mm,right=10mm, top=10mm, bottom=10mm}
|
||||
|
||||
% Title Page
|
||||
\titre{1}
|
||||
% \seconde \premiereS \PSTMG \TSTMG
|
||||
\classe{Troisième}
|
||||
\date{lundi 14 novembre 2016}
|
||||
%\duree{1 heure}
|
||||
\sujet{05}
|
||||
% DS DSCorr DM DMCorr Corr
|
||||
\typedoc{DM}
|
||||
|
||||
\geometry{left=10mm,right=10mm, bottom= 10mm, top=10mm}
|
||||
%\printanswers
|
||||
|
||||
\begin{document}
|
||||
|
||||
\maketitle
|
||||
|
||||
\vspace{-1cm}
|
||||
Vous devez rendre le sujet avec la copie.
|
||||
|
||||
\begin{questions}
|
||||
|
||||
\vfill
|
||||
\question
|
||||
\begin{parts}
|
||||
\part Compléter les pointillés pour qu'il y est bien égalité.
|
||||
\hspace{-1cm}
|
||||
\begin{center}
|
||||
%
|
||||
$\dfrac{4}{8} = \dfrac{\ldots}{24}$
|
||||
\hfill
|
||||
%
|
||||
$\dfrac{7}{10} = \dfrac{\ldots}{80}$
|
||||
\hfill
|
||||
%
|
||||
$\dfrac{\cdots}{80} = \dfrac{7}{10}$
|
||||
\hfill
|
||||
%
|
||||
$\dfrac{5}{3} = \dfrac{25}{\cdots}$
|
||||
\end{center}
|
||||
|
||||
\vfill
|
||||
\part Faire les calculs suivants en détaillant les étapes (penser à simplifier les fractions quand c'est possible).
|
||||
\begin{multicols}{3}
|
||||
\begin{subparts}
|
||||
|
||||
\subpart $A = \frac{ 5 }{ 7 } + \frac{ 5 }{ 7 }$
|
||||
\begin{solution}
|
||||
\begin{eqnarray*}
|
||||
A & = & \frac{ 5 }{ 7 } + \frac{ 5 }{ 7 } \\
|
||||
A & = & \frac{ 5 + 5 }{ 7 } \\
|
||||
A & = & \frac{ 10 }{ 7 }
|
||||
\end{eqnarray*}
|
||||
\end{solution}
|
||||
|
||||
\subpart $B = \frac{ 3 }{ 6 } + \frac{ 9 }{ 6 }$
|
||||
\begin{solution}
|
||||
\begin{eqnarray*}
|
||||
B & = & \frac{ 3 }{ 6 } + \frac{ 9 }{ 6 } \\
|
||||
B & = & \frac{ 3 + 9 }{ 6 } \\
|
||||
B & = & \frac{ 12 }{ 6 } \\
|
||||
B & = & 2
|
||||
\end{eqnarray*}
|
||||
\end{solution}
|
||||
|
||||
\subpart $C = \frac{ 8 }{ 3 } + \frac{ 7 }{ 18 }$
|
||||
\begin{solution}
|
||||
\begin{eqnarray*}
|
||||
C & = & \frac{ 8 }{ 3 } + \frac{ 7 }{ 18 } \\
|
||||
C & = & \frac{ 8 \times 6 }{ 3 \times 6 } + \frac{ 7 \times 1 }{ 18 \times 1 } \\
|
||||
C & = & \frac{ 48 }{ 18 } + \frac{ 7 }{ 18 } \\
|
||||
C & = & \frac{ 48 + 7 }{ 18 } \\
|
||||
C & = & \frac{ 55 }{ 18 }
|
||||
\end{eqnarray*}
|
||||
\end{solution}
|
||||
\end{subparts}
|
||||
\end{multicols}
|
||||
|
||||
\end{parts}
|
||||
|
||||
|
||||
\vfill
|
||||
|
||||
\question
|
||||
|
||||
|
||||
Dans la figure suivante, $(AB)$ et $(CD)$ sont parallèles, $AO = 2$, $OD = 6$, $CD = 17$ et $OB = 6$.
|
||||
|
||||
|
||||
\begin{minipage}{0.5\textwidth}
|
||||
\includegraphics[scale=0.4]{./fig/thales1}
|
||||
\end{minipage}
|
||||
\begin{minipage}{0.5\textwidth}
|
||||
Calculer les longueurs $AB$ et $BC$.
|
||||
\end{minipage}
|
||||
|
||||
|
||||
\vfill
|
||||
|
||||
\question
|
||||
|
||||
|
||||
|
||||
|
||||
|
||||
|
||||
Dans une urne, on a placé des boules colorées indiscernables au touché. Il y a 5 boules bleu, 7 boules jaunes, 6 boules vertes et 6 boules rouges.
|
||||
|
||||
\begin{parts}
|
||||
\part Quelle est la probabilité de tirer une boule bleu?
|
||||
\begin{solution}
|
||||
$\dfrac{5}{24} \approx 0.21$
|
||||
\end{solution}
|
||||
\part Quelle est la probabilté de tirer une boule jaune ou bleu?
|
||||
\begin{solution}
|
||||
$\dfrac{12}{24} \approx 0.5$
|
||||
\end{solution}
|
||||
\part A-t-on plus de chance de tirer une boule verte ou une boule rouge?
|
||||
\begin{solution}
|
||||
Boules vertes: $\dfrac{6}{24} \approx 0.25$
|
||||
|
||||
Boules rouges: $\dfrac{6}{24} \approx 0.25$
|
||||
|
||||
|
||||
Une boule rouge
|
||||
|
||||
\end{solution}
|
||||
\end{parts}
|
||||
|
||||
|
||||
|
||||
|
||||
\end{questions}
|
||||
|
||||
\end{document}
|
||||
|
||||
%%% Local Variables:
|
||||
%%% mode: latex
|
||||
%%% TeX-master: "master"
|
||||
%%% End:
|
147
3e/DM/DM_16_11_14/308/06_DM_15_11_12_308.tex
Normal file
147
3e/DM/DM_16_11_14/308/06_DM_15_11_12_308.tex
Normal file
@@ -0,0 +1,147 @@
|
||||
\documentclass[a5paper,12pt, table]{/media/documents/Cours/Prof/Enseignements/2016-2017/tools/style/classDS}
|
||||
\usepackage{/media/documents/Cours/Prof/Enseignements/2016-2017/theme}
|
||||
%\geometry{left=10mm,right=10mm, top=10mm, bottom=10mm}
|
||||
|
||||
% Title Page
|
||||
\titre{1}
|
||||
% \seconde \premiereS \PSTMG \TSTMG
|
||||
\classe{Troisième}
|
||||
\date{lundi 14 novembre 2016}
|
||||
%\duree{1 heure}
|
||||
\sujet{06}
|
||||
% DS DSCorr DM DMCorr Corr
|
||||
\typedoc{DM}
|
||||
|
||||
\geometry{left=10mm,right=10mm, bottom= 10mm, top=10mm}
|
||||
%\printanswers
|
||||
|
||||
\begin{document}
|
||||
|
||||
\maketitle
|
||||
|
||||
\vspace{-1cm}
|
||||
Vous devez rendre le sujet avec la copie.
|
||||
|
||||
\begin{questions}
|
||||
|
||||
\vfill
|
||||
\question
|
||||
\begin{parts}
|
||||
\part Compléter les pointillés pour qu'il y est bien égalité.
|
||||
\hspace{-1cm}
|
||||
\begin{center}
|
||||
%
|
||||
$\dfrac{6}{10} = \dfrac{\ldots}{80}$
|
||||
\hfill
|
||||
%
|
||||
$\dfrac{7}{2} = \dfrac{\ldots}{6}$
|
||||
\hfill
|
||||
%
|
||||
$\dfrac{\cdots}{81} = \dfrac{10}{9}$
|
||||
\hfill
|
||||
%
|
||||
$\dfrac{8}{10} = \dfrac{72}{\cdots}$
|
||||
\end{center}
|
||||
|
||||
\vfill
|
||||
\part Faire les calculs suivants en détaillant les étapes (penser à simplifier les fractions quand c'est possible).
|
||||
\begin{multicols}{3}
|
||||
\begin{subparts}
|
||||
|
||||
\subpart $A = \frac{ 9 }{ 10 } + \frac{ 3 }{ 10 }$
|
||||
\begin{solution}
|
||||
\begin{eqnarray*}
|
||||
A & = & \frac{ 9 }{ 10 } + \frac{ 3 }{ 10 } \\
|
||||
A & = & \frac{ 9 + 3 }{ 10 } \\
|
||||
A & = & \frac{ 12 }{ 10 } \\
|
||||
A & = & \frac{ 6 \times 2 }{ 5 \times 2 } \\
|
||||
A & = & \frac{ 6 }{ 5 }
|
||||
\end{eqnarray*}
|
||||
\end{solution}
|
||||
|
||||
\subpart $B = \frac{ -7 }{ 6 } + \frac{ -7 }{ 6 }$
|
||||
\begin{solution}
|
||||
\begin{eqnarray*}
|
||||
B & = & \frac{ -7 }{ 6 } + \frac{ -7 }{ 6 } \\
|
||||
B & = & \frac{ -7 - 7 }{ 6 } \\
|
||||
B & = & \frac{ -14 }{ 6 } \\
|
||||
B & = & \frac{ -7 \times 2 }{ 3 \times 2 } \\
|
||||
B & = & \frac{ -7 }{ 3 }
|
||||
\end{eqnarray*}
|
||||
\end{solution}
|
||||
|
||||
\subpart $C = \frac{ 1 }{ 10 } + \frac{ 5 }{ 20 }$
|
||||
\begin{solution}
|
||||
\begin{eqnarray*}
|
||||
C & = & \frac{ 1 }{ 10 } + \frac{ 5 }{ 20 } \\
|
||||
C & = & \frac{ 1 \times 2 }{ 10 \times 2 } + \frac{ 5 \times 1 }{ 20 \times 1 } \\
|
||||
C & = & \frac{ 2 }{ 20 } + \frac{ 5 }{ 20 } \\
|
||||
C & = & \frac{ 2 + 5 }{ 20 } \\
|
||||
C & = & \frac{ 7 }{ 20 }
|
||||
\end{eqnarray*}
|
||||
\end{solution}
|
||||
\end{subparts}
|
||||
\end{multicols}
|
||||
|
||||
\end{parts}
|
||||
|
||||
|
||||
\vfill
|
||||
|
||||
\question
|
||||
|
||||
|
||||
Dans la figure suivante, $(AB)$ et $(CD)$ sont parallèles, $AO = 7$, $OD = 11$, $CD = 13$ et $OB = 4$.
|
||||
|
||||
|
||||
\begin{minipage}{0.5\textwidth}
|
||||
\includegraphics[scale=0.4]{./fig/thales1}
|
||||
\end{minipage}
|
||||
\begin{minipage}{0.5\textwidth}
|
||||
Calculer les longueurs $AB$ et $BC$.
|
||||
\end{minipage}
|
||||
|
||||
|
||||
\vfill
|
||||
|
||||
\question
|
||||
|
||||
|
||||
|
||||
|
||||
|
||||
|
||||
Dans une urne, on a placé des boules colorées indiscernables au touché. Il y a 3 boules bleu, 3 boules jaunes, 7 boules vertes et 7 boules rouges.
|
||||
|
||||
\begin{parts}
|
||||
\part Quelle est la probabilité de tirer une boule bleu?
|
||||
\begin{solution}
|
||||
$\dfrac{3}{20} \approx 0.15$
|
||||
\end{solution}
|
||||
\part Quelle est la probabilté de tirer une boule jaune ou bleu?
|
||||
\begin{solution}
|
||||
$\dfrac{6}{20} \approx 0.3$
|
||||
\end{solution}
|
||||
\part A-t-on plus de chance de tirer une boule verte ou une boule rouge?
|
||||
\begin{solution}
|
||||
Boules vertes: $\dfrac{7}{20} \approx 0.35$
|
||||
|
||||
Boules rouges: $\dfrac{7}{20} \approx 0.35$
|
||||
|
||||
|
||||
Une boule rouge
|
||||
|
||||
\end{solution}
|
||||
\end{parts}
|
||||
|
||||
|
||||
|
||||
|
||||
\end{questions}
|
||||
|
||||
\end{document}
|
||||
|
||||
%%% Local Variables:
|
||||
%%% mode: latex
|
||||
%%% TeX-master: "master"
|
||||
%%% End:
|
147
3e/DM/DM_16_11_14/308/07_DM_15_11_12_308.tex
Normal file
147
3e/DM/DM_16_11_14/308/07_DM_15_11_12_308.tex
Normal file
@@ -0,0 +1,147 @@
|
||||
\documentclass[a5paper,12pt, table]{/media/documents/Cours/Prof/Enseignements/2016-2017/tools/style/classDS}
|
||||
\usepackage{/media/documents/Cours/Prof/Enseignements/2016-2017/theme}
|
||||
%\geometry{left=10mm,right=10mm, top=10mm, bottom=10mm}
|
||||
|
||||
% Title Page
|
||||
\titre{1}
|
||||
% \seconde \premiereS \PSTMG \TSTMG
|
||||
\classe{Troisième}
|
||||
\date{lundi 14 novembre 2016}
|
||||
%\duree{1 heure}
|
||||
\sujet{07}
|
||||
% DS DSCorr DM DMCorr Corr
|
||||
\typedoc{DM}
|
||||
|
||||
\geometry{left=10mm,right=10mm, bottom= 10mm, top=10mm}
|
||||
%\printanswers
|
||||
|
||||
\begin{document}
|
||||
|
||||
\maketitle
|
||||
|
||||
\vspace{-1cm}
|
||||
Vous devez rendre le sujet avec la copie.
|
||||
|
||||
\begin{questions}
|
||||
|
||||
\vfill
|
||||
\question
|
||||
\begin{parts}
|
||||
\part Compléter les pointillés pour qu'il y est bien égalité.
|
||||
\hspace{-1cm}
|
||||
\begin{center}
|
||||
%
|
||||
$\dfrac{7}{6} = \dfrac{\ldots}{24}$
|
||||
\hfill
|
||||
%
|
||||
$\dfrac{6}{9} = \dfrac{\ldots}{54}$
|
||||
\hfill
|
||||
%
|
||||
$\dfrac{\cdots}{24} = \dfrac{7}{8}$
|
||||
\hfill
|
||||
%
|
||||
$\dfrac{9}{10} = \dfrac{72}{\cdots}$
|
||||
\end{center}
|
||||
|
||||
\vfill
|
||||
\part Faire les calculs suivants en détaillant les étapes (penser à simplifier les fractions quand c'est possible).
|
||||
\begin{multicols}{3}
|
||||
\begin{subparts}
|
||||
|
||||
\subpart $A = \frac{ 9 }{ 5 } + \frac{ 1 }{ 5 }$
|
||||
\begin{solution}
|
||||
\begin{eqnarray*}
|
||||
A & = & \frac{ 9 }{ 5 } + \frac{ 1 }{ 5 } \\
|
||||
A & = & \frac{ 9 + 1 }{ 5 } \\
|
||||
A & = & \frac{ 10 }{ 5 } \\
|
||||
A & = & 2
|
||||
\end{eqnarray*}
|
||||
\end{solution}
|
||||
|
||||
\subpart $B = \frac{ 2 }{ 7 } + \frac{ -2 }{ 7 }$
|
||||
\begin{solution}
|
||||
\begin{eqnarray*}
|
||||
B & = & \frac{ 2 }{ 7 } + \frac{ -2 }{ 7 } \\
|
||||
B & = & \frac{ 2 - 2 }{ 7 } \\
|
||||
B & = & \frac{ 0 }{ 7 } \\
|
||||
B & = & 0
|
||||
\end{eqnarray*}
|
||||
\end{solution}
|
||||
|
||||
\subpart $C = \frac{ -4 }{ 9 } + \frac{ 6 }{ 72 }$
|
||||
\begin{solution}
|
||||
\begin{eqnarray*}
|
||||
C & = & \frac{ -4 }{ 9 } + \frac{ 6 }{ 72 } \\
|
||||
C & = & \frac{ -4 \times 8 }{ 9 \times 8 } + \frac{ 6 \times 1 }{ 72 \times 1 } \\
|
||||
C & = & \frac{ -32 }{ 72 } + \frac{ 6 }{ 72 } \\
|
||||
C & = & \frac{ -32 + 6 }{ 72 } \\
|
||||
C & = & \frac{ -26 }{ 72 } \\
|
||||
C & = & \frac{ -13 \times 2 }{ 36 \times 2 } \\
|
||||
C & = & \frac{ -13 }{ 36 }
|
||||
\end{eqnarray*}
|
||||
\end{solution}
|
||||
\end{subparts}
|
||||
\end{multicols}
|
||||
|
||||
\end{parts}
|
||||
|
||||
|
||||
\vfill
|
||||
|
||||
\question
|
||||
|
||||
|
||||
Dans la figure suivante, $(AB)$ et $(CD)$ sont parallèles, $AO = 4$, $OD = 8$, $CD = 8$ et $OB = 11$.
|
||||
|
||||
|
||||
\begin{minipage}{0.5\textwidth}
|
||||
\includegraphics[scale=0.4]{./fig/thales2}
|
||||
\end{minipage}
|
||||
\begin{minipage}{0.5\textwidth}
|
||||
Calculer les longueurs $AB$ et $BC$.
|
||||
\end{minipage}
|
||||
|
||||
|
||||
\vfill
|
||||
|
||||
\question
|
||||
|
||||
|
||||
|
||||
|
||||
|
||||
|
||||
Dans une urne, on a placé des boules colorées indiscernables au touché. Il y a 3 boules bleu, 10 boules jaunes, 3 boules vertes et 8 boules rouges.
|
||||
|
||||
\begin{parts}
|
||||
\part Quelle est la probabilité de tirer une boule bleu?
|
||||
\begin{solution}
|
||||
$\dfrac{3}{24} \approx 0.12$
|
||||
\end{solution}
|
||||
\part Quelle est la probabilté de tirer une boule jaune ou bleu?
|
||||
\begin{solution}
|
||||
$\dfrac{13}{24} \approx 0.54$
|
||||
\end{solution}
|
||||
\part A-t-on plus de chance de tirer une boule verte ou une boule rouge?
|
||||
\begin{solution}
|
||||
Boules vertes: $\dfrac{3}{24} \approx 0.12$
|
||||
|
||||
Boules rouges: $\dfrac{8}{24} \approx 0.33$
|
||||
|
||||
|
||||
Une boule rouge
|
||||
|
||||
\end{solution}
|
||||
\end{parts}
|
||||
|
||||
|
||||
|
||||
|
||||
\end{questions}
|
||||
|
||||
\end{document}
|
||||
|
||||
%%% Local Variables:
|
||||
%%% mode: latex
|
||||
%%% TeX-master: "master"
|
||||
%%% End:
|
145
3e/DM/DM_16_11_14/308/08_DM_15_11_12_308.tex
Normal file
145
3e/DM/DM_16_11_14/308/08_DM_15_11_12_308.tex
Normal file
@@ -0,0 +1,145 @@
|
||||
\documentclass[a5paper,12pt, table]{/media/documents/Cours/Prof/Enseignements/2016-2017/tools/style/classDS}
|
||||
\usepackage{/media/documents/Cours/Prof/Enseignements/2016-2017/theme}
|
||||
%\geometry{left=10mm,right=10mm, top=10mm, bottom=10mm}
|
||||
|
||||
% Title Page
|
||||
\titre{1}
|
||||
% \seconde \premiereS \PSTMG \TSTMG
|
||||
\classe{Troisième}
|
||||
\date{lundi 14 novembre 2016}
|
||||
%\duree{1 heure}
|
||||
\sujet{08}
|
||||
% DS DSCorr DM DMCorr Corr
|
||||
\typedoc{DM}
|
||||
|
||||
\geometry{left=10mm,right=10mm, bottom= 10mm, top=10mm}
|
||||
%\printanswers
|
||||
|
||||
\begin{document}
|
||||
|
||||
\maketitle
|
||||
|
||||
\vspace{-1cm}
|
||||
Vous devez rendre le sujet avec la copie.
|
||||
|
||||
\begin{questions}
|
||||
|
||||
\vfill
|
||||
\question
|
||||
\begin{parts}
|
||||
\part Compléter les pointillés pour qu'il y est bien égalité.
|
||||
\hspace{-1cm}
|
||||
\begin{center}
|
||||
%
|
||||
$\dfrac{6}{3} = \dfrac{\ldots}{30}$
|
||||
\hfill
|
||||
%
|
||||
$\dfrac{3}{5} = \dfrac{\ldots}{30}$
|
||||
\hfill
|
||||
%
|
||||
$\dfrac{\cdots}{18} = \dfrac{7}{2}$
|
||||
\hfill
|
||||
%
|
||||
$\dfrac{9}{8} = \dfrac{36}{\cdots}$
|
||||
\end{center}
|
||||
|
||||
\vfill
|
||||
\part Faire les calculs suivants en détaillant les étapes (penser à simplifier les fractions quand c'est possible).
|
||||
\begin{multicols}{3}
|
||||
\begin{subparts}
|
||||
|
||||
\subpart $A = \frac{ 2 }{ 5 } + \frac{ 2 }{ 5 }$
|
||||
\begin{solution}
|
||||
\begin{eqnarray*}
|
||||
A & = & \frac{ 2 }{ 5 } + \frac{ 2 }{ 5 } \\
|
||||
A & = & \frac{ 2 + 2 }{ 5 } \\
|
||||
A & = & \frac{ 4 }{ 5 }
|
||||
\end{eqnarray*}
|
||||
\end{solution}
|
||||
|
||||
\subpart $B = \frac{ 9 }{ 3 } + \frac{ 8 }{ 3 }$
|
||||
\begin{solution}
|
||||
\begin{eqnarray*}
|
||||
B & = & \frac{ 9 }{ 3 } + \frac{ 8 }{ 3 } \\
|
||||
B & = & \frac{ 9 + 8 }{ 3 } \\
|
||||
B & = & \frac{ 17 }{ 3 }
|
||||
\end{eqnarray*}
|
||||
\end{solution}
|
||||
|
||||
\subpart $C = \frac{ -2 }{ 7 } + \frac{ 9 }{ 63 }$
|
||||
\begin{solution}
|
||||
\begin{eqnarray*}
|
||||
C & = & \frac{ -2 }{ 7 } + \frac{ 9 }{ 63 } \\
|
||||
C & = & \frac{ -2 \times 9 }{ 7 \times 9 } + \frac{ 9 \times 1 }{ 63 \times 1 } \\
|
||||
C & = & \frac{ -18 }{ 63 } + \frac{ 9 }{ 63 } \\
|
||||
C & = & \frac{ -18 + 9 }{ 63 } \\
|
||||
C & = & \frac{ -9 }{ 63 } \\
|
||||
C & = & \frac{ -1 \times 9 }{ 7 \times 9 } \\
|
||||
C & = & \frac{ -1 }{ 7 }
|
||||
\end{eqnarray*}
|
||||
\end{solution}
|
||||
\end{subparts}
|
||||
\end{multicols}
|
||||
|
||||
\end{parts}
|
||||
|
||||
|
||||
\vfill
|
||||
|
||||
\question
|
||||
|
||||
|
||||
Dans la figure suivante, $(AB)$ et $(CD)$ sont parallèles, $AO = 7$, $OD = 18$, $CD = 19$ et $OB = 18$.
|
||||
|
||||
|
||||
\begin{minipage}{0.5\textwidth}
|
||||
\includegraphics[scale=0.4]{./fig/thales1}
|
||||
\end{minipage}
|
||||
\begin{minipage}{0.5\textwidth}
|
||||
Calculer les longueurs $AB$ et $BC$.
|
||||
\end{minipage}
|
||||
|
||||
|
||||
\vfill
|
||||
|
||||
\question
|
||||
|
||||
|
||||
|
||||
|
||||
|
||||
|
||||
Dans une urne, on a placé des boules colorées indiscernables au touché. Il y a 8 boules bleu, 10 boules jaunes, 10 boules vertes et 7 boules rouges.
|
||||
|
||||
\begin{parts}
|
||||
\part Quelle est la probabilité de tirer une boule bleu?
|
||||
\begin{solution}
|
||||
$\dfrac{8}{35} \approx 0.23$
|
||||
\end{solution}
|
||||
\part Quelle est la probabilté de tirer une boule jaune ou bleu?
|
||||
\begin{solution}
|
||||
$\dfrac{18}{35} \approx 0.51$
|
||||
\end{solution}
|
||||
\part A-t-on plus de chance de tirer une boule verte ou une boule rouge?
|
||||
\begin{solution}
|
||||
Boules vertes: $\dfrac{10}{35} \approx 0.29$
|
||||
|
||||
Boules rouges: $\dfrac{7}{35} \approx 0.2$
|
||||
|
||||
|
||||
Une boule verte
|
||||
|
||||
\end{solution}
|
||||
\end{parts}
|
||||
|
||||
|
||||
|
||||
|
||||
\end{questions}
|
||||
|
||||
\end{document}
|
||||
|
||||
%%% Local Variables:
|
||||
%%% mode: latex
|
||||
%%% TeX-master: "master"
|
||||
%%% End:
|
145
3e/DM/DM_16_11_14/308/09_DM_15_11_12_308.tex
Normal file
145
3e/DM/DM_16_11_14/308/09_DM_15_11_12_308.tex
Normal file
@@ -0,0 +1,145 @@
|
||||
\documentclass[a5paper,12pt, table]{/media/documents/Cours/Prof/Enseignements/2016-2017/tools/style/classDS}
|
||||
\usepackage{/media/documents/Cours/Prof/Enseignements/2016-2017/theme}
|
||||
%\geometry{left=10mm,right=10mm, top=10mm, bottom=10mm}
|
||||
|
||||
% Title Page
|
||||
\titre{1}
|
||||
% \seconde \premiereS \PSTMG \TSTMG
|
||||
\classe{Troisième}
|
||||
\date{lundi 14 novembre 2016}
|
||||
%\duree{1 heure}
|
||||
\sujet{09}
|
||||
% DS DSCorr DM DMCorr Corr
|
||||
\typedoc{DM}
|
||||
|
||||
\geometry{left=10mm,right=10mm, bottom= 10mm, top=10mm}
|
||||
%\printanswers
|
||||
|
||||
\begin{document}
|
||||
|
||||
\maketitle
|
||||
|
||||
\vspace{-1cm}
|
||||
Vous devez rendre le sujet avec la copie.
|
||||
|
||||
\begin{questions}
|
||||
|
||||
\vfill
|
||||
\question
|
||||
\begin{parts}
|
||||
\part Compléter les pointillés pour qu'il y est bien égalité.
|
||||
\hspace{-1cm}
|
||||
\begin{center}
|
||||
%
|
||||
$\dfrac{2}{9} = \dfrac{\ldots}{54}$
|
||||
\hfill
|
||||
%
|
||||
$\dfrac{2}{6} = \dfrac{\ldots}{24}$
|
||||
\hfill
|
||||
%
|
||||
$\dfrac{\cdots}{30} = \dfrac{5}{10}$
|
||||
\hfill
|
||||
%
|
||||
$\dfrac{2}{4} = \dfrac{10}{\cdots}$
|
||||
\end{center}
|
||||
|
||||
\vfill
|
||||
\part Faire les calculs suivants en détaillant les étapes (penser à simplifier les fractions quand c'est possible).
|
||||
\begin{multicols}{3}
|
||||
\begin{subparts}
|
||||
|
||||
\subpart $A = \frac{ 6 }{ 3 } + \frac{ 1 }{ 3 }$
|
||||
\begin{solution}
|
||||
\begin{eqnarray*}
|
||||
A & = & \frac{ 6 }{ 3 } + \frac{ 1 }{ 3 } \\
|
||||
A & = & \frac{ 6 + 1 }{ 3 } \\
|
||||
A & = & \frac{ 7 }{ 3 }
|
||||
\end{eqnarray*}
|
||||
\end{solution}
|
||||
|
||||
\subpart $B = \frac{ -7 }{ 10 } + \frac{ -5 }{ 10 }$
|
||||
\begin{solution}
|
||||
\begin{eqnarray*}
|
||||
B & = & \frac{ -7 }{ 10 } + \frac{ -5 }{ 10 } \\
|
||||
B & = & \frac{ -7 - 5 }{ 10 } \\
|
||||
B & = & \frac{ -12 }{ 10 } \\
|
||||
B & = & \frac{ -6 \times 2 }{ 5 \times 2 } \\
|
||||
B & = & \frac{ -6 }{ 5 }
|
||||
\end{eqnarray*}
|
||||
\end{solution}
|
||||
|
||||
\subpart $C = \frac{ 2 }{ 2 } + \frac{ 9 }{ 20 }$
|
||||
\begin{solution}
|
||||
\begin{eqnarray*}
|
||||
C & = & \frac{ 2 }{ 2 } + \frac{ 9 }{ 20 } \\
|
||||
C & = & \frac{ 2 \times 10 }{ 2 \times 10 } + \frac{ 9 \times 1 }{ 20 \times 1 } \\
|
||||
C & = & \frac{ 20 }{ 20 } + \frac{ 9 }{ 20 } \\
|
||||
C & = & \frac{ 20 + 9 }{ 20 } \\
|
||||
C & = & \frac{ 29 }{ 20 }
|
||||
\end{eqnarray*}
|
||||
\end{solution}
|
||||
\end{subparts}
|
||||
\end{multicols}
|
||||
|
||||
\end{parts}
|
||||
|
||||
|
||||
\vfill
|
||||
|
||||
\question
|
||||
|
||||
|
||||
Dans la figure suivante, $(AB)$ et $(CD)$ sont parallèles, $AO = 6$, $OD = 9$, $CD = 4$ et $OB = 11$.
|
||||
|
||||
|
||||
\begin{minipage}{0.5\textwidth}
|
||||
\includegraphics[scale=0.4]{./fig/thales1}
|
||||
\end{minipage}
|
||||
\begin{minipage}{0.5\textwidth}
|
||||
Calculer les longueurs $AB$ et $BC$.
|
||||
\end{minipage}
|
||||
|
||||
|
||||
\vfill
|
||||
|
||||
\question
|
||||
|
||||
|
||||
|
||||
|
||||
|
||||
|
||||
Dans une urne, on a placé des boules colorées indiscernables au touché. Il y a 8 boules bleu, 6 boules jaunes, 9 boules vertes et 4 boules rouges.
|
||||
|
||||
\begin{parts}
|
||||
\part Quelle est la probabilité de tirer une boule bleu?
|
||||
\begin{solution}
|
||||
$\dfrac{8}{27} \approx 0.3$
|
||||
\end{solution}
|
||||
\part Quelle est la probabilté de tirer une boule jaune ou bleu?
|
||||
\begin{solution}
|
||||
$\dfrac{14}{27} \approx 0.52$
|
||||
\end{solution}
|
||||
\part A-t-on plus de chance de tirer une boule verte ou une boule rouge?
|
||||
\begin{solution}
|
||||
Boules vertes: $\dfrac{9}{27} \approx 0.33$
|
||||
|
||||
Boules rouges: $\dfrac{4}{27} \approx 0.15$
|
||||
|
||||
|
||||
Une boule verte
|
||||
|
||||
\end{solution}
|
||||
\end{parts}
|
||||
|
||||
|
||||
|
||||
|
||||
\end{questions}
|
||||
|
||||
\end{document}
|
||||
|
||||
%%% Local Variables:
|
||||
%%% mode: latex
|
||||
%%% TeX-master: "master"
|
||||
%%% End:
|
144
3e/DM/DM_16_11_14/308/10_DM_15_11_12_308.tex
Normal file
144
3e/DM/DM_16_11_14/308/10_DM_15_11_12_308.tex
Normal file
@@ -0,0 +1,144 @@
|
||||
\documentclass[a5paper,12pt, table]{/media/documents/Cours/Prof/Enseignements/2016-2017/tools/style/classDS}
|
||||
\usepackage{/media/documents/Cours/Prof/Enseignements/2016-2017/theme}
|
||||
%\geometry{left=10mm,right=10mm, top=10mm, bottom=10mm}
|
||||
|
||||
% Title Page
|
||||
\titre{1}
|
||||
% \seconde \premiereS \PSTMG \TSTMG
|
||||
\classe{Troisième}
|
||||
\date{lundi 14 novembre 2016}
|
||||
%\duree{1 heure}
|
||||
\sujet{10}
|
||||
% DS DSCorr DM DMCorr Corr
|
||||
\typedoc{DM}
|
||||
|
||||
\geometry{left=10mm,right=10mm, bottom= 10mm, top=10mm}
|
||||
%\printanswers
|
||||
|
||||
\begin{document}
|
||||
|
||||
\maketitle
|
||||
|
||||
\vspace{-1cm}
|
||||
Vous devez rendre le sujet avec la copie.
|
||||
|
||||
\begin{questions}
|
||||
|
||||
\vfill
|
||||
\question
|
||||
\begin{parts}
|
||||
\part Compléter les pointillés pour qu'il y est bien égalité.
|
||||
\hspace{-1cm}
|
||||
\begin{center}
|
||||
%
|
||||
$\dfrac{6}{7} = \dfrac{\ldots}{70}$
|
||||
\hfill
|
||||
%
|
||||
$\dfrac{6}{8} = \dfrac{\ldots}{16}$
|
||||
\hfill
|
||||
%
|
||||
$\dfrac{\cdots}{12} = \dfrac{2}{6}$
|
||||
\hfill
|
||||
%
|
||||
$\dfrac{7}{2} = \dfrac{42}{\cdots}$
|
||||
\end{center}
|
||||
|
||||
\vfill
|
||||
\part Faire les calculs suivants en détaillant les étapes (penser à simplifier les fractions quand c'est possible).
|
||||
\begin{multicols}{3}
|
||||
\begin{subparts}
|
||||
|
||||
\subpart $A = \frac{ 8 }{ 3 } + \frac{ 6 }{ 3 }$
|
||||
\begin{solution}
|
||||
\begin{eqnarray*}
|
||||
A & = & \frac{ 8 }{ 3 } + \frac{ 6 }{ 3 } \\
|
||||
A & = & \frac{ 8 + 6 }{ 3 } \\
|
||||
A & = & \frac{ 14 }{ 3 }
|
||||
\end{eqnarray*}
|
||||
\end{solution}
|
||||
|
||||
\subpart $B = \frac{ -4 }{ 3 } + \frac{ -5 }{ 3 }$
|
||||
\begin{solution}
|
||||
\begin{eqnarray*}
|
||||
B & = & \frac{ -4 }{ 3 } + \frac{ -5 }{ 3 } \\
|
||||
B & = & \frac{ -4 - 5 }{ 3 } \\
|
||||
B & = & \frac{ -9 }{ 3 } \\
|
||||
B & = & -3
|
||||
\end{eqnarray*}
|
||||
\end{solution}
|
||||
|
||||
\subpart $C = \frac{ -10 }{ 6 } + \frac{ 9 }{ 42 }$
|
||||
\begin{solution}
|
||||
\begin{eqnarray*}
|
||||
C & = & \frac{ -10 }{ 6 } + \frac{ 9 }{ 42 } \\
|
||||
C & = & \frac{ -10 \times 7 }{ 6 \times 7 } + \frac{ 9 \times 1 }{ 42 \times 1 } \\
|
||||
C & = & \frac{ -70 }{ 42 } + \frac{ 9 }{ 42 } \\
|
||||
C & = & \frac{ -70 + 9 }{ 42 } \\
|
||||
C & = & \frac{ -61 }{ 42 }
|
||||
\end{eqnarray*}
|
||||
\end{solution}
|
||||
\end{subparts}
|
||||
\end{multicols}
|
||||
|
||||
\end{parts}
|
||||
|
||||
|
||||
\vfill
|
||||
|
||||
\question
|
||||
|
||||
|
||||
Dans la figure suivante, $(AB)$ et $(CD)$ sont parallèles, $AO = 2$, $OD = 20$, $CD = 12$ et $OB = 16$.
|
||||
|
||||
|
||||
\begin{minipage}{0.5\textwidth}
|
||||
\includegraphics[scale=0.4]{./fig/thales2}
|
||||
\end{minipage}
|
||||
\begin{minipage}{0.5\textwidth}
|
||||
Calculer les longueurs $AB$ et $BC$.
|
||||
\end{minipage}
|
||||
|
||||
|
||||
\vfill
|
||||
|
||||
\question
|
||||
|
||||
|
||||
|
||||
|
||||
|
||||
|
||||
Dans une urne, on a placé des boules colorées indiscernables au touché. Il y a 2 boules bleu, 9 boules jaunes, 6 boules vertes et 2 boules rouges.
|
||||
|
||||
\begin{parts}
|
||||
\part Quelle est la probabilité de tirer une boule bleu?
|
||||
\begin{solution}
|
||||
$\dfrac{2}{19} \approx 0.11$
|
||||
\end{solution}
|
||||
\part Quelle est la probabilté de tirer une boule jaune ou bleu?
|
||||
\begin{solution}
|
||||
$\dfrac{11}{19} \approx 0.58$
|
||||
\end{solution}
|
||||
\part A-t-on plus de chance de tirer une boule verte ou une boule rouge?
|
||||
\begin{solution}
|
||||
Boules vertes: $\dfrac{6}{19} \approx 0.32$
|
||||
|
||||
Boules rouges: $\dfrac{2}{19} \approx 0.11$
|
||||
|
||||
|
||||
Une boule verte
|
||||
|
||||
\end{solution}
|
||||
\end{parts}
|
||||
|
||||
|
||||
|
||||
|
||||
\end{questions}
|
||||
|
||||
\end{document}
|
||||
|
||||
%%% Local Variables:
|
||||
%%% mode: latex
|
||||
%%% TeX-master: "master"
|
||||
%%% End:
|
147
3e/DM/DM_16_11_14/308/11_DM_15_11_12_308.tex
Normal file
147
3e/DM/DM_16_11_14/308/11_DM_15_11_12_308.tex
Normal file
@@ -0,0 +1,147 @@
|
||||
\documentclass[a5paper,12pt, table]{/media/documents/Cours/Prof/Enseignements/2016-2017/tools/style/classDS}
|
||||
\usepackage{/media/documents/Cours/Prof/Enseignements/2016-2017/theme}
|
||||
%\geometry{left=10mm,right=10mm, top=10mm, bottom=10mm}
|
||||
|
||||
% Title Page
|
||||
\titre{1}
|
||||
% \seconde \premiereS \PSTMG \TSTMG
|
||||
\classe{Troisième}
|
||||
\date{lundi 14 novembre 2016}
|
||||
%\duree{1 heure}
|
||||
\sujet{11}
|
||||
% DS DSCorr DM DMCorr Corr
|
||||
\typedoc{DM}
|
||||
|
||||
\geometry{left=10mm,right=10mm, bottom= 10mm, top=10mm}
|
||||
%\printanswers
|
||||
|
||||
\begin{document}
|
||||
|
||||
\maketitle
|
||||
|
||||
\vspace{-1cm}
|
||||
Vous devez rendre le sujet avec la copie.
|
||||
|
||||
\begin{questions}
|
||||
|
||||
\vfill
|
||||
\question
|
||||
\begin{parts}
|
||||
\part Compléter les pointillés pour qu'il y est bien égalité.
|
||||
\hspace{-1cm}
|
||||
\begin{center}
|
||||
%
|
||||
$\dfrac{3}{4} = \dfrac{\ldots}{28}$
|
||||
\hfill
|
||||
%
|
||||
$\dfrac{2}{9} = \dfrac{\ldots}{45}$
|
||||
\hfill
|
||||
%
|
||||
$\dfrac{\cdots}{54} = \dfrac{7}{9}$
|
||||
\hfill
|
||||
%
|
||||
$\dfrac{7}{5} = \dfrac{70}{\cdots}$
|
||||
\end{center}
|
||||
|
||||
\vfill
|
||||
\part Faire les calculs suivants en détaillant les étapes (penser à simplifier les fractions quand c'est possible).
|
||||
\begin{multicols}{3}
|
||||
\begin{subparts}
|
||||
|
||||
\subpart $A = \frac{ 8 }{ 2 } + \frac{ 8 }{ 2 }$
|
||||
\begin{solution}
|
||||
\begin{eqnarray*}
|
||||
A & = & \frac{ 8 }{ 2 } + \frac{ 8 }{ 2 } \\
|
||||
A & = & \frac{ 8 + 8 }{ 2 } \\
|
||||
A & = & \frac{ 16 }{ 2 } \\
|
||||
A & = & 8
|
||||
\end{eqnarray*}
|
||||
\end{solution}
|
||||
|
||||
\subpart $B = \frac{ 6 }{ 4 } + \frac{ -4 }{ 4 }$
|
||||
\begin{solution}
|
||||
\begin{eqnarray*}
|
||||
B & = & \frac{ 6 }{ 4 } + \frac{ -4 }{ 4 } \\
|
||||
B & = & \frac{ 6 - 4 }{ 4 } \\
|
||||
B & = & \frac{ 2 }{ 4 } \\
|
||||
B & = & \frac{ 1 \times 2 }{ 2 \times 2 } \\
|
||||
B & = & \frac{ 1 }{ 2 }
|
||||
\end{eqnarray*}
|
||||
\end{solution}
|
||||
|
||||
\subpart $C = \frac{ -1 }{ 6 } + \frac{ 8 }{ 48 }$
|
||||
\begin{solution}
|
||||
\begin{eqnarray*}
|
||||
C & = & \frac{ -1 }{ 6 } + \frac{ 8 }{ 48 } \\
|
||||
C & = & \frac{ -1 \times 8 }{ 6 \times 8 } + \frac{ 8 \times 1 }{ 48 \times 1 } \\
|
||||
C & = & \frac{ -8 }{ 48 } + \frac{ 8 }{ 48 } \\
|
||||
C & = & \frac{ -8 + 8 }{ 48 } \\
|
||||
C & = & \frac{ 0 }{ 48 } \\
|
||||
C & = & 0
|
||||
\end{eqnarray*}
|
||||
\end{solution}
|
||||
\end{subparts}
|
||||
\end{multicols}
|
||||
|
||||
\end{parts}
|
||||
|
||||
|
||||
\vfill
|
||||
|
||||
\question
|
||||
|
||||
|
||||
Dans la figure suivante, $(AB)$ et $(CD)$ sont parallèles, $AO = 9$, $OD = 19$, $CD = 3$ et $OB = 13$.
|
||||
|
||||
|
||||
\begin{minipage}{0.5\textwidth}
|
||||
\includegraphics[scale=0.4]{./fig/thales1}
|
||||
\end{minipage}
|
||||
\begin{minipage}{0.5\textwidth}
|
||||
Calculer les longueurs $AB$ et $BC$.
|
||||
\end{minipage}
|
||||
|
||||
|
||||
\vfill
|
||||
|
||||
\question
|
||||
|
||||
|
||||
|
||||
|
||||
|
||||
|
||||
Dans une urne, on a placé des boules colorées indiscernables au touché. Il y a 10 boules bleu, 8 boules jaunes, 8 boules vertes et 5 boules rouges.
|
||||
|
||||
\begin{parts}
|
||||
\part Quelle est la probabilité de tirer une boule bleu?
|
||||
\begin{solution}
|
||||
$\dfrac{10}{31} \approx 0.32$
|
||||
\end{solution}
|
||||
\part Quelle est la probabilté de tirer une boule jaune ou bleu?
|
||||
\begin{solution}
|
||||
$\dfrac{18}{31} \approx 0.58$
|
||||
\end{solution}
|
||||
\part A-t-on plus de chance de tirer une boule verte ou une boule rouge?
|
||||
\begin{solution}
|
||||
Boules vertes: $\dfrac{8}{31} \approx 0.26$
|
||||
|
||||
Boules rouges: $\dfrac{5}{31} \approx 0.16$
|
||||
|
||||
|
||||
Une boule verte
|
||||
|
||||
\end{solution}
|
||||
\end{parts}
|
||||
|
||||
|
||||
|
||||
|
||||
\end{questions}
|
||||
|
||||
\end{document}
|
||||
|
||||
%%% Local Variables:
|
||||
%%% mode: latex
|
||||
%%% TeX-master: "master"
|
||||
%%% End:
|
146
3e/DM/DM_16_11_14/308/12_DM_15_11_12_308.tex
Normal file
146
3e/DM/DM_16_11_14/308/12_DM_15_11_12_308.tex
Normal file
@@ -0,0 +1,146 @@
|
||||
\documentclass[a5paper,12pt, table]{/media/documents/Cours/Prof/Enseignements/2016-2017/tools/style/classDS}
|
||||
\usepackage{/media/documents/Cours/Prof/Enseignements/2016-2017/theme}
|
||||
%\geometry{left=10mm,right=10mm, top=10mm, bottom=10mm}
|
||||
|
||||
% Title Page
|
||||
\titre{1}
|
||||
% \seconde \premiereS \PSTMG \TSTMG
|
||||
\classe{Troisième}
|
||||
\date{lundi 14 novembre 2016}
|
||||
%\duree{1 heure}
|
||||
\sujet{12}
|
||||
% DS DSCorr DM DMCorr Corr
|
||||
\typedoc{DM}
|
||||
|
||||
\geometry{left=10mm,right=10mm, bottom= 10mm, top=10mm}
|
||||
%\printanswers
|
||||
|
||||
\begin{document}
|
||||
|
||||
\maketitle
|
||||
|
||||
\vspace{-1cm}
|
||||
Vous devez rendre le sujet avec la copie.
|
||||
|
||||
\begin{questions}
|
||||
|
||||
\vfill
|
||||
\question
|
||||
\begin{parts}
|
||||
\part Compléter les pointillés pour qu'il y est bien égalité.
|
||||
\hspace{-1cm}
|
||||
\begin{center}
|
||||
%
|
||||
$\dfrac{4}{9} = \dfrac{\ldots}{27}$
|
||||
\hfill
|
||||
%
|
||||
$\dfrac{7}{3} = \dfrac{\ldots}{18}$
|
||||
\hfill
|
||||
%
|
||||
$\dfrac{\cdots}{81} = \dfrac{8}{9}$
|
||||
\hfill
|
||||
%
|
||||
$\dfrac{2}{3} = \dfrac{10}{\cdots}$
|
||||
\end{center}
|
||||
|
||||
\vfill
|
||||
\part Faire les calculs suivants en détaillant les étapes (penser à simplifier les fractions quand c'est possible).
|
||||
\begin{multicols}{3}
|
||||
\begin{subparts}
|
||||
|
||||
\subpart $A = \frac{ 5 }{ 3 } + \frac{ 10 }{ 3 }$
|
||||
\begin{solution}
|
||||
\begin{eqnarray*}
|
||||
A & = & \frac{ 5 }{ 3 } + \frac{ 10 }{ 3 } \\
|
||||
A & = & \frac{ 5 + 10 }{ 3 } \\
|
||||
A & = & \frac{ 15 }{ 3 } \\
|
||||
A & = & 5
|
||||
\end{eqnarray*}
|
||||
\end{solution}
|
||||
|
||||
\subpart $B = \frac{ 7 }{ 7 } + \frac{ -6 }{ 7 }$
|
||||
\begin{solution}
|
||||
\begin{eqnarray*}
|
||||
B & = & \frac{ 7 }{ 7 } + \frac{ -6 }{ 7 } \\
|
||||
B & = & \frac{ 7 - 6 }{ 7 } \\
|
||||
B & = & \frac{ 1 }{ 7 }
|
||||
\end{eqnarray*}
|
||||
\end{solution}
|
||||
|
||||
\subpart $C = \frac{ -10 }{ 6 } + \frac{ 2 }{ 24 }$
|
||||
\begin{solution}
|
||||
\begin{eqnarray*}
|
||||
C & = & \frac{ -10 }{ 6 } + \frac{ 2 }{ 24 } \\
|
||||
C & = & \frac{ -10 \times 4 }{ 6 \times 4 } + \frac{ 2 \times 1 }{ 24 \times 1 } \\
|
||||
C & = & \frac{ -40 }{ 24 } + \frac{ 2 }{ 24 } \\
|
||||
C & = & \frac{ -40 + 2 }{ 24 } \\
|
||||
C & = & \frac{ -38 }{ 24 } \\
|
||||
C & = & \frac{ -19 \times 2 }{ 12 \times 2 } \\
|
||||
C & = & \frac{ -19 }{ 12 }
|
||||
\end{eqnarray*}
|
||||
\end{solution}
|
||||
\end{subparts}
|
||||
\end{multicols}
|
||||
|
||||
\end{parts}
|
||||
|
||||
|
||||
\vfill
|
||||
|
||||
\question
|
||||
|
||||
|
||||
Dans la figure suivante, $(AB)$ et $(CD)$ sont parallèles, $AO = 17$, $OD = 20$, $CD = 16$ et $OB = 19$.
|
||||
|
||||
|
||||
\begin{minipage}{0.5\textwidth}
|
||||
\includegraphics[scale=0.4]{./fig/thales1}
|
||||
\end{minipage}
|
||||
\begin{minipage}{0.5\textwidth}
|
||||
Calculer les longueurs $AB$ et $BC$.
|
||||
\end{minipage}
|
||||
|
||||
|
||||
\vfill
|
||||
|
||||
\question
|
||||
|
||||
|
||||
|
||||
|
||||
|
||||
|
||||
Dans une urne, on a placé des boules colorées indiscernables au touché. Il y a 6 boules bleu, 3 boules jaunes, 9 boules vertes et 7 boules rouges.
|
||||
|
||||
\begin{parts}
|
||||
\part Quelle est la probabilité de tirer une boule bleu?
|
||||
\begin{solution}
|
||||
$\dfrac{6}{25} \approx 0.24$
|
||||
\end{solution}
|
||||
\part Quelle est la probabilté de tirer une boule jaune ou bleu?
|
||||
\begin{solution}
|
||||
$\dfrac{9}{25} \approx 0.36$
|
||||
\end{solution}
|
||||
\part A-t-on plus de chance de tirer une boule verte ou une boule rouge?
|
||||
\begin{solution}
|
||||
Boules vertes: $\dfrac{9}{25} \approx 0.36$
|
||||
|
||||
Boules rouges: $\dfrac{7}{25} \approx 0.28$
|
||||
|
||||
|
||||
Une boule verte
|
||||
|
||||
\end{solution}
|
||||
\end{parts}
|
||||
|
||||
|
||||
|
||||
|
||||
\end{questions}
|
||||
|
||||
\end{document}
|
||||
|
||||
%%% Local Variables:
|
||||
%%% mode: latex
|
||||
%%% TeX-master: "master"
|
||||
%%% End:
|
143
3e/DM/DM_16_11_14/308/13_DM_15_11_12_308.tex
Normal file
143
3e/DM/DM_16_11_14/308/13_DM_15_11_12_308.tex
Normal file
@@ -0,0 +1,143 @@
|
||||
\documentclass[a5paper,12pt, table]{/media/documents/Cours/Prof/Enseignements/2016-2017/tools/style/classDS}
|
||||
\usepackage{/media/documents/Cours/Prof/Enseignements/2016-2017/theme}
|
||||
%\geometry{left=10mm,right=10mm, top=10mm, bottom=10mm}
|
||||
|
||||
% Title Page
|
||||
\titre{1}
|
||||
% \seconde \premiereS \PSTMG \TSTMG
|
||||
\classe{Troisième}
|
||||
\date{lundi 14 novembre 2016}
|
||||
%\duree{1 heure}
|
||||
\sujet{13}
|
||||
% DS DSCorr DM DMCorr Corr
|
||||
\typedoc{DM}
|
||||
|
||||
\geometry{left=10mm,right=10mm, bottom= 10mm, top=10mm}
|
||||
%\printanswers
|
||||
|
||||
\begin{document}
|
||||
|
||||
\maketitle
|
||||
|
||||
\vspace{-1cm}
|
||||
Vous devez rendre le sujet avec la copie.
|
||||
|
||||
\begin{questions}
|
||||
|
||||
\vfill
|
||||
\question
|
||||
\begin{parts}
|
||||
\part Compléter les pointillés pour qu'il y est bien égalité.
|
||||
\hspace{-1cm}
|
||||
\begin{center}
|
||||
%
|
||||
$\dfrac{3}{5} = \dfrac{\ldots}{40}$
|
||||
\hfill
|
||||
%
|
||||
$\dfrac{8}{9} = \dfrac{\ldots}{63}$
|
||||
\hfill
|
||||
%
|
||||
$\dfrac{\cdots}{36} = \dfrac{4}{9}$
|
||||
\hfill
|
||||
%
|
||||
$\dfrac{2}{6} = \dfrac{8}{\cdots}$
|
||||
\end{center}
|
||||
|
||||
\vfill
|
||||
\part Faire les calculs suivants en détaillant les étapes (penser à simplifier les fractions quand c'est possible).
|
||||
\begin{multicols}{3}
|
||||
\begin{subparts}
|
||||
|
||||
\subpart $A = \frac{ 4 }{ 8 } + \frac{ 1 }{ 8 }$
|
||||
\begin{solution}
|
||||
\begin{eqnarray*}
|
||||
A & = & \frac{ 4 }{ 8 } + \frac{ 1 }{ 8 } \\
|
||||
A & = & \frac{ 4 + 1 }{ 8 } \\
|
||||
A & = & \frac{ 5 }{ 8 }
|
||||
\end{eqnarray*}
|
||||
\end{solution}
|
||||
|
||||
\subpart $B = \frac{ -8 }{ 2 } + \frac{ 3 }{ 2 }$
|
||||
\begin{solution}
|
||||
\begin{eqnarray*}
|
||||
B & = & \frac{ -8 }{ 2 } + \frac{ 3 }{ 2 } \\
|
||||
B & = & \frac{ -8 + 3 }{ 2 } \\
|
||||
B & = & \frac{ -5 }{ 2 }
|
||||
\end{eqnarray*}
|
||||
\end{solution}
|
||||
|
||||
\subpart $C = \frac{ 5 }{ 4 } + \frac{ 9 }{ 32 }$
|
||||
\begin{solution}
|
||||
\begin{eqnarray*}
|
||||
C & = & \frac{ 5 }{ 4 } + \frac{ 9 }{ 32 } \\
|
||||
C & = & \frac{ 5 \times 8 }{ 4 \times 8 } + \frac{ 9 \times 1 }{ 32 \times 1 } \\
|
||||
C & = & \frac{ 40 }{ 32 } + \frac{ 9 }{ 32 } \\
|
||||
C & = & \frac{ 40 + 9 }{ 32 } \\
|
||||
C & = & \frac{ 49 }{ 32 }
|
||||
\end{eqnarray*}
|
||||
\end{solution}
|
||||
\end{subparts}
|
||||
\end{multicols}
|
||||
|
||||
\end{parts}
|
||||
|
||||
|
||||
\vfill
|
||||
|
||||
\question
|
||||
|
||||
|
||||
Dans la figure suivante, $(AB)$ et $(CD)$ sont parallèles, $AO = 11$, $OD = 20$, $CD = 14$ et $OB = 12$.
|
||||
|
||||
|
||||
\begin{minipage}{0.5\textwidth}
|
||||
\includegraphics[scale=0.4]{./fig/thales1}
|
||||
\end{minipage}
|
||||
\begin{minipage}{0.5\textwidth}
|
||||
Calculer les longueurs $AB$ et $BC$.
|
||||
\end{minipage}
|
||||
|
||||
|
||||
\vfill
|
||||
|
||||
\question
|
||||
|
||||
|
||||
|
||||
|
||||
|
||||
|
||||
Dans une urne, on a placé des boules colorées indiscernables au touché. Il y a 9 boules bleu, 5 boules jaunes, 2 boules vertes et 4 boules rouges.
|
||||
|
||||
\begin{parts}
|
||||
\part Quelle est la probabilité de tirer une boule bleu?
|
||||
\begin{solution}
|
||||
$\dfrac{9}{20} \approx 0.45$
|
||||
\end{solution}
|
||||
\part Quelle est la probabilté de tirer une boule jaune ou bleu?
|
||||
\begin{solution}
|
||||
$\dfrac{14}{20} \approx 0.7$
|
||||
\end{solution}
|
||||
\part A-t-on plus de chance de tirer une boule verte ou une boule rouge?
|
||||
\begin{solution}
|
||||
Boules vertes: $\dfrac{2}{20} \approx 0.1$
|
||||
|
||||
Boules rouges: $\dfrac{4}{20} \approx 0.2$
|
||||
|
||||
|
||||
Une boule rouge
|
||||
|
||||
\end{solution}
|
||||
\end{parts}
|
||||
|
||||
|
||||
|
||||
|
||||
\end{questions}
|
||||
|
||||
\end{document}
|
||||
|
||||
%%% Local Variables:
|
||||
%%% mode: latex
|
||||
%%% TeX-master: "master"
|
||||
%%% End:
|
143
3e/DM/DM_16_11_14/308/14_DM_15_11_12_308.tex
Normal file
143
3e/DM/DM_16_11_14/308/14_DM_15_11_12_308.tex
Normal file
@@ -0,0 +1,143 @@
|
||||
\documentclass[a5paper,12pt, table]{/media/documents/Cours/Prof/Enseignements/2016-2017/tools/style/classDS}
|
||||
\usepackage{/media/documents/Cours/Prof/Enseignements/2016-2017/theme}
|
||||
%\geometry{left=10mm,right=10mm, top=10mm, bottom=10mm}
|
||||
|
||||
% Title Page
|
||||
\titre{1}
|
||||
% \seconde \premiereS \PSTMG \TSTMG
|
||||
\classe{Troisième}
|
||||
\date{lundi 14 novembre 2016}
|
||||
%\duree{1 heure}
|
||||
\sujet{14}
|
||||
% DS DSCorr DM DMCorr Corr
|
||||
\typedoc{DM}
|
||||
|
||||
\geometry{left=10mm,right=10mm, bottom= 10mm, top=10mm}
|
||||
%\printanswers
|
||||
|
||||
\begin{document}
|
||||
|
||||
\maketitle
|
||||
|
||||
\vspace{-1cm}
|
||||
Vous devez rendre le sujet avec la copie.
|
||||
|
||||
\begin{questions}
|
||||
|
||||
\vfill
|
||||
\question
|
||||
\begin{parts}
|
||||
\part Compléter les pointillés pour qu'il y est bien égalité.
|
||||
\hspace{-1cm}
|
||||
\begin{center}
|
||||
%
|
||||
$\dfrac{2}{4} = \dfrac{\ldots}{24}$
|
||||
\hfill
|
||||
%
|
||||
$\dfrac{8}{2} = \dfrac{\ldots}{4}$
|
||||
\hfill
|
||||
%
|
||||
$\dfrac{\cdots}{18} = \dfrac{2}{6}$
|
||||
\hfill
|
||||
%
|
||||
$\dfrac{4}{2} = \dfrac{40}{\cdots}$
|
||||
\end{center}
|
||||
|
||||
\vfill
|
||||
\part Faire les calculs suivants en détaillant les étapes (penser à simplifier les fractions quand c'est possible).
|
||||
\begin{multicols}{3}
|
||||
\begin{subparts}
|
||||
|
||||
\subpart $A = \frac{ 9 }{ 9 } + \frac{ 10 }{ 9 }$
|
||||
\begin{solution}
|
||||
\begin{eqnarray*}
|
||||
A & = & \frac{ 9 }{ 9 } + \frac{ 10 }{ 9 } \\
|
||||
A & = & \frac{ 9 + 10 }{ 9 } \\
|
||||
A & = & \frac{ 19 }{ 9 }
|
||||
\end{eqnarray*}
|
||||
\end{solution}
|
||||
|
||||
\subpart $B = \frac{ -4 }{ 4 } + \frac{ 1 }{ 4 }$
|
||||
\begin{solution}
|
||||
\begin{eqnarray*}
|
||||
B & = & \frac{ -4 }{ 4 } + \frac{ 1 }{ 4 } \\
|
||||
B & = & \frac{ -4 + 1 }{ 4 } \\
|
||||
B & = & \frac{ -3 }{ 4 }
|
||||
\end{eqnarray*}
|
||||
\end{solution}
|
||||
|
||||
\subpart $C = \frac{ -6 }{ 5 } + \frac{ 10 }{ 35 }$
|
||||
\begin{solution}
|
||||
\begin{eqnarray*}
|
||||
C & = & \frac{ -6 }{ 5 } + \frac{ 10 }{ 35 } \\
|
||||
C & = & \frac{ -6 \times 7 }{ 5 \times 7 } + \frac{ 10 \times 1 }{ 35 \times 1 } \\
|
||||
C & = & \frac{ -42 }{ 35 } + \frac{ 10 }{ 35 } \\
|
||||
C & = & \frac{ -42 + 10 }{ 35 } \\
|
||||
C & = & \frac{ -32 }{ 35 }
|
||||
\end{eqnarray*}
|
||||
\end{solution}
|
||||
\end{subparts}
|
||||
\end{multicols}
|
||||
|
||||
\end{parts}
|
||||
|
||||
|
||||
\vfill
|
||||
|
||||
\question
|
||||
|
||||
|
||||
Dans la figure suivante, $(AB)$ et $(CD)$ sont parallèles, $AO = 3$, $OD = 6$, $CD = 10$ et $OB = 13$.
|
||||
|
||||
|
||||
\begin{minipage}{0.5\textwidth}
|
||||
\includegraphics[scale=0.4]{./fig/thales1}
|
||||
\end{minipage}
|
||||
\begin{minipage}{0.5\textwidth}
|
||||
Calculer les longueurs $AB$ et $BC$.
|
||||
\end{minipage}
|
||||
|
||||
|
||||
\vfill
|
||||
|
||||
\question
|
||||
|
||||
|
||||
|
||||
|
||||
|
||||
|
||||
Dans une urne, on a placé des boules colorées indiscernables au touché. Il y a 8 boules bleu, 10 boules jaunes, 8 boules vertes et 2 boules rouges.
|
||||
|
||||
\begin{parts}
|
||||
\part Quelle est la probabilité de tirer une boule bleu?
|
||||
\begin{solution}
|
||||
$\dfrac{8}{28} \approx 0.29$
|
||||
\end{solution}
|
||||
\part Quelle est la probabilté de tirer une boule jaune ou bleu?
|
||||
\begin{solution}
|
||||
$\dfrac{18}{28} \approx 0.64$
|
||||
\end{solution}
|
||||
\part A-t-on plus de chance de tirer une boule verte ou une boule rouge?
|
||||
\begin{solution}
|
||||
Boules vertes: $\dfrac{8}{28} \approx 0.29$
|
||||
|
||||
Boules rouges: $\dfrac{2}{28} \approx 0.07$
|
||||
|
||||
|
||||
Une boule verte
|
||||
|
||||
\end{solution}
|
||||
\end{parts}
|
||||
|
||||
|
||||
|
||||
|
||||
\end{questions}
|
||||
|
||||
\end{document}
|
||||
|
||||
%%% Local Variables:
|
||||
%%% mode: latex
|
||||
%%% TeX-master: "master"
|
||||
%%% End:
|
146
3e/DM/DM_16_11_14/308/15_DM_15_11_12_308.tex
Normal file
146
3e/DM/DM_16_11_14/308/15_DM_15_11_12_308.tex
Normal file
@@ -0,0 +1,146 @@
|
||||
\documentclass[a5paper,12pt, table]{/media/documents/Cours/Prof/Enseignements/2016-2017/tools/style/classDS}
|
||||
\usepackage{/media/documents/Cours/Prof/Enseignements/2016-2017/theme}
|
||||
%\geometry{left=10mm,right=10mm, top=10mm, bottom=10mm}
|
||||
|
||||
% Title Page
|
||||
\titre{1}
|
||||
% \seconde \premiereS \PSTMG \TSTMG
|
||||
\classe{Troisième}
|
||||
\date{lundi 14 novembre 2016}
|
||||
%\duree{1 heure}
|
||||
\sujet{15}
|
||||
% DS DSCorr DM DMCorr Corr
|
||||
\typedoc{DM}
|
||||
|
||||
\geometry{left=10mm,right=10mm, bottom= 10mm, top=10mm}
|
||||
%\printanswers
|
||||
|
||||
\begin{document}
|
||||
|
||||
\maketitle
|
||||
|
||||
\vspace{-1cm}
|
||||
Vous devez rendre le sujet avec la copie.
|
||||
|
||||
\begin{questions}
|
||||
|
||||
\vfill
|
||||
\question
|
||||
\begin{parts}
|
||||
\part Compléter les pointillés pour qu'il y est bien égalité.
|
||||
\hspace{-1cm}
|
||||
\begin{center}
|
||||
%
|
||||
$\dfrac{8}{2} = \dfrac{\ldots}{12}$
|
||||
\hfill
|
||||
%
|
||||
$\dfrac{8}{10} = \dfrac{\ldots}{30}$
|
||||
\hfill
|
||||
%
|
||||
$\dfrac{\cdots}{30} = \dfrac{5}{10}$
|
||||
\hfill
|
||||
%
|
||||
$\dfrac{2}{7} = \dfrac{6}{\cdots}$
|
||||
\end{center}
|
||||
|
||||
\vfill
|
||||
\part Faire les calculs suivants en détaillant les étapes (penser à simplifier les fractions quand c'est possible).
|
||||
\begin{multicols}{3}
|
||||
\begin{subparts}
|
||||
|
||||
\subpart $A = \frac{ 2 }{ 2 } + \frac{ 5 }{ 2 }$
|
||||
\begin{solution}
|
||||
\begin{eqnarray*}
|
||||
A & = & \frac{ 2 }{ 2 } + \frac{ 5 }{ 2 } \\
|
||||
A & = & \frac{ 2 + 5 }{ 2 } \\
|
||||
A & = & \frac{ 7 }{ 2 }
|
||||
\end{eqnarray*}
|
||||
\end{solution}
|
||||
|
||||
\subpart $B = \frac{ -3 }{ 8 } + \frac{ -9 }{ 8 }$
|
||||
\begin{solution}
|
||||
\begin{eqnarray*}
|
||||
B & = & \frac{ -3 }{ 8 } + \frac{ -9 }{ 8 } \\
|
||||
B & = & \frac{ -3 - 9 }{ 8 } \\
|
||||
B & = & \frac{ -12 }{ 8 } \\
|
||||
B & = & \frac{ -3 \times 4 }{ 2 \times 4 } \\
|
||||
B & = & \frac{ -3 }{ 2 }
|
||||
\end{eqnarray*}
|
||||
\end{solution}
|
||||
|
||||
\subpart $C = \frac{ 8 }{ 9 } + \frac{ 5 }{ 45 }$
|
||||
\begin{solution}
|
||||
\begin{eqnarray*}
|
||||
C & = & \frac{ 8 }{ 9 } + \frac{ 5 }{ 45 } \\
|
||||
C & = & \frac{ 8 \times 5 }{ 9 \times 5 } + \frac{ 5 \times 1 }{ 45 \times 1 } \\
|
||||
C & = & \frac{ 40 }{ 45 } + \frac{ 5 }{ 45 } \\
|
||||
C & = & \frac{ 40 + 5 }{ 45 } \\
|
||||
C & = & \frac{ 45 }{ 45 } \\
|
||||
C & = & 1
|
||||
\end{eqnarray*}
|
||||
\end{solution}
|
||||
\end{subparts}
|
||||
\end{multicols}
|
||||
|
||||
\end{parts}
|
||||
|
||||
|
||||
\vfill
|
||||
|
||||
\question
|
||||
|
||||
|
||||
Dans la figure suivante, $(AB)$ et $(CD)$ sont parallèles, $AO = 3$, $OD = 14$, $CD = 14$ et $OB = 6$.
|
||||
|
||||
|
||||
\begin{minipage}{0.5\textwidth}
|
||||
\includegraphics[scale=0.4]{./fig/thales1}
|
||||
\end{minipage}
|
||||
\begin{minipage}{0.5\textwidth}
|
||||
Calculer les longueurs $AB$ et $BC$.
|
||||
\end{minipage}
|
||||
|
||||
|
||||
\vfill
|
||||
|
||||
\question
|
||||
|
||||
|
||||
|
||||
|
||||
|
||||
|
||||
Dans une urne, on a placé des boules colorées indiscernables au touché. Il y a 6 boules bleu, 7 boules jaunes, 10 boules vertes et 4 boules rouges.
|
||||
|
||||
\begin{parts}
|
||||
\part Quelle est la probabilité de tirer une boule bleu?
|
||||
\begin{solution}
|
||||
$\dfrac{6}{27} \approx 0.22$
|
||||
\end{solution}
|
||||
\part Quelle est la probabilté de tirer une boule jaune ou bleu?
|
||||
\begin{solution}
|
||||
$\dfrac{13}{27} \approx 0.48$
|
||||
\end{solution}
|
||||
\part A-t-on plus de chance de tirer une boule verte ou une boule rouge?
|
||||
\begin{solution}
|
||||
Boules vertes: $\dfrac{10}{27} \approx 0.37$
|
||||
|
||||
Boules rouges: $\dfrac{4}{27} \approx 0.15$
|
||||
|
||||
|
||||
Une boule verte
|
||||
|
||||
\end{solution}
|
||||
\end{parts}
|
||||
|
||||
|
||||
|
||||
|
||||
\end{questions}
|
||||
|
||||
\end{document}
|
||||
|
||||
%%% Local Variables:
|
||||
%%% mode: latex
|
||||
%%% TeX-master: "master"
|
||||
%%% End:
|
149
3e/DM/DM_16_11_14/308/16_DM_15_11_12_308.tex
Normal file
149
3e/DM/DM_16_11_14/308/16_DM_15_11_12_308.tex
Normal file
@@ -0,0 +1,149 @@
|
||||
\documentclass[a5paper,12pt, table]{/media/documents/Cours/Prof/Enseignements/2016-2017/tools/style/classDS}
|
||||
\usepackage{/media/documents/Cours/Prof/Enseignements/2016-2017/theme}
|
||||
%\geometry{left=10mm,right=10mm, top=10mm, bottom=10mm}
|
||||
|
||||
% Title Page
|
||||
\titre{1}
|
||||
% \seconde \premiereS \PSTMG \TSTMG
|
||||
\classe{Troisième}
|
||||
\date{lundi 14 novembre 2016}
|
||||
%\duree{1 heure}
|
||||
\sujet{16}
|
||||
% DS DSCorr DM DMCorr Corr
|
||||
\typedoc{DM}
|
||||
|
||||
\geometry{left=10mm,right=10mm, bottom= 10mm, top=10mm}
|
||||
%\printanswers
|
||||
|
||||
\begin{document}
|
||||
|
||||
\maketitle
|
||||
|
||||
\vspace{-1cm}
|
||||
Vous devez rendre le sujet avec la copie.
|
||||
|
||||
\begin{questions}
|
||||
|
||||
\vfill
|
||||
\question
|
||||
\begin{parts}
|
||||
\part Compléter les pointillés pour qu'il y est bien égalité.
|
||||
\hspace{-1cm}
|
||||
\begin{center}
|
||||
%
|
||||
$\dfrac{2}{6} = \dfrac{\ldots}{18}$
|
||||
\hfill
|
||||
%
|
||||
$\dfrac{6}{7} = \dfrac{\ldots}{56}$
|
||||
\hfill
|
||||
%
|
||||
$\dfrac{\cdots}{4} = \dfrac{3}{2}$
|
||||
\hfill
|
||||
%
|
||||
$\dfrac{9}{2} = \dfrac{27}{\cdots}$
|
||||
\end{center}
|
||||
|
||||
\vfill
|
||||
\part Faire les calculs suivants en détaillant les étapes (penser à simplifier les fractions quand c'est possible).
|
||||
\begin{multicols}{3}
|
||||
\begin{subparts}
|
||||
|
||||
\subpart $A = \frac{ 7 }{ 6 } + \frac{ 2 }{ 6 }$
|
||||
\begin{solution}
|
||||
\begin{eqnarray*}
|
||||
A & = & \frac{ 7 }{ 6 } + \frac{ 2 }{ 6 } \\
|
||||
A & = & \frac{ 7 + 2 }{ 6 } \\
|
||||
A & = & \frac{ 9 }{ 6 } \\
|
||||
A & = & \frac{ 3 \times 3 }{ 2 \times 3 } \\
|
||||
A & = & \frac{ 3 }{ 2 }
|
||||
\end{eqnarray*}
|
||||
\end{solution}
|
||||
|
||||
\subpart $B = \frac{ -6 }{ 8 } + \frac{ -4 }{ 8 }$
|
||||
\begin{solution}
|
||||
\begin{eqnarray*}
|
||||
B & = & \frac{ -6 }{ 8 } + \frac{ -4 }{ 8 } \\
|
||||
B & = & \frac{ -6 - 4 }{ 8 } \\
|
||||
B & = & \frac{ -10 }{ 8 } \\
|
||||
B & = & \frac{ -5 \times 2 }{ 4 \times 2 } \\
|
||||
B & = & \frac{ -5 }{ 4 }
|
||||
\end{eqnarray*}
|
||||
\end{solution}
|
||||
|
||||
\subpart $C = \frac{ 7 }{ 7 } + \frac{ 6 }{ 14 }$
|
||||
\begin{solution}
|
||||
\begin{eqnarray*}
|
||||
C & = & \frac{ 7 }{ 7 } + \frac{ 6 }{ 14 } \\
|
||||
C & = & \frac{ 7 \times 2 }{ 7 \times 2 } + \frac{ 6 \times 1 }{ 14 \times 1 } \\
|
||||
C & = & \frac{ 14 }{ 14 } + \frac{ 6 }{ 14 } \\
|
||||
C & = & \frac{ 14 + 6 }{ 14 } \\
|
||||
C & = & \frac{ 20 }{ 14 } \\
|
||||
C & = & \frac{ 10 \times 2 }{ 7 \times 2 } \\
|
||||
C & = & \frac{ 10 }{ 7 }
|
||||
\end{eqnarray*}
|
||||
\end{solution}
|
||||
\end{subparts}
|
||||
\end{multicols}
|
||||
|
||||
\end{parts}
|
||||
|
||||
|
||||
\vfill
|
||||
|
||||
\question
|
||||
|
||||
|
||||
Dans la figure suivante, $(AB)$ et $(CD)$ sont parallèles, $AO = 4$, $OD = 10$, $CD = 4$ et $OB = 3$.
|
||||
|
||||
|
||||
\begin{minipage}{0.5\textwidth}
|
||||
\includegraphics[scale=0.4]{./fig/thales2}
|
||||
\end{minipage}
|
||||
\begin{minipage}{0.5\textwidth}
|
||||
Calculer les longueurs $AB$ et $BC$.
|
||||
\end{minipage}
|
||||
|
||||
|
||||
\vfill
|
||||
|
||||
\question
|
||||
|
||||
|
||||
|
||||
|
||||
|
||||
|
||||
Dans une urne, on a placé des boules colorées indiscernables au touché. Il y a 4 boules bleu, 5 boules jaunes, 4 boules vertes et 9 boules rouges.
|
||||
|
||||
\begin{parts}
|
||||
\part Quelle est la probabilité de tirer une boule bleu?
|
||||
\begin{solution}
|
||||
$\dfrac{4}{22} \approx 0.18$
|
||||
\end{solution}
|
||||
\part Quelle est la probabilté de tirer une boule jaune ou bleu?
|
||||
\begin{solution}
|
||||
$\dfrac{9}{22} \approx 0.41$
|
||||
\end{solution}
|
||||
\part A-t-on plus de chance de tirer une boule verte ou une boule rouge?
|
||||
\begin{solution}
|
||||
Boules vertes: $\dfrac{4}{22} \approx 0.18$
|
||||
|
||||
Boules rouges: $\dfrac{9}{22} \approx 0.41$
|
||||
|
||||
|
||||
Une boule rouge
|
||||
|
||||
\end{solution}
|
||||
\end{parts}
|
||||
|
||||
|
||||
|
||||
|
||||
\end{questions}
|
||||
|
||||
\end{document}
|
||||
|
||||
%%% Local Variables:
|
||||
%%% mode: latex
|
||||
%%% TeX-master: "master"
|
||||
%%% End:
|
148
3e/DM/DM_16_11_14/308/17_DM_15_11_12_308.tex
Normal file
148
3e/DM/DM_16_11_14/308/17_DM_15_11_12_308.tex
Normal file
@@ -0,0 +1,148 @@
|
||||
\documentclass[a5paper,12pt, table]{/media/documents/Cours/Prof/Enseignements/2016-2017/tools/style/classDS}
|
||||
\usepackage{/media/documents/Cours/Prof/Enseignements/2016-2017/theme}
|
||||
%\geometry{left=10mm,right=10mm, top=10mm, bottom=10mm}
|
||||
|
||||
% Title Page
|
||||
\titre{1}
|
||||
% \seconde \premiereS \PSTMG \TSTMG
|
||||
\classe{Troisième}
|
||||
\date{lundi 14 novembre 2016}
|
||||
%\duree{1 heure}
|
||||
\sujet{17}
|
||||
% DS DSCorr DM DMCorr Corr
|
||||
\typedoc{DM}
|
||||
|
||||
\geometry{left=10mm,right=10mm, bottom= 10mm, top=10mm}
|
||||
%\printanswers
|
||||
|
||||
\begin{document}
|
||||
|
||||
\maketitle
|
||||
|
||||
\vspace{-1cm}
|
||||
Vous devez rendre le sujet avec la copie.
|
||||
|
||||
\begin{questions}
|
||||
|
||||
\vfill
|
||||
\question
|
||||
\begin{parts}
|
||||
\part Compléter les pointillés pour qu'il y est bien égalité.
|
||||
\hspace{-1cm}
|
||||
\begin{center}
|
||||
%
|
||||
$\dfrac{10}{2} = \dfrac{\ldots}{14}$
|
||||
\hfill
|
||||
%
|
||||
$\dfrac{4}{7} = \dfrac{\ldots}{35}$
|
||||
\hfill
|
||||
%
|
||||
$\dfrac{\cdots}{72} = \dfrac{6}{8}$
|
||||
\hfill
|
||||
%
|
||||
$\dfrac{5}{8} = \dfrac{20}{\cdots}$
|
||||
\end{center}
|
||||
|
||||
\vfill
|
||||
\part Faire les calculs suivants en détaillant les étapes (penser à simplifier les fractions quand c'est possible).
|
||||
\begin{multicols}{3}
|
||||
\begin{subparts}
|
||||
|
||||
\subpart $A = \frac{ 4 }{ 7 } + \frac{ 3 }{ 7 }$
|
||||
\begin{solution}
|
||||
\begin{eqnarray*}
|
||||
A & = & \frac{ 4 }{ 7 } + \frac{ 3 }{ 7 } \\
|
||||
A & = & \frac{ 4 + 3 }{ 7 } \\
|
||||
A & = & \frac{ 7 }{ 7 } \\
|
||||
A & = & 1
|
||||
\end{eqnarray*}
|
||||
\end{solution}
|
||||
|
||||
\subpart $B = \frac{ 6 }{ 6 } + \frac{ 3 }{ 6 }$
|
||||
\begin{solution}
|
||||
\begin{eqnarray*}
|
||||
B & = & \frac{ 6 }{ 6 } + \frac{ 3 }{ 6 } \\
|
||||
B & = & \frac{ 6 + 3 }{ 6 } \\
|
||||
B & = & \frac{ 9 }{ 6 } \\
|
||||
B & = & \frac{ 3 \times 3 }{ 2 \times 3 } \\
|
||||
B & = & \frac{ 3 }{ 2 }
|
||||
\end{eqnarray*}
|
||||
\end{solution}
|
||||
|
||||
\subpart $C = \frac{ -10 }{ 2 } + \frac{ 2 }{ 14 }$
|
||||
\begin{solution}
|
||||
\begin{eqnarray*}
|
||||
C & = & \frac{ -10 }{ 2 } + \frac{ 2 }{ 14 } \\
|
||||
C & = & \frac{ -10 \times 7 }{ 2 \times 7 } + \frac{ 2 \times 1 }{ 14 \times 1 } \\
|
||||
C & = & \frac{ -70 }{ 14 } + \frac{ 2 }{ 14 } \\
|
||||
C & = & \frac{ -70 + 2 }{ 14 } \\
|
||||
C & = & \frac{ -68 }{ 14 } \\
|
||||
C & = & \frac{ -34 \times 2 }{ 7 \times 2 } \\
|
||||
C & = & \frac{ -34 }{ 7 }
|
||||
\end{eqnarray*}
|
||||
\end{solution}
|
||||
\end{subparts}
|
||||
\end{multicols}
|
||||
|
||||
\end{parts}
|
||||
|
||||
|
||||
\vfill
|
||||
|
||||
\question
|
||||
|
||||
|
||||
Dans la figure suivante, $(AB)$ et $(CD)$ sont parallèles, $AO = 2$, $OD = 20$, $CD = 6$ et $OB = 18$.
|
||||
|
||||
|
||||
\begin{minipage}{0.5\textwidth}
|
||||
\includegraphics[scale=0.4]{./fig/thales2}
|
||||
\end{minipage}
|
||||
\begin{minipage}{0.5\textwidth}
|
||||
Calculer les longueurs $AB$ et $BC$.
|
||||
\end{minipage}
|
||||
|
||||
|
||||
\vfill
|
||||
|
||||
\question
|
||||
|
||||
|
||||
|
||||
|
||||
|
||||
|
||||
Dans une urne, on a placé des boules colorées indiscernables au touché. Il y a 9 boules bleu, 4 boules jaunes, 5 boules vertes et 5 boules rouges.
|
||||
|
||||
\begin{parts}
|
||||
\part Quelle est la probabilité de tirer une boule bleu?
|
||||
\begin{solution}
|
||||
$\dfrac{9}{23} \approx 0.39$
|
||||
\end{solution}
|
||||
\part Quelle est la probabilté de tirer une boule jaune ou bleu?
|
||||
\begin{solution}
|
||||
$\dfrac{13}{23} \approx 0.57$
|
||||
\end{solution}
|
||||
\part A-t-on plus de chance de tirer une boule verte ou une boule rouge?
|
||||
\begin{solution}
|
||||
Boules vertes: $\dfrac{5}{23} \approx 0.22$
|
||||
|
||||
Boules rouges: $\dfrac{5}{23} \approx 0.22$
|
||||
|
||||
|
||||
Une boule rouge
|
||||
|
||||
\end{solution}
|
||||
\end{parts}
|
||||
|
||||
|
||||
|
||||
|
||||
\end{questions}
|
||||
|
||||
\end{document}
|
||||
|
||||
%%% Local Variables:
|
||||
%%% mode: latex
|
||||
%%% TeX-master: "master"
|
||||
%%% End:
|
148
3e/DM/DM_16_11_14/308/18_DM_15_11_12_308.tex
Normal file
148
3e/DM/DM_16_11_14/308/18_DM_15_11_12_308.tex
Normal file
@@ -0,0 +1,148 @@
|
||||
\documentclass[a5paper,12pt, table]{/media/documents/Cours/Prof/Enseignements/2016-2017/tools/style/classDS}
|
||||
\usepackage{/media/documents/Cours/Prof/Enseignements/2016-2017/theme}
|
||||
%\geometry{left=10mm,right=10mm, top=10mm, bottom=10mm}
|
||||
|
||||
% Title Page
|
||||
\titre{1}
|
||||
% \seconde \premiereS \PSTMG \TSTMG
|
||||
\classe{Troisième}
|
||||
\date{lundi 14 novembre 2016}
|
||||
%\duree{1 heure}
|
||||
\sujet{18}
|
||||
% DS DSCorr DM DMCorr Corr
|
||||
\typedoc{DM}
|
||||
|
||||
\geometry{left=10mm,right=10mm, bottom= 10mm, top=10mm}
|
||||
%\printanswers
|
||||
|
||||
\begin{document}
|
||||
|
||||
\maketitle
|
||||
|
||||
\vspace{-1cm}
|
||||
Vous devez rendre le sujet avec la copie.
|
||||
|
||||
\begin{questions}
|
||||
|
||||
\vfill
|
||||
\question
|
||||
\begin{parts}
|
||||
\part Compléter les pointillés pour qu'il y est bien égalité.
|
||||
\hspace{-1cm}
|
||||
\begin{center}
|
||||
%
|
||||
$\dfrac{2}{4} = \dfrac{\ldots}{20}$
|
||||
\hfill
|
||||
%
|
||||
$\dfrac{6}{10} = \dfrac{\ldots}{60}$
|
||||
\hfill
|
||||
%
|
||||
$\dfrac{\cdots}{24} = \dfrac{4}{3}$
|
||||
\hfill
|
||||
%
|
||||
$\dfrac{5}{8} = \dfrac{35}{\cdots}$
|
||||
\end{center}
|
||||
|
||||
\vfill
|
||||
\part Faire les calculs suivants en détaillant les étapes (penser à simplifier les fractions quand c'est possible).
|
||||
\begin{multicols}{3}
|
||||
\begin{subparts}
|
||||
|
||||
\subpart $A = \frac{ 2 }{ 9 } + \frac{ 10 }{ 9 }$
|
||||
\begin{solution}
|
||||
\begin{eqnarray*}
|
||||
A & = & \frac{ 2 }{ 9 } + \frac{ 10 }{ 9 } \\
|
||||
A & = & \frac{ 2 + 10 }{ 9 } \\
|
||||
A & = & \frac{ 12 }{ 9 } \\
|
||||
A & = & \frac{ 4 \times 3 }{ 3 \times 3 } \\
|
||||
A & = & \frac{ 4 }{ 3 }
|
||||
\end{eqnarray*}
|
||||
\end{solution}
|
||||
|
||||
\subpart $B = \frac{ -6 }{ 3 } + \frac{ -3 }{ 3 }$
|
||||
\begin{solution}
|
||||
\begin{eqnarray*}
|
||||
B & = & \frac{ -6 }{ 3 } + \frac{ -3 }{ 3 } \\
|
||||
B & = & \frac{ -6 - 3 }{ 3 } \\
|
||||
B & = & \frac{ -9 }{ 3 } \\
|
||||
B & = & -3
|
||||
\end{eqnarray*}
|
||||
\end{solution}
|
||||
|
||||
\subpart $C = \frac{ -4 }{ 2 } + \frac{ 8 }{ 20 }$
|
||||
\begin{solution}
|
||||
\begin{eqnarray*}
|
||||
C & = & \frac{ -4 }{ 2 } + \frac{ 8 }{ 20 } \\
|
||||
C & = & \frac{ -4 \times 10 }{ 2 \times 10 } + \frac{ 8 \times 1 }{ 20 \times 1 } \\
|
||||
C & = & \frac{ -40 }{ 20 } + \frac{ 8 }{ 20 } \\
|
||||
C & = & \frac{ -40 + 8 }{ 20 } \\
|
||||
C & = & \frac{ -32 }{ 20 } \\
|
||||
C & = & \frac{ -8 \times 4 }{ 5 \times 4 } \\
|
||||
C & = & \frac{ -8 }{ 5 }
|
||||
\end{eqnarray*}
|
||||
\end{solution}
|
||||
\end{subparts}
|
||||
\end{multicols}
|
||||
|
||||
\end{parts}
|
||||
|
||||
|
||||
\vfill
|
||||
|
||||
\question
|
||||
|
||||
|
||||
Dans la figure suivante, $(AB)$ et $(CD)$ sont parallèles, $AO = 4$, $OD = 18$, $CD = 17$ et $OB = 16$.
|
||||
|
||||
|
||||
\begin{minipage}{0.5\textwidth}
|
||||
\includegraphics[scale=0.4]{./fig/thales1}
|
||||
\end{minipage}
|
||||
\begin{minipage}{0.5\textwidth}
|
||||
Calculer les longueurs $AB$ et $BC$.
|
||||
\end{minipage}
|
||||
|
||||
|
||||
\vfill
|
||||
|
||||
\question
|
||||
|
||||
|
||||
|
||||
|
||||
|
||||
|
||||
Dans une urne, on a placé des boules colorées indiscernables au touché. Il y a 3 boules bleu, 3 boules jaunes, 2 boules vertes et 4 boules rouges.
|
||||
|
||||
\begin{parts}
|
||||
\part Quelle est la probabilité de tirer une boule bleu?
|
||||
\begin{solution}
|
||||
$\dfrac{3}{12} \approx 0.25$
|
||||
\end{solution}
|
||||
\part Quelle est la probabilté de tirer une boule jaune ou bleu?
|
||||
\begin{solution}
|
||||
$\dfrac{6}{12} \approx 0.5$
|
||||
\end{solution}
|
||||
\part A-t-on plus de chance de tirer une boule verte ou une boule rouge?
|
||||
\begin{solution}
|
||||
Boules vertes: $\dfrac{2}{12} \approx 0.17$
|
||||
|
||||
Boules rouges: $\dfrac{4}{12} \approx 0.33$
|
||||
|
||||
|
||||
Une boule rouge
|
||||
|
||||
\end{solution}
|
||||
\end{parts}
|
||||
|
||||
|
||||
|
||||
|
||||
\end{questions}
|
||||
|
||||
\end{document}
|
||||
|
||||
%%% Local Variables:
|
||||
%%% mode: latex
|
||||
%%% TeX-master: "master"
|
||||
%%% End:
|
147
3e/DM/DM_16_11_14/308/19_DM_15_11_12_308.tex
Normal file
147
3e/DM/DM_16_11_14/308/19_DM_15_11_12_308.tex
Normal file
@@ -0,0 +1,147 @@
|
||||
\documentclass[a5paper,12pt, table]{/media/documents/Cours/Prof/Enseignements/2016-2017/tools/style/classDS}
|
||||
\usepackage{/media/documents/Cours/Prof/Enseignements/2016-2017/theme}
|
||||
%\geometry{left=10mm,right=10mm, top=10mm, bottom=10mm}
|
||||
|
||||
% Title Page
|
||||
\titre{1}
|
||||
% \seconde \premiereS \PSTMG \TSTMG
|
||||
\classe{Troisième}
|
||||
\date{lundi 14 novembre 2016}
|
||||
%\duree{1 heure}
|
||||
\sujet{19}
|
||||
% DS DSCorr DM DMCorr Corr
|
||||
\typedoc{DM}
|
||||
|
||||
\geometry{left=10mm,right=10mm, bottom= 10mm, top=10mm}
|
||||
%\printanswers
|
||||
|
||||
\begin{document}
|
||||
|
||||
\maketitle
|
||||
|
||||
\vspace{-1cm}
|
||||
Vous devez rendre le sujet avec la copie.
|
||||
|
||||
\begin{questions}
|
||||
|
||||
\vfill
|
||||
\question
|
||||
\begin{parts}
|
||||
\part Compléter les pointillés pour qu'il y est bien égalité.
|
||||
\hspace{-1cm}
|
||||
\begin{center}
|
||||
%
|
||||
$\dfrac{3}{9} = \dfrac{\ldots}{90}$
|
||||
\hfill
|
||||
%
|
||||
$\dfrac{8}{7} = \dfrac{\ldots}{49}$
|
||||
\hfill
|
||||
%
|
||||
$\dfrac{\cdots}{100} = \dfrac{4}{10}$
|
||||
\hfill
|
||||
%
|
||||
$\dfrac{6}{4} = \dfrac{54}{\cdots}$
|
||||
\end{center}
|
||||
|
||||
\vfill
|
||||
\part Faire les calculs suivants en détaillant les étapes (penser à simplifier les fractions quand c'est possible).
|
||||
\begin{multicols}{3}
|
||||
\begin{subparts}
|
||||
|
||||
\subpart $A = \frac{ 9 }{ 6 } + \frac{ 3 }{ 6 }$
|
||||
\begin{solution}
|
||||
\begin{eqnarray*}
|
||||
A & = & \frac{ 9 }{ 6 } + \frac{ 3 }{ 6 } \\
|
||||
A & = & \frac{ 9 + 3 }{ 6 } \\
|
||||
A & = & \frac{ 12 }{ 6 } \\
|
||||
A & = & 2
|
||||
\end{eqnarray*}
|
||||
\end{solution}
|
||||
|
||||
\subpart $B = \frac{ -4 }{ 2 } + \frac{ -8 }{ 2 }$
|
||||
\begin{solution}
|
||||
\begin{eqnarray*}
|
||||
B & = & \frac{ -4 }{ 2 } + \frac{ -8 }{ 2 } \\
|
||||
B & = & \frac{ -4 - 8 }{ 2 } \\
|
||||
B & = & \frac{ -12 }{ 2 } \\
|
||||
B & = & -6
|
||||
\end{eqnarray*}
|
||||
\end{solution}
|
||||
|
||||
\subpart $C = \frac{ 3 }{ 3 } + \frac{ 6 }{ 21 }$
|
||||
\begin{solution}
|
||||
\begin{eqnarray*}
|
||||
C & = & \frac{ 3 }{ 3 } + \frac{ 6 }{ 21 } \\
|
||||
C & = & \frac{ 3 \times 7 }{ 3 \times 7 } + \frac{ 6 \times 1 }{ 21 \times 1 } \\
|
||||
C & = & \frac{ 21 }{ 21 } + \frac{ 6 }{ 21 } \\
|
||||
C & = & \frac{ 21 + 6 }{ 21 } \\
|
||||
C & = & \frac{ 27 }{ 21 } \\
|
||||
C & = & \frac{ 9 \times 3 }{ 7 \times 3 } \\
|
||||
C & = & \frac{ 9 }{ 7 }
|
||||
\end{eqnarray*}
|
||||
\end{solution}
|
||||
\end{subparts}
|
||||
\end{multicols}
|
||||
|
||||
\end{parts}
|
||||
|
||||
|
||||
\vfill
|
||||
|
||||
\question
|
||||
|
||||
|
||||
Dans la figure suivante, $(AB)$ et $(CD)$ sont parallèles, $AO = 10$, $OD = 16$, $CD = 7$ et $OB = 10$.
|
||||
|
||||
|
||||
\begin{minipage}{0.5\textwidth}
|
||||
\includegraphics[scale=0.4]{./fig/thales2}
|
||||
\end{minipage}
|
||||
\begin{minipage}{0.5\textwidth}
|
||||
Calculer les longueurs $AB$ et $BC$.
|
||||
\end{minipage}
|
||||
|
||||
|
||||
\vfill
|
||||
|
||||
\question
|
||||
|
||||
|
||||
|
||||
|
||||
|
||||
|
||||
Dans une urne, on a placé des boules colorées indiscernables au touché. Il y a 10 boules bleu, 2 boules jaunes, 9 boules vertes et 8 boules rouges.
|
||||
|
||||
\begin{parts}
|
||||
\part Quelle est la probabilité de tirer une boule bleu?
|
||||
\begin{solution}
|
||||
$\dfrac{10}{29} \approx 0.34$
|
||||
\end{solution}
|
||||
\part Quelle est la probabilté de tirer une boule jaune ou bleu?
|
||||
\begin{solution}
|
||||
$\dfrac{12}{29} \approx 0.41$
|
||||
\end{solution}
|
||||
\part A-t-on plus de chance de tirer une boule verte ou une boule rouge?
|
||||
\begin{solution}
|
||||
Boules vertes: $\dfrac{9}{29} \approx 0.31$
|
||||
|
||||
Boules rouges: $\dfrac{8}{29} \approx 0.28$
|
||||
|
||||
|
||||
Une boule verte
|
||||
|
||||
\end{solution}
|
||||
\end{parts}
|
||||
|
||||
|
||||
|
||||
|
||||
\end{questions}
|
||||
|
||||
\end{document}
|
||||
|
||||
%%% Local Variables:
|
||||
%%% mode: latex
|
||||
%%% TeX-master: "master"
|
||||
%%% End:
|
143
3e/DM/DM_16_11_14/308/20_DM_15_11_12_308.tex
Normal file
143
3e/DM/DM_16_11_14/308/20_DM_15_11_12_308.tex
Normal file
@@ -0,0 +1,143 @@
|
||||
\documentclass[a5paper,12pt, table]{/media/documents/Cours/Prof/Enseignements/2016-2017/tools/style/classDS}
|
||||
\usepackage{/media/documents/Cours/Prof/Enseignements/2016-2017/theme}
|
||||
%\geometry{left=10mm,right=10mm, top=10mm, bottom=10mm}
|
||||
|
||||
% Title Page
|
||||
\titre{1}
|
||||
% \seconde \premiereS \PSTMG \TSTMG
|
||||
\classe{Troisième}
|
||||
\date{lundi 14 novembre 2016}
|
||||
%\duree{1 heure}
|
||||
\sujet{20}
|
||||
% DS DSCorr DM DMCorr Corr
|
||||
\typedoc{DM}
|
||||
|
||||
\geometry{left=10mm,right=10mm, bottom= 10mm, top=10mm}
|
||||
%\printanswers
|
||||
|
||||
\begin{document}
|
||||
|
||||
\maketitle
|
||||
|
||||
\vspace{-1cm}
|
||||
Vous devez rendre le sujet avec la copie.
|
||||
|
||||
\begin{questions}
|
||||
|
||||
\vfill
|
||||
\question
|
||||
\begin{parts}
|
||||
\part Compléter les pointillés pour qu'il y est bien égalité.
|
||||
\hspace{-1cm}
|
||||
\begin{center}
|
||||
%
|
||||
$\dfrac{8}{6} = \dfrac{\ldots}{36}$
|
||||
\hfill
|
||||
%
|
||||
$\dfrac{2}{6} = \dfrac{\ldots}{30}$
|
||||
\hfill
|
||||
%
|
||||
$\dfrac{\cdots}{24} = \dfrac{2}{6}$
|
||||
\hfill
|
||||
%
|
||||
$\dfrac{3}{6} = \dfrac{30}{\cdots}$
|
||||
\end{center}
|
||||
|
||||
\vfill
|
||||
\part Faire les calculs suivants en détaillant les étapes (penser à simplifier les fractions quand c'est possible).
|
||||
\begin{multicols}{3}
|
||||
\begin{subparts}
|
||||
|
||||
\subpart $A = \frac{ 3 }{ 7 } + \frac{ 5 }{ 7 }$
|
||||
\begin{solution}
|
||||
\begin{eqnarray*}
|
||||
A & = & \frac{ 3 }{ 7 } + \frac{ 5 }{ 7 } \\
|
||||
A & = & \frac{ 3 + 5 }{ 7 } \\
|
||||
A & = & \frac{ 8 }{ 7 }
|
||||
\end{eqnarray*}
|
||||
\end{solution}
|
||||
|
||||
\subpart $B = \frac{ 2 }{ 3 } + \frac{ -4 }{ 3 }$
|
||||
\begin{solution}
|
||||
\begin{eqnarray*}
|
||||
B & = & \frac{ 2 }{ 3 } + \frac{ -4 }{ 3 } \\
|
||||
B & = & \frac{ 2 - 4 }{ 3 } \\
|
||||
B & = & \frac{ -2 }{ 3 }
|
||||
\end{eqnarray*}
|
||||
\end{solution}
|
||||
|
||||
\subpart $C = \frac{ 5 }{ 9 } + \frac{ 8 }{ 27 }$
|
||||
\begin{solution}
|
||||
\begin{eqnarray*}
|
||||
C & = & \frac{ 5 }{ 9 } + \frac{ 8 }{ 27 } \\
|
||||
C & = & \frac{ 5 \times 3 }{ 9 \times 3 } + \frac{ 8 \times 1 }{ 27 \times 1 } \\
|
||||
C & = & \frac{ 15 }{ 27 } + \frac{ 8 }{ 27 } \\
|
||||
C & = & \frac{ 15 + 8 }{ 27 } \\
|
||||
C & = & \frac{ 23 }{ 27 }
|
||||
\end{eqnarray*}
|
||||
\end{solution}
|
||||
\end{subparts}
|
||||
\end{multicols}
|
||||
|
||||
\end{parts}
|
||||
|
||||
|
||||
\vfill
|
||||
|
||||
\question
|
||||
|
||||
|
||||
Dans la figure suivante, $(AB)$ et $(CD)$ sont parallèles, $AO = 11$, $OD = 14$, $CD = 8$ et $OB = 4$.
|
||||
|
||||
|
||||
\begin{minipage}{0.5\textwidth}
|
||||
\includegraphics[scale=0.4]{./fig/thales2}
|
||||
\end{minipage}
|
||||
\begin{minipage}{0.5\textwidth}
|
||||
Calculer les longueurs $AB$ et $BC$.
|
||||
\end{minipage}
|
||||
|
||||
|
||||
\vfill
|
||||
|
||||
\question
|
||||
|
||||
|
||||
|
||||
|
||||
|
||||
|
||||
Dans une urne, on a placé des boules colorées indiscernables au touché. Il y a 6 boules bleu, 7 boules jaunes, 9 boules vertes et 9 boules rouges.
|
||||
|
||||
\begin{parts}
|
||||
\part Quelle est la probabilité de tirer une boule bleu?
|
||||
\begin{solution}
|
||||
$\dfrac{6}{31} \approx 0.19$
|
||||
\end{solution}
|
||||
\part Quelle est la probabilté de tirer une boule jaune ou bleu?
|
||||
\begin{solution}
|
||||
$\dfrac{13}{31} \approx 0.42$
|
||||
\end{solution}
|
||||
\part A-t-on plus de chance de tirer une boule verte ou une boule rouge?
|
||||
\begin{solution}
|
||||
Boules vertes: $\dfrac{9}{31} \approx 0.29$
|
||||
|
||||
Boules rouges: $\dfrac{9}{31} \approx 0.29$
|
||||
|
||||
|
||||
Une boule rouge
|
||||
|
||||
\end{solution}
|
||||
\end{parts}
|
||||
|
||||
|
||||
|
||||
|
||||
\end{questions}
|
||||
|
||||
\end{document}
|
||||
|
||||
%%% Local Variables:
|
||||
%%% mode: latex
|
||||
%%% TeX-master: "master"
|
||||
%%% End:
|
146
3e/DM/DM_16_11_14/308/21_DM_15_11_12_308.tex
Normal file
146
3e/DM/DM_16_11_14/308/21_DM_15_11_12_308.tex
Normal file
@@ -0,0 +1,146 @@
|
||||
\documentclass[a5paper,12pt, table]{/media/documents/Cours/Prof/Enseignements/2016-2017/tools/style/classDS}
|
||||
\usepackage{/media/documents/Cours/Prof/Enseignements/2016-2017/theme}
|
||||
%\geometry{left=10mm,right=10mm, top=10mm, bottom=10mm}
|
||||
|
||||
% Title Page
|
||||
\titre{1}
|
||||
% \seconde \premiereS \PSTMG \TSTMG
|
||||
\classe{Troisième}
|
||||
\date{lundi 14 novembre 2016}
|
||||
%\duree{1 heure}
|
||||
\sujet{21}
|
||||
% DS DSCorr DM DMCorr Corr
|
||||
\typedoc{DM}
|
||||
|
||||
\geometry{left=10mm,right=10mm, bottom= 10mm, top=10mm}
|
||||
%\printanswers
|
||||
|
||||
\begin{document}
|
||||
|
||||
\maketitle
|
||||
|
||||
\vspace{-1cm}
|
||||
Vous devez rendre le sujet avec la copie.
|
||||
|
||||
\begin{questions}
|
||||
|
||||
\vfill
|
||||
\question
|
||||
\begin{parts}
|
||||
\part Compléter les pointillés pour qu'il y est bien égalité.
|
||||
\hspace{-1cm}
|
||||
\begin{center}
|
||||
%
|
||||
$\dfrac{7}{8} = \dfrac{\ldots}{80}$
|
||||
\hfill
|
||||
%
|
||||
$\dfrac{9}{2} = \dfrac{\ldots}{10}$
|
||||
\hfill
|
||||
%
|
||||
$\dfrac{\cdots}{48} = \dfrac{3}{6}$
|
||||
\hfill
|
||||
%
|
||||
$\dfrac{6}{5} = \dfrac{48}{\cdots}$
|
||||
\end{center}
|
||||
|
||||
\vfill
|
||||
\part Faire les calculs suivants en détaillant les étapes (penser à simplifier les fractions quand c'est possible).
|
||||
\begin{multicols}{3}
|
||||
\begin{subparts}
|
||||
|
||||
\subpart $A = \frac{ 9 }{ 3 } + \frac{ 8 }{ 3 }$
|
||||
\begin{solution}
|
||||
\begin{eqnarray*}
|
||||
A & = & \frac{ 9 }{ 3 } + \frac{ 8 }{ 3 } \\
|
||||
A & = & \frac{ 9 + 8 }{ 3 } \\
|
||||
A & = & \frac{ 17 }{ 3 }
|
||||
\end{eqnarray*}
|
||||
\end{solution}
|
||||
|
||||
\subpart $B = \frac{ 10 }{ 9 } + \frac{ -10 }{ 9 }$
|
||||
\begin{solution}
|
||||
\begin{eqnarray*}
|
||||
B & = & \frac{ 10 }{ 9 } + \frac{ -10 }{ 9 } \\
|
||||
B & = & \frac{ 10 - 10 }{ 9 } \\
|
||||
B & = & \frac{ 0 }{ 9 } \\
|
||||
B & = & 0
|
||||
\end{eqnarray*}
|
||||
\end{solution}
|
||||
|
||||
\subpart $C = \frac{ 10 }{ 10 } + \frac{ 5 }{ 50 }$
|
||||
\begin{solution}
|
||||
\begin{eqnarray*}
|
||||
C & = & \frac{ 10 }{ 10 } + \frac{ 5 }{ 50 } \\
|
||||
C & = & \frac{ 10 \times 5 }{ 10 \times 5 } + \frac{ 5 \times 1 }{ 50 \times 1 } \\
|
||||
C & = & \frac{ 50 }{ 50 } + \frac{ 5 }{ 50 } \\
|
||||
C & = & \frac{ 50 + 5 }{ 50 } \\
|
||||
C & = & \frac{ 55 }{ 50 } \\
|
||||
C & = & \frac{ 11 \times 5 }{ 10 \times 5 } \\
|
||||
C & = & \frac{ 11 }{ 10 }
|
||||
\end{eqnarray*}
|
||||
\end{solution}
|
||||
\end{subparts}
|
||||
\end{multicols}
|
||||
|
||||
\end{parts}
|
||||
|
||||
|
||||
\vfill
|
||||
|
||||
\question
|
||||
|
||||
|
||||
Dans la figure suivante, $(AB)$ et $(CD)$ sont parallèles, $AO = 12$, $OD = 18$, $CD = 12$ et $OB = 10$.
|
||||
|
||||
|
||||
\begin{minipage}{0.5\textwidth}
|
||||
\includegraphics[scale=0.4]{./fig/thales1}
|
||||
\end{minipage}
|
||||
\begin{minipage}{0.5\textwidth}
|
||||
Calculer les longueurs $AB$ et $BC$.
|
||||
\end{minipage}
|
||||
|
||||
|
||||
\vfill
|
||||
|
||||
\question
|
||||
|
||||
|
||||
|
||||
|
||||
|
||||
|
||||
Dans une urne, on a placé des boules colorées indiscernables au touché. Il y a 2 boules bleu, 8 boules jaunes, 7 boules vertes et 6 boules rouges.
|
||||
|
||||
\begin{parts}
|
||||
\part Quelle est la probabilité de tirer une boule bleu?
|
||||
\begin{solution}
|
||||
$\dfrac{2}{23} \approx 0.09$
|
||||
\end{solution}
|
||||
\part Quelle est la probabilté de tirer une boule jaune ou bleu?
|
||||
\begin{solution}
|
||||
$\dfrac{10}{23} \approx 0.43$
|
||||
\end{solution}
|
||||
\part A-t-on plus de chance de tirer une boule verte ou une boule rouge?
|
||||
\begin{solution}
|
||||
Boules vertes: $\dfrac{7}{23} \approx 0.3$
|
||||
|
||||
Boules rouges: $\dfrac{6}{23} \approx 0.26$
|
||||
|
||||
|
||||
Une boule verte
|
||||
|
||||
\end{solution}
|
||||
\end{parts}
|
||||
|
||||
|
||||
|
||||
|
||||
\end{questions}
|
||||
|
||||
\end{document}
|
||||
|
||||
%%% Local Variables:
|
||||
%%% mode: latex
|
||||
%%% TeX-master: "master"
|
||||
%%% End:
|
147
3e/DM/DM_16_11_14/308/22_DM_15_11_12_308.tex
Normal file
147
3e/DM/DM_16_11_14/308/22_DM_15_11_12_308.tex
Normal file
@@ -0,0 +1,147 @@
|
||||
\documentclass[a5paper,12pt, table]{/media/documents/Cours/Prof/Enseignements/2016-2017/tools/style/classDS}
|
||||
\usepackage{/media/documents/Cours/Prof/Enseignements/2016-2017/theme}
|
||||
%\geometry{left=10mm,right=10mm, top=10mm, bottom=10mm}
|
||||
|
||||
% Title Page
|
||||
\titre{1}
|
||||
% \seconde \premiereS \PSTMG \TSTMG
|
||||
\classe{Troisième}
|
||||
\date{lundi 14 novembre 2016}
|
||||
%\duree{1 heure}
|
||||
\sujet{22}
|
||||
% DS DSCorr DM DMCorr Corr
|
||||
\typedoc{DM}
|
||||
|
||||
\geometry{left=10mm,right=10mm, bottom= 10mm, top=10mm}
|
||||
%\printanswers
|
||||
|
||||
\begin{document}
|
||||
|
||||
\maketitle
|
||||
|
||||
\vspace{-1cm}
|
||||
Vous devez rendre le sujet avec la copie.
|
||||
|
||||
\begin{questions}
|
||||
|
||||
\vfill
|
||||
\question
|
||||
\begin{parts}
|
||||
\part Compléter les pointillés pour qu'il y est bien égalité.
|
||||
\hspace{-1cm}
|
||||
\begin{center}
|
||||
%
|
||||
$\dfrac{5}{2} = \dfrac{\ldots}{10}$
|
||||
\hfill
|
||||
%
|
||||
$\dfrac{6}{4} = \dfrac{\ldots}{32}$
|
||||
\hfill
|
||||
%
|
||||
$\dfrac{\cdots}{28} = \dfrac{9}{7}$
|
||||
\hfill
|
||||
%
|
||||
$\dfrac{8}{3} = \dfrac{16}{\cdots}$
|
||||
\end{center}
|
||||
|
||||
\vfill
|
||||
\part Faire les calculs suivants en détaillant les étapes (penser à simplifier les fractions quand c'est possible).
|
||||
\begin{multicols}{3}
|
||||
\begin{subparts}
|
||||
|
||||
\subpart $A = \frac{ 9 }{ 6 } + \frac{ 7 }{ 6 }$
|
||||
\begin{solution}
|
||||
\begin{eqnarray*}
|
||||
A & = & \frac{ 9 }{ 6 } + \frac{ 7 }{ 6 } \\
|
||||
A & = & \frac{ 9 + 7 }{ 6 } \\
|
||||
A & = & \frac{ 16 }{ 6 } \\
|
||||
A & = & \frac{ 8 \times 2 }{ 3 \times 2 } \\
|
||||
A & = & \frac{ 8 }{ 3 }
|
||||
\end{eqnarray*}
|
||||
\end{solution}
|
||||
|
||||
\subpart $B = \frac{ -9 }{ 9 } + \frac{ 4 }{ 9 }$
|
||||
\begin{solution}
|
||||
\begin{eqnarray*}
|
||||
B & = & \frac{ -9 }{ 9 } + \frac{ 4 }{ 9 } \\
|
||||
B & = & \frac{ -9 + 4 }{ 9 } \\
|
||||
B & = & \frac{ -5 }{ 9 }
|
||||
\end{eqnarray*}
|
||||
\end{solution}
|
||||
|
||||
\subpart $C = \frac{ 3 }{ 10 } + \frac{ 10 }{ 80 }$
|
||||
\begin{solution}
|
||||
\begin{eqnarray*}
|
||||
C & = & \frac{ 3 }{ 10 } + \frac{ 10 }{ 80 } \\
|
||||
C & = & \frac{ 3 \times 8 }{ 10 \times 8 } + \frac{ 10 \times 1 }{ 80 \times 1 } \\
|
||||
C & = & \frac{ 24 }{ 80 } + \frac{ 10 }{ 80 } \\
|
||||
C & = & \frac{ 24 + 10 }{ 80 } \\
|
||||
C & = & \frac{ 34 }{ 80 } \\
|
||||
C & = & \frac{ 17 \times 2 }{ 40 \times 2 } \\
|
||||
C & = & \frac{ 17 }{ 40 }
|
||||
\end{eqnarray*}
|
||||
\end{solution}
|
||||
\end{subparts}
|
||||
\end{multicols}
|
||||
|
||||
\end{parts}
|
||||
|
||||
|
||||
\vfill
|
||||
|
||||
\question
|
||||
|
||||
|
||||
Dans la figure suivante, $(AB)$ et $(CD)$ sont parallèles, $AO = 11$, $OD = 14$, $CD = 5$ et $OB = 11$.
|
||||
|
||||
|
||||
\begin{minipage}{0.5\textwidth}
|
||||
\includegraphics[scale=0.4]{./fig/thales2}
|
||||
\end{minipage}
|
||||
\begin{minipage}{0.5\textwidth}
|
||||
Calculer les longueurs $AB$ et $BC$.
|
||||
\end{minipage}
|
||||
|
||||
|
||||
\vfill
|
||||
|
||||
\question
|
||||
|
||||
|
||||
|
||||
|
||||
|
||||
|
||||
Dans une urne, on a placé des boules colorées indiscernables au touché. Il y a 9 boules bleu, 8 boules jaunes, 3 boules vertes et 4 boules rouges.
|
||||
|
||||
\begin{parts}
|
||||
\part Quelle est la probabilité de tirer une boule bleu?
|
||||
\begin{solution}
|
||||
$\dfrac{9}{24} \approx 0.38$
|
||||
\end{solution}
|
||||
\part Quelle est la probabilté de tirer une boule jaune ou bleu?
|
||||
\begin{solution}
|
||||
$\dfrac{17}{24} \approx 0.71$
|
||||
\end{solution}
|
||||
\part A-t-on plus de chance de tirer une boule verte ou une boule rouge?
|
||||
\begin{solution}
|
||||
Boules vertes: $\dfrac{3}{24} \approx 0.12$
|
||||
|
||||
Boules rouges: $\dfrac{4}{24} \approx 0.17$
|
||||
|
||||
|
||||
Une boule rouge
|
||||
|
||||
\end{solution}
|
||||
\end{parts}
|
||||
|
||||
|
||||
|
||||
|
||||
\end{questions}
|
||||
|
||||
\end{document}
|
||||
|
||||
%%% Local Variables:
|
||||
%%% mode: latex
|
||||
%%% TeX-master: "master"
|
||||
%%% End:
|
147
3e/DM/DM_16_11_14/308/23_DM_15_11_12_308.tex
Normal file
147
3e/DM/DM_16_11_14/308/23_DM_15_11_12_308.tex
Normal file
@@ -0,0 +1,147 @@
|
||||
\documentclass[a5paper,12pt, table]{/media/documents/Cours/Prof/Enseignements/2016-2017/tools/style/classDS}
|
||||
\usepackage{/media/documents/Cours/Prof/Enseignements/2016-2017/theme}
|
||||
%\geometry{left=10mm,right=10mm, top=10mm, bottom=10mm}
|
||||
|
||||
% Title Page
|
||||
\titre{1}
|
||||
% \seconde \premiereS \PSTMG \TSTMG
|
||||
\classe{Troisième}
|
||||
\date{lundi 14 novembre 2016}
|
||||
%\duree{1 heure}
|
||||
\sujet{23}
|
||||
% DS DSCorr DM DMCorr Corr
|
||||
\typedoc{DM}
|
||||
|
||||
\geometry{left=10mm,right=10mm, bottom= 10mm, top=10mm}
|
||||
%\printanswers
|
||||
|
||||
\begin{document}
|
||||
|
||||
\maketitle
|
||||
|
||||
\vspace{-1cm}
|
||||
Vous devez rendre le sujet avec la copie.
|
||||
|
||||
\begin{questions}
|
||||
|
||||
\vfill
|
||||
\question
|
||||
\begin{parts}
|
||||
\part Compléter les pointillés pour qu'il y est bien égalité.
|
||||
\hspace{-1cm}
|
||||
\begin{center}
|
||||
%
|
||||
$\dfrac{4}{7} = \dfrac{\ldots}{35}$
|
||||
\hfill
|
||||
%
|
||||
$\dfrac{4}{3} = \dfrac{\ldots}{9}$
|
||||
\hfill
|
||||
%
|
||||
$\dfrac{\cdots}{20} = \dfrac{9}{4}$
|
||||
\hfill
|
||||
%
|
||||
$\dfrac{6}{10} = \dfrac{12}{\cdots}$
|
||||
\end{center}
|
||||
|
||||
\vfill
|
||||
\part Faire les calculs suivants en détaillant les étapes (penser à simplifier les fractions quand c'est possible).
|
||||
\begin{multicols}{3}
|
||||
\begin{subparts}
|
||||
|
||||
\subpart $A = \frac{ 9 }{ 6 } + \frac{ 6 }{ 6 }$
|
||||
\begin{solution}
|
||||
\begin{eqnarray*}
|
||||
A & = & \frac{ 9 }{ 6 } + \frac{ 6 }{ 6 } \\
|
||||
A & = & \frac{ 9 + 6 }{ 6 } \\
|
||||
A & = & \frac{ 15 }{ 6 } \\
|
||||
A & = & \frac{ 5 \times 3 }{ 2 \times 3 } \\
|
||||
A & = & \frac{ 5 }{ 2 }
|
||||
\end{eqnarray*}
|
||||
\end{solution}
|
||||
|
||||
\subpart $B = \frac{ 9 }{ 4 } + \frac{ -7 }{ 4 }$
|
||||
\begin{solution}
|
||||
\begin{eqnarray*}
|
||||
B & = & \frac{ 9 }{ 4 } + \frac{ -7 }{ 4 } \\
|
||||
B & = & \frac{ 9 - 7 }{ 4 } \\
|
||||
B & = & \frac{ 2 }{ 4 } \\
|
||||
B & = & \frac{ 1 \times 2 }{ 2 \times 2 } \\
|
||||
B & = & \frac{ 1 }{ 2 }
|
||||
\end{eqnarray*}
|
||||
\end{solution}
|
||||
|
||||
\subpart $C = \frac{ 2 }{ 5 } + \frac{ 9 }{ 25 }$
|
||||
\begin{solution}
|
||||
\begin{eqnarray*}
|
||||
C & = & \frac{ 2 }{ 5 } + \frac{ 9 }{ 25 } \\
|
||||
C & = & \frac{ 2 \times 5 }{ 5 \times 5 } + \frac{ 9 \times 1 }{ 25 \times 1 } \\
|
||||
C & = & \frac{ 10 }{ 25 } + \frac{ 9 }{ 25 } \\
|
||||
C & = & \frac{ 10 + 9 }{ 25 } \\
|
||||
C & = & \frac{ 19 }{ 25 }
|
||||
\end{eqnarray*}
|
||||
\end{solution}
|
||||
\end{subparts}
|
||||
\end{multicols}
|
||||
|
||||
\end{parts}
|
||||
|
||||
|
||||
\vfill
|
||||
|
||||
\question
|
||||
|
||||
|
||||
Dans la figure suivante, $(AB)$ et $(CD)$ sont parallèles, $AO = 5$, $OD = 9$, $CD = 7$ et $OB = 16$.
|
||||
|
||||
|
||||
\begin{minipage}{0.5\textwidth}
|
||||
\includegraphics[scale=0.4]{./fig/thales1}
|
||||
\end{minipage}
|
||||
\begin{minipage}{0.5\textwidth}
|
||||
Calculer les longueurs $AB$ et $BC$.
|
||||
\end{minipage}
|
||||
|
||||
|
||||
\vfill
|
||||
|
||||
\question
|
||||
|
||||
|
||||
|
||||
|
||||
|
||||
|
||||
Dans une urne, on a placé des boules colorées indiscernables au touché. Il y a 2 boules bleu, 10 boules jaunes, 6 boules vertes et 6 boules rouges.
|
||||
|
||||
\begin{parts}
|
||||
\part Quelle est la probabilité de tirer une boule bleu?
|
||||
\begin{solution}
|
||||
$\dfrac{2}{24} \approx 0.08$
|
||||
\end{solution}
|
||||
\part Quelle est la probabilté de tirer une boule jaune ou bleu?
|
||||
\begin{solution}
|
||||
$\dfrac{12}{24} \approx 0.5$
|
||||
\end{solution}
|
||||
\part A-t-on plus de chance de tirer une boule verte ou une boule rouge?
|
||||
\begin{solution}
|
||||
Boules vertes: $\dfrac{6}{24} \approx 0.25$
|
||||
|
||||
Boules rouges: $\dfrac{6}{24} \approx 0.25$
|
||||
|
||||
|
||||
Une boule rouge
|
||||
|
||||
\end{solution}
|
||||
\end{parts}
|
||||
|
||||
|
||||
|
||||
|
||||
\end{questions}
|
||||
|
||||
\end{document}
|
||||
|
||||
%%% Local Variables:
|
||||
%%% mode: latex
|
||||
%%% TeX-master: "master"
|
||||
%%% End:
|
145
3e/DM/DM_16_11_14/308/24_DM_15_11_12_308.tex
Normal file
145
3e/DM/DM_16_11_14/308/24_DM_15_11_12_308.tex
Normal file
@@ -0,0 +1,145 @@
|
||||
\documentclass[a5paper,12pt, table]{/media/documents/Cours/Prof/Enseignements/2016-2017/tools/style/classDS}
|
||||
\usepackage{/media/documents/Cours/Prof/Enseignements/2016-2017/theme}
|
||||
%\geometry{left=10mm,right=10mm, top=10mm, bottom=10mm}
|
||||
|
||||
% Title Page
|
||||
\titre{1}
|
||||
% \seconde \premiereS \PSTMG \TSTMG
|
||||
\classe{Troisième}
|
||||
\date{lundi 14 novembre 2016}
|
||||
%\duree{1 heure}
|
||||
\sujet{24}
|
||||
% DS DSCorr DM DMCorr Corr
|
||||
\typedoc{DM}
|
||||
|
||||
\geometry{left=10mm,right=10mm, bottom= 10mm, top=10mm}
|
||||
%\printanswers
|
||||
|
||||
\begin{document}
|
||||
|
||||
\maketitle
|
||||
|
||||
\vspace{-1cm}
|
||||
Vous devez rendre le sujet avec la copie.
|
||||
|
||||
\begin{questions}
|
||||
|
||||
\vfill
|
||||
\question
|
||||
\begin{parts}
|
||||
\part Compléter les pointillés pour qu'il y est bien égalité.
|
||||
\hspace{-1cm}
|
||||
\begin{center}
|
||||
%
|
||||
$\dfrac{10}{5} = \dfrac{\ldots}{15}$
|
||||
\hfill
|
||||
%
|
||||
$\dfrac{9}{3} = \dfrac{\ldots}{18}$
|
||||
\hfill
|
||||
%
|
||||
$\dfrac{\cdots}{63} = \dfrac{10}{9}$
|
||||
\hfill
|
||||
%
|
||||
$\dfrac{7}{6} = \dfrac{49}{\cdots}$
|
||||
\end{center}
|
||||
|
||||
\vfill
|
||||
\part Faire les calculs suivants en détaillant les étapes (penser à simplifier les fractions quand c'est possible).
|
||||
\begin{multicols}{3}
|
||||
\begin{subparts}
|
||||
|
||||
\subpart $A = \frac{ 9 }{ 2 } + \frac{ 7 }{ 2 }$
|
||||
\begin{solution}
|
||||
\begin{eqnarray*}
|
||||
A & = & \frac{ 9 }{ 2 } + \frac{ 7 }{ 2 } \\
|
||||
A & = & \frac{ 9 + 7 }{ 2 } \\
|
||||
A & = & \frac{ 16 }{ 2 } \\
|
||||
A & = & 8
|
||||
\end{eqnarray*}
|
||||
\end{solution}
|
||||
|
||||
\subpart $B = \frac{ -5 }{ 6 } + \frac{ 5 }{ 6 }$
|
||||
\begin{solution}
|
||||
\begin{eqnarray*}
|
||||
B & = & \frac{ -5 }{ 6 } + \frac{ 5 }{ 6 } \\
|
||||
B & = & \frac{ -5 + 5 }{ 6 } \\
|
||||
B & = & \frac{ 0 }{ 6 } \\
|
||||
B & = & 0
|
||||
\end{eqnarray*}
|
||||
\end{solution}
|
||||
|
||||
\subpart $C = \frac{ -7 }{ 4 } + \frac{ 3 }{ 40 }$
|
||||
\begin{solution}
|
||||
\begin{eqnarray*}
|
||||
C & = & \frac{ -7 }{ 4 } + \frac{ 3 }{ 40 } \\
|
||||
C & = & \frac{ -7 \times 10 }{ 4 \times 10 } + \frac{ 3 \times 1 }{ 40 \times 1 } \\
|
||||
C & = & \frac{ -70 }{ 40 } + \frac{ 3 }{ 40 } \\
|
||||
C & = & \frac{ -70 + 3 }{ 40 } \\
|
||||
C & = & \frac{ -67 }{ 40 }
|
||||
\end{eqnarray*}
|
||||
\end{solution}
|
||||
\end{subparts}
|
||||
\end{multicols}
|
||||
|
||||
\end{parts}
|
||||
|
||||
|
||||
\vfill
|
||||
|
||||
\question
|
||||
|
||||
|
||||
Dans la figure suivante, $(AB)$ et $(CD)$ sont parallèles, $AO = 4$, $OD = 19$, $CD = 15$ et $OB = 19$.
|
||||
|
||||
|
||||
\begin{minipage}{0.5\textwidth}
|
||||
\includegraphics[scale=0.4]{./fig/thales1}
|
||||
\end{minipage}
|
||||
\begin{minipage}{0.5\textwidth}
|
||||
Calculer les longueurs $AB$ et $BC$.
|
||||
\end{minipage}
|
||||
|
||||
|
||||
\vfill
|
||||
|
||||
\question
|
||||
|
||||
|
||||
|
||||
|
||||
|
||||
|
||||
Dans une urne, on a placé des boules colorées indiscernables au touché. Il y a 10 boules bleu, 3 boules jaunes, 2 boules vertes et 9 boules rouges.
|
||||
|
||||
\begin{parts}
|
||||
\part Quelle est la probabilité de tirer une boule bleu?
|
||||
\begin{solution}
|
||||
$\dfrac{10}{24} \approx 0.42$
|
||||
\end{solution}
|
||||
\part Quelle est la probabilté de tirer une boule jaune ou bleu?
|
||||
\begin{solution}
|
||||
$\dfrac{13}{24} \approx 0.54$
|
||||
\end{solution}
|
||||
\part A-t-on plus de chance de tirer une boule verte ou une boule rouge?
|
||||
\begin{solution}
|
||||
Boules vertes: $\dfrac{2}{24} \approx 0.08$
|
||||
|
||||
Boules rouges: $\dfrac{9}{24} \approx 0.38$
|
||||
|
||||
|
||||
Une boule rouge
|
||||
|
||||
\end{solution}
|
||||
\end{parts}
|
||||
|
||||
|
||||
|
||||
|
||||
\end{questions}
|
||||
|
||||
\end{document}
|
||||
|
||||
%%% Local Variables:
|
||||
%%% mode: latex
|
||||
%%% TeX-master: "master"
|
||||
%%% End:
|
147
3e/DM/DM_16_11_14/308/25_DM_15_11_12_308.tex
Normal file
147
3e/DM/DM_16_11_14/308/25_DM_15_11_12_308.tex
Normal file
@@ -0,0 +1,147 @@
|
||||
\documentclass[a5paper,12pt, table]{/media/documents/Cours/Prof/Enseignements/2016-2017/tools/style/classDS}
|
||||
\usepackage{/media/documents/Cours/Prof/Enseignements/2016-2017/theme}
|
||||
%\geometry{left=10mm,right=10mm, top=10mm, bottom=10mm}
|
||||
|
||||
% Title Page
|
||||
\titre{1}
|
||||
% \seconde \premiereS \PSTMG \TSTMG
|
||||
\classe{Troisième}
|
||||
\date{lundi 14 novembre 2016}
|
||||
%\duree{1 heure}
|
||||
\sujet{25}
|
||||
% DS DSCorr DM DMCorr Corr
|
||||
\typedoc{DM}
|
||||
|
||||
\geometry{left=10mm,right=10mm, bottom= 10mm, top=10mm}
|
||||
%\printanswers
|
||||
|
||||
\begin{document}
|
||||
|
||||
\maketitle
|
||||
|
||||
\vspace{-1cm}
|
||||
Vous devez rendre le sujet avec la copie.
|
||||
|
||||
\begin{questions}
|
||||
|
||||
\vfill
|
||||
\question
|
||||
\begin{parts}
|
||||
\part Compléter les pointillés pour qu'il y est bien égalité.
|
||||
\hspace{-1cm}
|
||||
\begin{center}
|
||||
%
|
||||
$\dfrac{2}{7} = \dfrac{\ldots}{70}$
|
||||
\hfill
|
||||
%
|
||||
$\dfrac{3}{4} = \dfrac{\ldots}{40}$
|
||||
\hfill
|
||||
%
|
||||
$\dfrac{\cdots}{28} = \dfrac{7}{4}$
|
||||
\hfill
|
||||
%
|
||||
$\dfrac{5}{3} = \dfrac{50}{\cdots}$
|
||||
\end{center}
|
||||
|
||||
\vfill
|
||||
\part Faire les calculs suivants en détaillant les étapes (penser à simplifier les fractions quand c'est possible).
|
||||
\begin{multicols}{3}
|
||||
\begin{subparts}
|
||||
|
||||
\subpart $A = \frac{ 8 }{ 6 } + \frac{ 8 }{ 6 }$
|
||||
\begin{solution}
|
||||
\begin{eqnarray*}
|
||||
A & = & \frac{ 8 }{ 6 } + \frac{ 8 }{ 6 } \\
|
||||
A & = & \frac{ 8 + 8 }{ 6 } \\
|
||||
A & = & \frac{ 16 }{ 6 } \\
|
||||
A & = & \frac{ 8 \times 2 }{ 3 \times 2 } \\
|
||||
A & = & \frac{ 8 }{ 3 }
|
||||
\end{eqnarray*}
|
||||
\end{solution}
|
||||
|
||||
\subpart $B = \frac{ 1 }{ 6 } + \frac{ 4 }{ 6 }$
|
||||
\begin{solution}
|
||||
\begin{eqnarray*}
|
||||
B & = & \frac{ 1 }{ 6 } + \frac{ 4 }{ 6 } \\
|
||||
B & = & \frac{ 1 + 4 }{ 6 } \\
|
||||
B & = & \frac{ 5 }{ 6 }
|
||||
\end{eqnarray*}
|
||||
\end{solution}
|
||||
|
||||
\subpart $C = \frac{ -3 }{ 9 } + \frac{ 4 }{ 36 }$
|
||||
\begin{solution}
|
||||
\begin{eqnarray*}
|
||||
C & = & \frac{ -3 }{ 9 } + \frac{ 4 }{ 36 } \\
|
||||
C & = & \frac{ -3 \times 4 }{ 9 \times 4 } + \frac{ 4 \times 1 }{ 36 \times 1 } \\
|
||||
C & = & \frac{ -12 }{ 36 } + \frac{ 4 }{ 36 } \\
|
||||
C & = & \frac{ -12 + 4 }{ 36 } \\
|
||||
C & = & \frac{ -8 }{ 36 } \\
|
||||
C & = & \frac{ -2 \times 4 }{ 9 \times 4 } \\
|
||||
C & = & \frac{ -2 }{ 9 }
|
||||
\end{eqnarray*}
|
||||
\end{solution}
|
||||
\end{subparts}
|
||||
\end{multicols}
|
||||
|
||||
\end{parts}
|
||||
|
||||
|
||||
\vfill
|
||||
|
||||
\question
|
||||
|
||||
|
||||
Dans la figure suivante, $(AB)$ et $(CD)$ sont parallèles, $AO = 2$, $OD = 13$, $CD = 19$ et $OB = 8$.
|
||||
|
||||
|
||||
\begin{minipage}{0.5\textwidth}
|
||||
\includegraphics[scale=0.4]{./fig/thales2}
|
||||
\end{minipage}
|
||||
\begin{minipage}{0.5\textwidth}
|
||||
Calculer les longueurs $AB$ et $BC$.
|
||||
\end{minipage}
|
||||
|
||||
|
||||
\vfill
|
||||
|
||||
\question
|
||||
|
||||
|
||||
|
||||
|
||||
|
||||
|
||||
Dans une urne, on a placé des boules colorées indiscernables au touché. Il y a 8 boules bleu, 5 boules jaunes, 10 boules vertes et 8 boules rouges.
|
||||
|
||||
\begin{parts}
|
||||
\part Quelle est la probabilité de tirer une boule bleu?
|
||||
\begin{solution}
|
||||
$\dfrac{8}{31} \approx 0.26$
|
||||
\end{solution}
|
||||
\part Quelle est la probabilté de tirer une boule jaune ou bleu?
|
||||
\begin{solution}
|
||||
$\dfrac{13}{31} \approx 0.42$
|
||||
\end{solution}
|
||||
\part A-t-on plus de chance de tirer une boule verte ou une boule rouge?
|
||||
\begin{solution}
|
||||
Boules vertes: $\dfrac{10}{31} \approx 0.32$
|
||||
|
||||
Boules rouges: $\dfrac{8}{31} \approx 0.26$
|
||||
|
||||
|
||||
Une boule verte
|
||||
|
||||
\end{solution}
|
||||
\end{parts}
|
||||
|
||||
|
||||
|
||||
|
||||
\end{questions}
|
||||
|
||||
\end{document}
|
||||
|
||||
%%% Local Variables:
|
||||
%%% mode: latex
|
||||
%%% TeX-master: "master"
|
||||
%%% End:
|
144
3e/DM/DM_16_11_14/308/26_DM_15_11_12_308.tex
Normal file
144
3e/DM/DM_16_11_14/308/26_DM_15_11_12_308.tex
Normal file
@@ -0,0 +1,144 @@
|
||||
\documentclass[a5paper,12pt, table]{/media/documents/Cours/Prof/Enseignements/2016-2017/tools/style/classDS}
|
||||
\usepackage{/media/documents/Cours/Prof/Enseignements/2016-2017/theme}
|
||||
%\geometry{left=10mm,right=10mm, top=10mm, bottom=10mm}
|
||||
|
||||
% Title Page
|
||||
\titre{1}
|
||||
% \seconde \premiereS \PSTMG \TSTMG
|
||||
\classe{Troisième}
|
||||
\date{lundi 14 novembre 2016}
|
||||
%\duree{1 heure}
|
||||
\sujet{26}
|
||||
% DS DSCorr DM DMCorr Corr
|
||||
\typedoc{DM}
|
||||
|
||||
\geometry{left=10mm,right=10mm, bottom= 10mm, top=10mm}
|
||||
%\printanswers
|
||||
|
||||
\begin{document}
|
||||
|
||||
\maketitle
|
||||
|
||||
\vspace{-1cm}
|
||||
Vous devez rendre le sujet avec la copie.
|
||||
|
||||
\begin{questions}
|
||||
|
||||
\vfill
|
||||
\question
|
||||
\begin{parts}
|
||||
\part Compléter les pointillés pour qu'il y est bien égalité.
|
||||
\hspace{-1cm}
|
||||
\begin{center}
|
||||
%
|
||||
$\dfrac{10}{7} = \dfrac{\ldots}{63}$
|
||||
\hfill
|
||||
%
|
||||
$\dfrac{3}{10} = \dfrac{\ldots}{70}$
|
||||
\hfill
|
||||
%
|
||||
$\dfrac{\cdots}{18} = \dfrac{10}{3}$
|
||||
\hfill
|
||||
%
|
||||
$\dfrac{2}{9} = \dfrac{16}{\cdots}$
|
||||
\end{center}
|
||||
|
||||
\vfill
|
||||
\part Faire les calculs suivants en détaillant les étapes (penser à simplifier les fractions quand c'est possible).
|
||||
\begin{multicols}{3}
|
||||
\begin{subparts}
|
||||
|
||||
\subpart $A = \frac{ 6 }{ 6 } + \frac{ 6 }{ 6 }$
|
||||
\begin{solution}
|
||||
\begin{eqnarray*}
|
||||
A & = & \frac{ 6 }{ 6 } + \frac{ 6 }{ 6 } \\
|
||||
A & = & \frac{ 6 + 6 }{ 6 } \\
|
||||
A & = & \frac{ 12 }{ 6 } \\
|
||||
A & = & 2
|
||||
\end{eqnarray*}
|
||||
\end{solution}
|
||||
|
||||
\subpart $B = \frac{ 3 }{ 5 } + \frac{ 8 }{ 5 }$
|
||||
\begin{solution}
|
||||
\begin{eqnarray*}
|
||||
B & = & \frac{ 3 }{ 5 } + \frac{ 8 }{ 5 } \\
|
||||
B & = & \frac{ 3 + 8 }{ 5 } \\
|
||||
B & = & \frac{ 11 }{ 5 }
|
||||
\end{eqnarray*}
|
||||
\end{solution}
|
||||
|
||||
\subpart $C = \frac{ -4 }{ 8 } + \frac{ 9 }{ 56 }$
|
||||
\begin{solution}
|
||||
\begin{eqnarray*}
|
||||
C & = & \frac{ -4 }{ 8 } + \frac{ 9 }{ 56 } \\
|
||||
C & = & \frac{ -4 \times 7 }{ 8 \times 7 } + \frac{ 9 \times 1 }{ 56 \times 1 } \\
|
||||
C & = & \frac{ -28 }{ 56 } + \frac{ 9 }{ 56 } \\
|
||||
C & = & \frac{ -28 + 9 }{ 56 } \\
|
||||
C & = & \frac{ -19 }{ 56 }
|
||||
\end{eqnarray*}
|
||||
\end{solution}
|
||||
\end{subparts}
|
||||
\end{multicols}
|
||||
|
||||
\end{parts}
|
||||
|
||||
|
||||
\vfill
|
||||
|
||||
\question
|
||||
|
||||
|
||||
Dans la figure suivante, $(AB)$ et $(CD)$ sont parallèles, $AO = 4$, $OD = 18$, $CD = 3$ et $OB = 5$.
|
||||
|
||||
|
||||
\begin{minipage}{0.5\textwidth}
|
||||
\includegraphics[scale=0.4]{./fig/thales2}
|
||||
\end{minipage}
|
||||
\begin{minipage}{0.5\textwidth}
|
||||
Calculer les longueurs $AB$ et $BC$.
|
||||
\end{minipage}
|
||||
|
||||
|
||||
\vfill
|
||||
|
||||
\question
|
||||
|
||||
|
||||
|
||||
|
||||
|
||||
|
||||
Dans une urne, on a placé des boules colorées indiscernables au touché. Il y a 10 boules bleu, 5 boules jaunes, 9 boules vertes et 2 boules rouges.
|
||||
|
||||
\begin{parts}
|
||||
\part Quelle est la probabilité de tirer une boule bleu?
|
||||
\begin{solution}
|
||||
$\dfrac{10}{26} \approx 0.38$
|
||||
\end{solution}
|
||||
\part Quelle est la probabilté de tirer une boule jaune ou bleu?
|
||||
\begin{solution}
|
||||
$\dfrac{15}{26} \approx 0.58$
|
||||
\end{solution}
|
||||
\part A-t-on plus de chance de tirer une boule verte ou une boule rouge?
|
||||
\begin{solution}
|
||||
Boules vertes: $\dfrac{9}{26} \approx 0.35$
|
||||
|
||||
Boules rouges: $\dfrac{2}{26} \approx 0.08$
|
||||
|
||||
|
||||
Une boule verte
|
||||
|
||||
\end{solution}
|
||||
\end{parts}
|
||||
|
||||
|
||||
|
||||
|
||||
\end{questions}
|
||||
|
||||
\end{document}
|
||||
|
||||
%%% Local Variables:
|
||||
%%% mode: latex
|
||||
%%% TeX-master: "master"
|
||||
%%% End:
|
145
3e/DM/DM_16_11_14/308/27_DM_15_11_12_308.tex
Normal file
145
3e/DM/DM_16_11_14/308/27_DM_15_11_12_308.tex
Normal file
@@ -0,0 +1,145 @@
|
||||
\documentclass[a5paper,12pt, table]{/media/documents/Cours/Prof/Enseignements/2016-2017/tools/style/classDS}
|
||||
\usepackage{/media/documents/Cours/Prof/Enseignements/2016-2017/theme}
|
||||
%\geometry{left=10mm,right=10mm, top=10mm, bottom=10mm}
|
||||
|
||||
% Title Page
|
||||
\titre{1}
|
||||
% \seconde \premiereS \PSTMG \TSTMG
|
||||
\classe{Troisième}
|
||||
\date{lundi 14 novembre 2016}
|
||||
%\duree{1 heure}
|
||||
\sujet{27}
|
||||
% DS DSCorr DM DMCorr Corr
|
||||
\typedoc{DM}
|
||||
|
||||
\geometry{left=10mm,right=10mm, bottom= 10mm, top=10mm}
|
||||
%\printanswers
|
||||
|
||||
\begin{document}
|
||||
|
||||
\maketitle
|
||||
|
||||
\vspace{-1cm}
|
||||
Vous devez rendre le sujet avec la copie.
|
||||
|
||||
\begin{questions}
|
||||
|
||||
\vfill
|
||||
\question
|
||||
\begin{parts}
|
||||
\part Compléter les pointillés pour qu'il y est bien égalité.
|
||||
\hspace{-1cm}
|
||||
\begin{center}
|
||||
%
|
||||
$\dfrac{2}{8} = \dfrac{\ldots}{32}$
|
||||
\hfill
|
||||
%
|
||||
$\dfrac{7}{5} = \dfrac{\ldots}{15}$
|
||||
\hfill
|
||||
%
|
||||
$\dfrac{\cdots}{35} = \dfrac{2}{7}$
|
||||
\hfill
|
||||
%
|
||||
$\dfrac{9}{7} = \dfrac{18}{\cdots}$
|
||||
\end{center}
|
||||
|
||||
\vfill
|
||||
\part Faire les calculs suivants en détaillant les étapes (penser à simplifier les fractions quand c'est possible).
|
||||
\begin{multicols}{3}
|
||||
\begin{subparts}
|
||||
|
||||
\subpart $A = \frac{ 2 }{ 6 } + \frac{ 1 }{ 6 }$
|
||||
\begin{solution}
|
||||
\begin{eqnarray*}
|
||||
A & = & \frac{ 2 }{ 6 } + \frac{ 1 }{ 6 } \\
|
||||
A & = & \frac{ 2 + 1 }{ 6 } \\
|
||||
A & = & \frac{ 3 }{ 6 } \\
|
||||
A & = & \frac{ 1 \times 3 }{ 2 \times 3 } \\
|
||||
A & = & \frac{ 1 }{ 2 }
|
||||
\end{eqnarray*}
|
||||
\end{solution}
|
||||
|
||||
\subpart $B = \frac{ 6 }{ 7 } + \frac{ -3 }{ 7 }$
|
||||
\begin{solution}
|
||||
\begin{eqnarray*}
|
||||
B & = & \frac{ 6 }{ 7 } + \frac{ -3 }{ 7 } \\
|
||||
B & = & \frac{ 6 - 3 }{ 7 } \\
|
||||
B & = & \frac{ 3 }{ 7 }
|
||||
\end{eqnarray*}
|
||||
\end{solution}
|
||||
|
||||
\subpart $C = \frac{ -2 }{ 5 } + \frac{ 7 }{ 15 }$
|
||||
\begin{solution}
|
||||
\begin{eqnarray*}
|
||||
C & = & \frac{ -2 }{ 5 } + \frac{ 7 }{ 15 } \\
|
||||
C & = & \frac{ -2 \times 3 }{ 5 \times 3 } + \frac{ 7 \times 1 }{ 15 \times 1 } \\
|
||||
C & = & \frac{ -6 }{ 15 } + \frac{ 7 }{ 15 } \\
|
||||
C & = & \frac{ -6 + 7 }{ 15 } \\
|
||||
C & = & \frac{ 1 }{ 15 }
|
||||
\end{eqnarray*}
|
||||
\end{solution}
|
||||
\end{subparts}
|
||||
\end{multicols}
|
||||
|
||||
\end{parts}
|
||||
|
||||
|
||||
\vfill
|
||||
|
||||
\question
|
||||
|
||||
|
||||
Dans la figure suivante, $(AB)$ et $(CD)$ sont parallèles, $AO = 10$, $OD = 17$, $CD = 9$ et $OB = 16$.
|
||||
|
||||
|
||||
\begin{minipage}{0.5\textwidth}
|
||||
\includegraphics[scale=0.4]{./fig/thales2}
|
||||
\end{minipage}
|
||||
\begin{minipage}{0.5\textwidth}
|
||||
Calculer les longueurs $AB$ et $BC$.
|
||||
\end{minipage}
|
||||
|
||||
|
||||
\vfill
|
||||
|
||||
\question
|
||||
|
||||
|
||||
|
||||
|
||||
|
||||
|
||||
Dans une urne, on a placé des boules colorées indiscernables au touché. Il y a 10 boules bleu, 6 boules jaunes, 3 boules vertes et 4 boules rouges.
|
||||
|
||||
\begin{parts}
|
||||
\part Quelle est la probabilité de tirer une boule bleu?
|
||||
\begin{solution}
|
||||
$\dfrac{10}{23} \approx 0.43$
|
||||
\end{solution}
|
||||
\part Quelle est la probabilté de tirer une boule jaune ou bleu?
|
||||
\begin{solution}
|
||||
$\dfrac{16}{23} \approx 0.7$
|
||||
\end{solution}
|
||||
\part A-t-on plus de chance de tirer une boule verte ou une boule rouge?
|
||||
\begin{solution}
|
||||
Boules vertes: $\dfrac{3}{23} \approx 0.13$
|
||||
|
||||
Boules rouges: $\dfrac{4}{23} \approx 0.17$
|
||||
|
||||
|
||||
Une boule rouge
|
||||
|
||||
\end{solution}
|
||||
\end{parts}
|
||||
|
||||
|
||||
|
||||
|
||||
\end{questions}
|
||||
|
||||
\end{document}
|
||||
|
||||
%%% Local Variables:
|
||||
%%% mode: latex
|
||||
%%% TeX-master: "master"
|
||||
%%% End:
|
148
3e/DM/DM_16_11_14/308/28_DM_15_11_12_308.tex
Normal file
148
3e/DM/DM_16_11_14/308/28_DM_15_11_12_308.tex
Normal file
@@ -0,0 +1,148 @@
|
||||
\documentclass[a5paper,12pt, table]{/media/documents/Cours/Prof/Enseignements/2016-2017/tools/style/classDS}
|
||||
\usepackage{/media/documents/Cours/Prof/Enseignements/2016-2017/theme}
|
||||
%\geometry{left=10mm,right=10mm, top=10mm, bottom=10mm}
|
||||
|
||||
% Title Page
|
||||
\titre{1}
|
||||
% \seconde \premiereS \PSTMG \TSTMG
|
||||
\classe{Troisième}
|
||||
\date{lundi 14 novembre 2016}
|
||||
%\duree{1 heure}
|
||||
\sujet{28}
|
||||
% DS DSCorr DM DMCorr Corr
|
||||
\typedoc{DM}
|
||||
|
||||
\geometry{left=10mm,right=10mm, bottom= 10mm, top=10mm}
|
||||
%\printanswers
|
||||
|
||||
\begin{document}
|
||||
|
||||
\maketitle
|
||||
|
||||
\vspace{-1cm}
|
||||
Vous devez rendre le sujet avec la copie.
|
||||
|
||||
\begin{questions}
|
||||
|
||||
\vfill
|
||||
\question
|
||||
\begin{parts}
|
||||
\part Compléter les pointillés pour qu'il y est bien égalité.
|
||||
\hspace{-1cm}
|
||||
\begin{center}
|
||||
%
|
||||
$\dfrac{2}{6} = \dfrac{\ldots}{18}$
|
||||
\hfill
|
||||
%
|
||||
$\dfrac{6}{7} = \dfrac{\ldots}{28}$
|
||||
\hfill
|
||||
%
|
||||
$\dfrac{\cdots}{6} = \dfrac{6}{2}$
|
||||
\hfill
|
||||
%
|
||||
$\dfrac{3}{4} = \dfrac{30}{\cdots}$
|
||||
\end{center}
|
||||
|
||||
\vfill
|
||||
\part Faire les calculs suivants en détaillant les étapes (penser à simplifier les fractions quand c'est possible).
|
||||
\begin{multicols}{3}
|
||||
\begin{subparts}
|
||||
|
||||
\subpart $A = \frac{ 2 }{ 9 } + \frac{ 10 }{ 9 }$
|
||||
\begin{solution}
|
||||
\begin{eqnarray*}
|
||||
A & = & \frac{ 2 }{ 9 } + \frac{ 10 }{ 9 } \\
|
||||
A & = & \frac{ 2 + 10 }{ 9 } \\
|
||||
A & = & \frac{ 12 }{ 9 } \\
|
||||
A & = & \frac{ 4 \times 3 }{ 3 \times 3 } \\
|
||||
A & = & \frac{ 4 }{ 3 }
|
||||
\end{eqnarray*}
|
||||
\end{solution}
|
||||
|
||||
\subpart $B = \frac{ 6 }{ 10 } + \frac{ -6 }{ 10 }$
|
||||
\begin{solution}
|
||||
\begin{eqnarray*}
|
||||
B & = & \frac{ 6 }{ 10 } + \frac{ -6 }{ 10 } \\
|
||||
B & = & \frac{ 6 - 6 }{ 10 } \\
|
||||
B & = & \frac{ 0 }{ 10 } \\
|
||||
B & = & 0
|
||||
\end{eqnarray*}
|
||||
\end{solution}
|
||||
|
||||
\subpart $C = \frac{ -1 }{ 8 } + \frac{ 4 }{ 80 }$
|
||||
\begin{solution}
|
||||
\begin{eqnarray*}
|
||||
C & = & \frac{ -1 }{ 8 } + \frac{ 4 }{ 80 } \\
|
||||
C & = & \frac{ -1 \times 10 }{ 8 \times 10 } + \frac{ 4 \times 1 }{ 80 \times 1 } \\
|
||||
C & = & \frac{ -10 }{ 80 } + \frac{ 4 }{ 80 } \\
|
||||
C & = & \frac{ -10 + 4 }{ 80 } \\
|
||||
C & = & \frac{ -6 }{ 80 } \\
|
||||
C & = & \frac{ -3 \times 2 }{ 40 \times 2 } \\
|
||||
C & = & \frac{ -3 }{ 40 }
|
||||
\end{eqnarray*}
|
||||
\end{solution}
|
||||
\end{subparts}
|
||||
\end{multicols}
|
||||
|
||||
\end{parts}
|
||||
|
||||
|
||||
\vfill
|
||||
|
||||
\question
|
||||
|
||||
|
||||
Dans la figure suivante, $(AB)$ et $(CD)$ sont parallèles, $AO = 12$, $OD = 20$, $CD = 10$ et $OB = 19$.
|
||||
|
||||
|
||||
\begin{minipage}{0.5\textwidth}
|
||||
\includegraphics[scale=0.4]{./fig/thales1}
|
||||
\end{minipage}
|
||||
\begin{minipage}{0.5\textwidth}
|
||||
Calculer les longueurs $AB$ et $BC$.
|
||||
\end{minipage}
|
||||
|
||||
|
||||
\vfill
|
||||
|
||||
\question
|
||||
|
||||
|
||||
|
||||
|
||||
|
||||
|
||||
Dans une urne, on a placé des boules colorées indiscernables au touché. Il y a 10 boules bleu, 2 boules jaunes, 6 boules vertes et 10 boules rouges.
|
||||
|
||||
\begin{parts}
|
||||
\part Quelle est la probabilité de tirer une boule bleu?
|
||||
\begin{solution}
|
||||
$\dfrac{10}{28} \approx 0.36$
|
||||
\end{solution}
|
||||
\part Quelle est la probabilté de tirer une boule jaune ou bleu?
|
||||
\begin{solution}
|
||||
$\dfrac{12}{28} \approx 0.43$
|
||||
\end{solution}
|
||||
\part A-t-on plus de chance de tirer une boule verte ou une boule rouge?
|
||||
\begin{solution}
|
||||
Boules vertes: $\dfrac{6}{28} \approx 0.21$
|
||||
|
||||
Boules rouges: $\dfrac{10}{28} \approx 0.36$
|
||||
|
||||
|
||||
Une boule rouge
|
||||
|
||||
\end{solution}
|
||||
\end{parts}
|
||||
|
||||
|
||||
|
||||
|
||||
\end{questions}
|
||||
|
||||
\end{document}
|
||||
|
||||
%%% Local Variables:
|
||||
%%% mode: latex
|
||||
%%% TeX-master: "master"
|
||||
%%% End:
|
BIN
3e/DM/DM_16_11_14/308/all_DM_15_11_12_308.pdf
Normal file
BIN
3e/DM/DM_16_11_14/308/all_DM_15_11_12_308.pdf
Normal file
Binary file not shown.
BIN
3e/DM/DM_16_11_14/308/corr_DM_15_11_12_308.pdf
Normal file
BIN
3e/DM/DM_16_11_14/308/corr_DM_15_11_12_308.pdf
Normal file
Binary file not shown.
BIN
3e/DM/DM_16_11_14/308/fig/thales1.pdf
Normal file
BIN
3e/DM/DM_16_11_14/308/fig/thales1.pdf
Normal file
Binary file not shown.
BIN
3e/DM/DM_16_11_14/308/fig/thales2.pdf
Normal file
BIN
3e/DM/DM_16_11_14/308/fig/thales2.pdf
Normal file
Binary file not shown.
51
3e/DM/DM_16_11_14/308/tpl_DM_15_11_12_308.tex
Normal file
51
3e/DM/DM_16_11_14/308/tpl_DM_15_11_12_308.tex
Normal file
@@ -0,0 +1,51 @@
|
||||
\documentclass[a5paper,12pt, table]{/media/documents/Cours/Prof/Enseignements/2016-2017/tools/style/classDS}
|
||||
\usepackage{/media/documents/Cours/Prof/Enseignements/2016-2017/theme}
|
||||
%\geometry{left=10mm,right=10mm, top=10mm, bottom=10mm}
|
||||
|
||||
% Title Page
|
||||
\titre{1}
|
||||
% \seconde \premiereS \PSTMG \TSTMG
|
||||
\classe{Troisième}
|
||||
\date{lundi 14 novembre 2016}
|
||||
%\duree{1 heure}
|
||||
\sujet{\Var{infos.num}}
|
||||
% DS DSCorr DM DMCorr Corr
|
||||
\typedoc{DM}
|
||||
|
||||
\geometry{left=10mm,right=10mm, bottom= 10mm, top=10mm}
|
||||
%\printanswers
|
||||
|
||||
\begin{document}
|
||||
|
||||
\maketitle
|
||||
|
||||
\vspace{-1cm}
|
||||
Vous devez rendre le sujet avec la copie.
|
||||
|
||||
\begin{questions}
|
||||
|
||||
\vfill
|
||||
\question
|
||||
\Block{include "./exo_tech_fractions_308.tex"}
|
||||
|
||||
\vfill
|
||||
|
||||
\question
|
||||
\Block{include "./exo_tech_thales.tex"}
|
||||
|
||||
\vfill
|
||||
|
||||
\question
|
||||
\Block{include "./exo_proba.tex"}
|
||||
|
||||
|
||||
|
||||
\end{questions}
|
||||
|
||||
\end{document}
|
||||
|
||||
%%% Local Variables:
|
||||
%%% mode: latex
|
||||
%%% TeX-master: "master"
|
||||
%%% End:
|
||||
|
Some files were not shown because too many files have changed in this diff Show More
Reference in New Issue
Block a user